You are on page 1of 93

Year - 2020 – 21

STRAIGHT LINES &


PAIR OF STRAIGHT LINES

AMBITION + LEARNER

CONTENTS

1. Theory (Straight Line) ............................ 01 - 42

2. Theory (Pair of Straight Line) ............................ 43 - 53

3. Exercise-1 (Elementary) ............................ 54 - 57

4. Exercise-2 (Main) ............................ 58 - 72

5. Exercise-3 (Advanced) ............................ 73 - 90

6. Answer Key ............................ 91 - 92


STRAIGHT LINES
COORDINATE SYSTEMS
There are basically three coordinate systems we use in 2-Dimensional geomery.
(1) Rectangular / Cartesian co-ordinate system
(2) Oblique co-ordinate system
(3) Polar co-ordinate system

(1) RECTANGULAR SYSTEM


We draw two mutually perpendicular straight lines X OX and Y OY
in a plane intersecting at O. The point O is called the origin. The horizontal
line X OX is called X-axis and the vertical line Y OY is called Y-axis.
Any point P in this plane can be uniquely represented by specifying its
distance from the two lines X OX and Y OY .

The distance of the point P from the y-axis, i.e. PM = ON = x is called


x-co-ordinte (or abscissa) and the distance of the point P from the x-axis,
i.e. PN = OM = y is called y-co-ordinate (or ordinate) of the point P. The
ordered pair (x, y) is called the co-ordinates of the point P.

Distances measured along x-axis right to the origin are taken positive and left to the origin are taken
negative. Similarly, distances measured along y-axis above the origin are taken positive and below the
origin are taken negative.

The co-ordinate axes X OX and Y OY divide the plane into four parts, called QUADRANTS number
I, II, III and IV. Thus

In I Quadrant, x > 0 and y > 0


In II Quadrant, x < 0 and y > 0
In III Quadrant, x < 0 and y < 0
In IV Quadrant, x > 0 and y < 0

The abscissa of a point on Y-axis is zero and the ordinate of the point on X-axis is zero. Thus co-ordinates
of origin O are clearly (0, 0).
(2) OBLIQUE COORDINATE SYSTEM
:

In this coordinate system co-ordinate axes X OX and YOY 


are at angle other the 90o.
Let P be any point in the plane draw lines PM and PN from P
parallel to reference lines YOY and X OX respectively..
Then the lengths PM and PN may be called co-ordinates of the point P.
They can also be called y and x co-ordinates respectively. We call
x-coordinate as abscissa and y-coordinate as ordinate of the point P.

MOMENTUM : Betiahata Chowk, GKP. PH. 0551- 2332808, 2205569 Page # 1


POLAR SYSTEM
In this we choose an initial line OX (O origin).
If P be any point such that OP = r, POX   then
( r , ) are called polar co-ordinates of the point P. Note that
r and ( r  0,0    2  ) uniquely define the position of the point P..
The length r is called radius vector and angle  is called vecotrial angle of P..
The radius vector is positive if it is measured from the origin O along the line bounding the vectorial angle;
if measured in the opposite direction it is negative.
Conversion of rectangular cartesion coordinates to polar coordinates and its converse.

Let P be any point whose coordinates are (x,y) and ( r , ) Y


respectively in rectangular cartesion and polar co-ordinat
system (with O as pole and OX as initial line). P
Draw PM perpendicular to OX so that we have
OM = x, MP = y,  MOP we have
y
x = OM = OP cos  MOP = r cos  ............(1),
y = MP = OP sin  MOP = r sin  ................(2), 
O X
x M
2 2 2 2
r = OP = OM  MP  x  y .........(3),

and MP y ........................................(4).
tan   
OM x

DISTANCE FORMULA
Let P and Q be two points with coordinates (x1y1) and (x2,y2) then distance
between them is,
PQ = ( x 2  x1 ) 2  ( y 2  y 1 ) 2
* In oblique co-ordinate system
If the axes are at angle  then distance between the points P ( x1 , y1 ) and Q ( x 2 , y 2 ) is given by..
PQ  ( x 1  x 2 ) 2  ( y 1  y 2 ) 2  2 ( x 1  x 2 )( y 1  y 2 ) cos 
* In polar co-ordinate system
The distance between two points P ( r1 , 1 ) and Q( r2 ,  2 ) is given by..

PQ  r12  r22  2 r1r2 cos(1   2 )

TAP THE CONCEPTS - I

Example 1 :- Find the distance between the points whose co-ordinates are
(i) (2,3) and (5,7) (ii) (3,–2) and (–6, 7) axes are inclined at 600
0 0
(iii) (2,30 ) and (4,120 ) (iv) (–3,450) and (7,1050)
Solution :- (i) d  ( 2  5) 2  ( 3  7) 2  5

(ii) d  ( 3  6) 2  ( 2  7) 2  2 ( 3  6)( 2  7 ) cos 60 0 .  63


MOMENTUM : Betiahata Chowk, GKP. PH. 0551- 2332808, 2205569 Page # 2
(iii) Here r1  2, r2  4 , 1  300 ,  2  120
2 2 0 0 2
 d  2  4  2.2.4 cos( 30  120 )  5
(iv) Here (–3, 450)  (3,2250)
Hence, r1  3, 1  2250 , r2  7,  2  1050

 d  32  7 2  2.3.7.cos( 2250  1050 )  79 .


Example 2 :- If the point P(x,y) is equidistant from the points A(a + b, b – a) and B(a – b, a + b) then prove that
bx = ay
Solution :- given PA = PB  PA2 = PB2
 [ x  (a  b)]2  [ y  ( b  a )]2  [ x  (a  b)]2  [ xy  (a  b)]2
  ( a  b) x  ( b  a ) y  2 x( a  b)  2 y( a  b)  bx  ay
Example 3 :- Show that the points A (2,1), B(5,4), C(4,7) and D (1,4) are the vertices of a parallelogram.
Solution :- AB  (5  2 ) 2  ( 4  1) 2  18 BC  ( 4  5) 2  ( 7  4 ) 2  10

CD  (1  4 ) 2  ( 4  7 ) 2  18 DA  ( 2  1) 2  (1  4 ) 2  10
therefore AB = CD and BC = DA hence the point A, B, D must be vertices of a parallelogram.
Example 4 :- Let S be a square of unit area. Consider any quadrilateral which has one vertex on each sides of S.
If a, b, c and d denote the lengths of the sides of the quadrilateral, prove that
2  a 2  b 2  c 2  d 2  4.
Solution :- Let S be a square of unit area with vertices
R(r,1) B(1,1)
O(0,0), A(1,0), B(1,1) and C(0,1). C
(0,1)
Let PQRL be the quadrilateral with vertices c
P(p,0), Q(1,q) R(r,1) and L(0,l) Q(1,q)
L d
 K  a2  b2  c2  d2 (0,l) b
= [(p2 + l2) + (1–P)2 + q2 + (1–q)2 + (1–r)2 + r2 + 1–l)2] a
2 2 2 2
 2[ p  q  r  l  p  q  r  l  2]
(0,0) O P(p,0) A(1,0)
 2[( p  1 / 2) 2  (q  1 / 2) 2  ( r  1 / 2)2  (l  1 / 2) 2  1]

F 1I 1
 0  p  1 0  Gp  J 
2

H 2K 4
F 1I 1 F 1I 2

Similarly 0  GH q  JK  , 0  GH r  JK
2
1 FG 1 IJ 2
1
2 4 2 4 H
 , 0 
2 K 
4
Thus. 2  a 2  b 2  c 2  d 2  4.
Example 5 :- The distance between two parallel lines is unity. A point P lies between the lines at a distance a from
one of them. find the length of a side of an equilateral  PQR vertex Q of which lies an one of the
parallel lines & vertex an other line
Solution :
R  (0,0), P  ( , ), Q  (,   600 )
 sin   1  a ,  sin( 60   )  a
 3 cos    sin   2a

MOMENTUM : Betiahata Chowk, GKP. PH. 0551- 2332808, 2205569 Page # 3


a 1
 3 cos  1  a  2a   cos 
3

b cos g  b sin  g F a  1I
G
2

H 3 JK
2 2
  (1  a ) 2
2

Example 6 :- If O is the origin and the coordinates of A and B are ( x1 , y1 ) and ( x 2 , y 2 ) respectively, prove that
OA. OB cos ( AOB)  x1 x 2  y1y 2 .
Solution :- We have (OA ) 2  x12  y12 and (OB) 2  x 22  y 22 . ,y 2)
B( x 2
 (AB) 2  ( x 2  x1 ) 2  ( y 2  y1 ) 2

c h c h
 x12  y12  x 22  y 22  2 x1 x 2  2 y1 y 2
O
A(x1,y2)
 (OA ) 2  (OB) 2  2( x1x 2  y1y 2 )
(OA ) 2  (OB) 2  (AB) 2
Now, cos( AOB) 
2(OA ) (OB)
 OA OB cos( AOB)  x1x 2  y1 y 2

SECTION FORMULA
Let P (x,y) be a point on the line joining the two points A (x1,y1) and B (x2,y2)
PA m
dividing it, in the ratio of m : n,   . i.e.
PB n
(i) If P lies within the points A and B, then P is said to divide AB internally.
mx 2  nx1 my 2  ny1
We have x = and y =
m n m n
(ii) If P lies outside the points A and B, i.e. on the line AB extended
then P is said to divide AB externally.
mx 2  nx1 my 2  ny1
We have x = and y =
m n m n
NOTE : The straight line ax + by + c = 0 divides the line joining

the points (x1,y1) and (x2,y2) in the ratio of  :1 where   


LM ax  by  c OP
1 1

N ax  by  c Q
2 2

If two points (x1,y1) and (x2,y2 ) are on same side of a line ax + by + c = 0 then  is negative (external
division)
ax1  by1  c ax  by1  c
Hence :  0 1 0
ax 2  by 2  c ax 2  by 2  c
therefore ax1  by1  c and ax 2  by 2  c will be of same sign.
and if two points (x1,y1) and (x2,y2 ) are on the opposite side of a line ax + by + c = 0 then
 is positive (internal division)

MOMENTUM : Betiahata Chowk, GKP. PH. 0551- 2332808, 2205569 Page # 4


ax1  by1  c ax  by1  c
Hence :  0 1 0
ax 2  by 2  c ax 2  by 2  c
therefore ax1  by1  c and ax 2  by 2  c will be of opposite sign.

AREA
Area of Triangle :
Let ABC be a triangle such that the coordinates of its vertieces are A ( x1 , y1 ), B( x 2 , y 2 ) and C(x 3 , y 3 ).
1
Area of triangle ABC = | x1 ( y 2  y 3 )  x 2 ( y 3  y1 )  x 3 ( y1  y 2 )|
2

x1 y1 1
1
 | x2 y 2 1|
2
x3 y3 1

1
NOTE : Area of triangle whose vertices are (0,0), (x1,y1) and (x2,y2) = |( x1 y 2  y1 x 2 )|
2
* If (x1,y1), (x2,y2), (x3,y3), (x4,y4), ........., (xn,yn) are the vertices of a regular polygon of n sides in an
LM
x 1 y1 x 2 y 2 x 3 y 3 x 4 y 4 xn yn OP
order then area of polygon = x y  x y  x y  x y ..........  x y
2 2N 3 3 4 4 5 5 1 1 Q
x1 y1
x2 y2
x3 y3 1
Trick : Above area = = ( x y  x 2 y 2 )  ( x 2 y 3  x 2 y 3  x 3 y 2 ) ....... ( x n y1  x1 y n )
2 1 2
xn yn
x1 y1

* Area of a triangle formed by the lines whose equations are


a 1x  b 1y  c1  0 , a 2 x  b 2 y  c 2  0 an d a 3x  b 3 y  c 3  0
a1 b1 c1
2
A= where   a 2 b2 c 2 and C1, C2, C3 are the cofactors of c1, c2, c3.
2| C1C2 C3 |
a3 b3 c3
* If the points A(x1,y1), B(x2,y2), C(x3,y3) are collinear then area ( ABC ) = 0
x1 y1 1
 x2 y2 10
x3 y3 1

MOMENTUM : Betiahata Chowk, GKP. PH. 0551- 2332808, 2205569 Page # 5


TAP THE CONCEPTS - II

Example 7 :- Find the point which divides the join of (2,3) and (5,–3) in the ratio 1 : 2.
2  2  51 3  2  ( 3)  1
Solution :- x  3, y   1  The required point is (3,1).
1 2 1 2
Example 8 :- In what ratio does y-axis divide the line segment joining A (–3,5) and B (2,7).
AP x 3
Solution :- If AB cuts y-axis at P such that    1  . But as P lies on y-axis the abscissa of P must
PB x2 2
be zero.
2k  3
 0
k 1
3
k . Thus the required ratio is 3 : 2.
2
AP x 3
Altenative : Equation of y-axis is x = 0   1 
PB x2 2
Example 9 :- Find the area of the quadrilateral whose vertices are A (2,2), B (–2,3), C(–3,–3) and D(1,2).
Solution :- The area of quadrilateral  Area ABC  Area ACD
2 2 1 2 2 1
1 1 25 15
 | 2 3 1 | | 3 3 1|    20 square units
2 2 2 2
3 3 1 1 2 1
Example 10 :- If the area of triangle formed by the points A(x,y), B(1,2) and C(2,1) be 6,then prove that
x + y =  12
Solution :- given, area = 6
x y 1
1
 | 1 2 1 |= 6  | x  y |  12  x  y   12
2
2 1 1
Example 11 :- For what values of the parameter  does the point M (,   1) lies within the triangle ABC the
vertices of which are A (0,3), B(–2,0) and C(6,1)?
Solution :- The point M will be inside the triangle if and only if
Area ( MBC )Area ( MCA ) Area ( MAB) Area ( AB)| ...............(1)

  1 1
1 1
Area ( MBC)  | 2 0 1 |  ( 7  6)
2 2
6 1 1

  1 1
1 1
Area ( MCA )  | 6 1 1 |  ( 8  12)
2 2
0 3 1

MOMENTUM : Betiahata Chowk, GKP. PH. 0551- 2332808, 2205569 Page # 6


 3 1
1 1
Area ( MAB)  | 0 0 1 |  (  4)
2 2
2 1 1

0 3 1
1 1
Area( ABC) | 2 0 1 | .22
2 2
6 1 1
From equation (1) |7  6||8  12||   4|  22
6 3 FG IJ
 The desired values of  lie in the interval  , .
7 2 H K
Example 12 :- Find the point of intersection of line segment joining (2,3), (4,1) and (1,2), (4,3)
FG 4  1 , 3  2 IJ also it is FG 4  2 ,   3IJ
Solution :- point P is
H  1  1 K H   1   1K (1,2)
1
(4,1)

1
(4,3)
4  1 4  2 1  3
hence   2  3  1    P
 1  1 2 P
3  2   3 1  (2,3)
and   2    1   
 1  1 2
1  3 1   1
     1 or  =   1 is not possible
2 2 3
F 4. 1 + 2 1  3 I
F 4 + 2 ,   3IJ = G 3 , 3 J  FG 5 , 5 IJ
= G
 Required point
H  + 1   1K GG 1 + 1 1  1JJ H 2 2 K
H3 3 K
Example 13 :- Find the co-ordinates of the vertices of a triangle if the mid-points of its sides are the points
(6,–1), (–1,–2) and (1,–4)
x2  x3 x  x1 x  x2
Solution :-  6, 3  1, 1 1 A(x1,y1)
2 2 2
 x 2  x 3  12, x 3  x1  2, x1  x 2  2 (1,-4)F E(-1,-2)
 2( x1  x 2  x 3 )  12 or x1  x 2  x 3  6
 x1  6, x 2  8, x 3  4 B C
Similarly y1  5, y 2  3, y 3  1 (x 2,y2) D(6,-1) (x 3,y3)

Hence the vertices are (–6, –5), (8, –3) and (4, 1)
Example 14 :- The area of a triangle is 5. Two of its vertices are (2,1) and (3,–2), the third lies on y = x + 3.
Find the co-ordinates of the third vertex can be
Solution :- Let the third vertex is (x , x+3).
x x3 1 FG  3 , 3IJ or FG 7 , 13IJ
given, 1
2
|2 1
1
1 |  5  | 4 x  4 |  5  x   3 2 , 7 2 hence,
2 H 2 2K H 2 2 K
3 2 1

MOMENTUM : Betiahata Chowk, GKP. PH. 0551- 2332808, 2205569 Page # 7


Example 15:- The coordinates of three points O,A and B are (0,0), (0,4) and (6,0), respectively. If a point P
moves so that area of POA is always twice the area of  POB , then find the locus of P..
1
Solution :- Let P(h, k) be the moving point. Then the area of  POA  | h.4  k.0 |  | 2 h |, and that of area
2

b POBg  21 | h.0  k.6 |  |  3k |, given area b POA g  2 area bPOBg


 |2 h|  2|3k| orh   ( 3k )
 h  3k  0 or h  3k  0
Example 16:- The points (k, 2 – 2k), (–k+1, 2k) and (– 4 – k, 6 –2k) are collinear , then find k
Solution :- The given points are collinear.

k 2  2k 1 k 2  2k 1
 k  1 2k 1  0  2 k  1 4k  2 0  0 [R 2  R 2  R1 , R 3  R 3  R1 ]
4  k 6  2k 1 4  2 k 4 0

 4( 2 k  1)  ( 4  2 k )( 4 k  2)  0  k  1 or k  1 / 2
Example 17:- If  1 is the area of the triangle with vertices (0,0), ( a tan  , b cot  ), ( a sin  , b cos  ),  2 is the
area of the triangle with vertices (a,b), (a sec2  , b cos ec 2  ), (a  a sin 2  , b  b cos2  ), and  3
is the area of the triangle with vertices (0,0), ( a tan  , b cot  ), ( a sin  , b cos  ), Show that
there is no value of  for which  1 ,  2 and  3 are in G.P..

0 0 1
1 1
Solution :- We have  1  | a tan  b cot  1 |  ab|sin   cos  |
2 2
a sin  b cos 1

a b 1
1
and  2  | a sec 2  2
b cosec  1|
2
a  a sin 2  b  b cos2  1

0 0 1
1 1
 ab| tan2  2
cot  1|  ab sin2   cos  Applying C  C  aC andC  C  bC
2 2 1 1 3 2 2 3
sin2  cos2  1

0 0 1
1 1
and  3  | a tan   b cot  1  ab|sin   cos |
2 2
a sin  b cos  1

1 1 1
Hence  1 3  ab 2 . Now if  1 ,  2 and  3 are G.P.  1 3  22  ab 2  22   2  ab
2 2 2
1 1
 ab sin 2   cos2   ab  sin 2   cos2   1    m 2 , m  I
2 2

MOMENTUM : Betiahata Chowk, GKP. PH. 0551- 2332808, 2205569 Page # 8


But for his value of , the vertices of the given triangle are not defined. Hence  1 ,  2 and  3
cannot be in G.P. for any value of  .

Example 18:- A line L intersects the three sides BC, CA and AB of a triangle ABC at P, Q and R, respectively.
BP . CQ . AR
Show that is algebraically equal to -1
PC Q A RB
Solution :- Let A ( x1 , y1 ), B( x 2 , y 2 ) and C( x 3 , y 3 ) be the vertices of  ABC, and let l x + my + n = 0 be the
equation of the line L.
P lx  my 2  n
 2 ............... (1)
PC lx 3  my 3  n
CQ lx  my 3  n
 3 ( 2)
QA lx1  my1  n
AR lx  my1  n
and  1 (3)
RB lx 2  my 2  n
BP CQ AR
Multiplying (1), (2) and (3) weget, . .  1
PC QA RB
Example 19:- A 1 , A 2 , A 3 ,.... A n are n points in a plane whose coordinates are ( x1 , y1 ),( x 2 , y 2 ),....., ( x n , y n ),
re-spectively. A 1A 2 is bisected at the point G1 , G 1 A 3 is divided in the ratio 1 : 2 at G 2 , G 2 A 4 is
divided in the ratio 1 : 3 at G 3 , G 3 A 5 is divided in the ratio 1 : 4 at G 4 , and so on unit all n points are
exhausted. Show that the coordinates of the final point so obtained are
FG 1 (x  x ..... x ), 1 (y  y ..... y )IJ
Hn 1 2
n
n
K 1 2 n

The coordinates of G are GH


F x  x , y  y IJ
2 K
1 2 1 2
Solution :- 12

G 2G 1 1 FG FG
1 2( x1  x 2 ) 1 2( y1  y 2 ) IJ FG IJ IJ = FG x  x 2  x 3 y1  y 2  y 3 IJ
H H K H KK H K
1
given,   G2   x3 ,  y3 ,
G 2G 3 2 3 2 3 2 3 3
Again, G 3 dividesG 2 A 4 in the ratio 1 : 3. Therefore, the coordinates of G 3 are
FG 1 FG 3( x  x  x )  x IJ , 1 FG 3( y  y  y )  y IJ IJ
H4H 3 K 4H 3 KK
1 2 3 1 2 3
G3  4 4

FG x  x  x  x , y  y  y y IJ
H K
1 2 3 4 1 2 3 4
or
4 4
Example 20:- The line joining A (b cos , b sin  ) and B (a cos , a sin  ) is produced to the point M (x,y) so
   
that AM : MB = b :a, then calculate x cos  y sin .
2 2
AM
Solution :-  b a
MB

MOMENTUM : Betiahata Chowk, GKP. PH. 0551- 2332808, 2205569 Page # 9


b( a cos )  a ( b cos  ) b( a sin )  a ( b sin  )
x and y 
ba ba
  
x cos   cos  2 sin sin
  2 2  
 y sin   sin  2 cos        x cos  y sin  0.
sin 2 2
2 2
Example 21 :- Prove that all the points inside the triangle formed by the points (1,3),(5,0) and (–1,2) satisfy
equations 3x  2 y  0 and 2 x  3y  12  0 .
Solution : 3x+2y  0 for (1, 3), (5, 0) and (–1, 2)
2x–3y–12  0 for (1, 3), (5, 0) and (–1, 2)
If all the vertices satisfy any inequality then all points inside the triangle also satisfy the inequality.
Example22:- OPQR is a square and M, N are the middle points of the sides PQ and QR respectively then
calculator the ratio of the areas of the square and the triangle OMN.
N(a/2, a)
R(0, a) Q (a, a)
0 0 1
1 3a 2
Solution : Area of the  OMN  | a a / 2 1| 
2 8 M(a, a/2)
a/2 a 1
Area of the square = a2  Hence the ratio is 8 : 3. O(0, 0) P(a, 0)

Example 23:- The four points A (  ,0), B(,0), C(  ,0) and ( ,0) are such that  ,  are the roots of
ax 2  2 hx  b  0 and  ,  are the roots of a 1x 2  2 h1x  b1  0.
Show that the sum of the ratios in which C and D divide AB is zero if ab1  a 1 b  2 hh1 .

Solution :- Let C divide AB in the ratio m : 1 and D in the ratio n : 1.


  m   n  
Then  ,   m    ,n   
1 m 1 n
   
given, m + n = 0   0 or (  )(    )  2(   )
   

FG 2h IJ FG 2h IJ  FG b  b IJ
H a KH a K Ha a K
1 1
 or 2 hh1  ab1  a 1b.
1 1

SLOPE OF STRAIGHT LINE


The angle  formed by the straight line with the positve direction of X-axis and measured counter-clockwise is
called the INCLINATION of the line. Obviously 00    1800 .

If  is the inclination then tan  defined as the slope of the straight line
and denoted by m. Thus Slope of the line, m = tan  ,   900 .
The slope of any line parallel to y-axis is not defined.
if 00    900 , then m  0and if 900    1800 then m < 0.
* The slope of line joining two points ( x1 , y1 ) and (x 2 , y 2 )

MOMENTUM : Betiahata Chowk, GKP. PH. 0551- 2332808, 2205569 Page # 10


y2  y1
is equal to x  x (x1  x2 )
2 1

BM y 2  y1
since slope of line AB = tan   
AM x 2  x1
EQUATIONS OF STRAIGHT LINE
1. Point slope form : If a line passes through A ( x1 , y1 ) and has slope m, its equation will be
y  y1  m ( x  x1 )
y 2  y1
2. Two point form : If a line passes through A ( x1 , y1 ) and B (x 2 , y 2 ) then its slope =
x 2  x1

y 2  y1
Hence the equation of line is y – y1 = x  x ( x  x1 )
2 1

3. Determinant form : The equation of a line passing through two points A ( x1 , y1 ) and B (x 2 , y 2 ) can
also be put in the form
x y 1
x1 y1 1 0
x 2 y2 1

4. Slope y- intercept form :


Let a line cut x and y-axis at A and B respectively. Then actual x-co-
ordinate of A (positive or negative) is callled x-intercept of the line while
actual y-co-ordinate of B is called y-intercept of line. If OB = c then B is (o,c)
(0,c).
Let the slope of the line be m, then the equation of line is
y – c = m (x –0) [point –slope form]
 y = mx + c.
5. Slope x-intercept form : If x-intercept of line is a and slope is m,
Y
then its equation is given as,  y  m( x  a )
6. Intercept Form : If a line cuts intercepts a and b from x and y axes respectively
(0,b)
b0 b
Since the line passes through two points (a,0) and (0,b). Slope = 
0a a (a,0)
Now from point slope form the equation of the line O X
b x y
y–0=– ( x  a ) or ay + bx = ab or   1
a a b
7. Normal form : Let the length of the perpendicular from origin on a line be p (p>0) and the angle which the
perpendicular makes with positive x-axis b is  .
In the adjoining figure OM = p,  MOA  
we have OA = p sec , OB = p cosec 
x y
 Equation of line AB is  p [intercept form]
p sec  p cos ec
MOMENTUM : Betiahata Chowk, GKP. PH. 0551- 2332808, 2205569 Page # 11
cos > 0, sin > 0, p>0 cos < 0, sin > 0, p>0

Parametric form :-
To find the equation of a straight line passing through a given point A(x1,y1) and makes a given angle  with
the positive direction of the x-axis. P(x,y) is any point on the line.
Let AP = r. AN = x – x1 = r cos  , PN = y – y1 = r sin 
y  x1 y  y1
   r is the equation of the straight line
cos  sin 

We can observe the following facts


(i) Any point on the line will be form ( x1  r cos , y1  r sin ) , where | r | gives the distance of
the point P from the fixed point (x1,y1).
(ii) For any given r there are two points on the line they are obtained by taking + and – signs. If we
want to be precise in this we have to observe the increasing and decreasing directions of r.
(iii) If we put x = x1 + r cos  , y = y1 + r sin  in the equation of a second degree curve f (x,y) = 0

A quadratic equation in r is formed. The values of r represents the distances AP


and AQ in the figure. where P and Q are points where the line
x  x1 y  y1
  r meet the second degree curve f(x,y) = 0.
cos  sin 

To convert an equation of a line into perpendicular form (or normal form) we first shift constant term to the
other side we make it positive and divide by ( coeff . of x) 2  ( coeff . of y) 2
General form
The equation Ax + By + C = 0 where A, B, C are constants represents simplest locus in coordinate
geometry which is the straight line.
property : The straight line has the property that if two points A ( x1 , y1 ) and B (x 2 , y 2 ) lie on it, all the
points on the segment joining A and B also lie on it.
proof : Since, A and B lie on the locus Ax + By + C = 0 we have Ax1 + By1 + C = 0, Ax2 + By2 + C = 0.
FG x  x IJ
 y  y1
H  1 K
2 1
Any point P on AB may be taken as , . As  varies we get all the points on AB.
 1
Now for this point P
FG x  x IJ  BFG y  y IJ  C
H  1 K H  1 K
2 1 2 1
Ax  By  C  A

 ( Ax 2  By 2  C )  ( Ax1  By 1  C ) x0  0
= =  0 for all .
 1  1
 P lies on Ax + By + C = 0 for all . Thus the equation Ax + By + C = 0 which is called the general
equation of first degree represents a straight line.
MOMENTUM : Betiahata Chowk, GKP. PH. 0551- 2332808, 2205569 Page # 12
REDUCTION OF THE GENERAL EQUATION TO DIFFERENT STANDARD FORMS
A C
 Slope-Intercept form : Given equation is Ax + By + C = 0  y = x
B B
A C
which is of the form y = mx + c where m =  , c   ( B  0)
B B
A FG
coefficient of x IJ
H
hence slope of the line Ax + By + C = 0 is  B , i. e. coefficient of y
K
and y-intercept of the line is – C/B.
 Intercept form : Given equation is Ax + By = – C
x y x y
   1 which is of the form   1. where x-intercept,
C C a b
A B
C C
a and y-intercept, b  
A B
 Normal form : Given equation is Ax + By + C = 0 or, Ax + By = – C.
A B C
Case 1 : When – C > 0, then normal form is x y
A 2  B2 A 2  B2 A 2  B2
A B C
where cos   , sin   , p
A 2  B2 A 2  B2 A  B2
2

Case 2 : When – C > 0,equation can be writen as –Ax – By = C


A B C
hence normal form is x y
A 2  B2 A 2  B2 A 2  B2
A B C
where cos  , sin   , p
A 2  B2 A 2  B2 A  B2
2

SPECIAL CASES : 1 The equation of x-axis is y = 0.


2. The equation of y-axis is x = 0.
3. The equation of a straight line parallel to x-axist at a distance
b from it is y = b.
4. The equation of a straight line parallel to y-axis at a distance a
from it is x = a.
5. Lines passing through origin : Let the slope of a line be m and suppose it passes
through the origin.

y0 y
Let P (x,y) be any point on the line then slope of the line = 
x0 x
But slope is also equal to m.
y
  m, hence it' sequation is y = mx
x

MOMENTUM : Betiahata Chowk, GKP. PH. 0551- 2332808, 2205569 Page # 13


TAP THE CONCEPTS - III

Example 24 :- Find the equation to the straight line cutting off an intercept 3 from the negative direction of the axis
of y, and inclined at 1200 to the axis of x
solution: y = x tan 1200 + (–3)
i.e. y = x  3  3,

Example 25 :- Find the equation to the straight line passing through the point (3,–4) and cutting off intercepts,
equal but of opposite signs, from the two axes.
x y
solution : Let the equation of line is,   1, or x–y=a
a a
since the straight line is passing through the point (3,–4)
 3  ( 4)  a , a  7. Therefore required equation is x – y = 7.
Example 26 :- Find the equation to the straight line
(i) Which passes through the point (–5,4) and is such that the portion of it between the axes is
divided by the point in the ratio of 1 : 2.
(ii) Which passes through the points (3,3) and (7,6) and find the length of the its portion cut off by
the axes.
(iii) Which passes through the point (–3, 8) and cuts off + ive intercepts on the axes whose sum is 7.
(iv) On which the perpendicular from the origin makes an angle of 300 with x-axis and which forms
a triangle of area 50/ 3 with the axes.
x y
Solution (i) : Let the required straight line is   1.
a b

2 . a  1 .0 2 .0  1 . b 15
 5 and  4  a= – and b = 12.
2  1 2  1 2
Therefore the required straight line is
x y
 1 i.e.5y – 8x = 60.
15 12
2
y 2  y1 6  3 3
Solution (ii) : Slope  x  x  7  3  4
2 1

3 x y
 y3 ( x  3) or 3x  4 y  3  0   1
4 1 3 4
 a  1, b  3 4
 Length of the portion intercepted between the axes = a 2  b 2  5 / 4 .
x y
Solution (iii) : Let the equation of straight line is  1
a b
3 8
given,   1 and a  b  7  a  3,  7 since intercepts are positive
a b

MOMENTUM : Betiahata Chowk, GKP. PH. 0551- 2332808, 2205569 Page # 14


 a  3 hence, b  4
x 7
Hence its equation is   1 or 4 x  3y  12
3 4

3 1 x y
Solution (iv): = 30 . Line is x cos 30 + y sin 30 = P or x.
0 0 0  y.  P   1
2p 2p
2 2
3
2p 1
a , b  2 p , Area of triangle 
3 2ab


1
2
d i
2 p / 3 . 2 p  50 / 3  p 2  25  p   5.

Hence the lines are x 3  y  10.


Example 27 :- Convert the line 3x + 4y + 25 = 0 in the normal form and state the quadrant of the line.
Solution :– We have –3x – 4y = + 25
3 4
On dividing by ( 3) 2  ( 4 ) 2  5 we get  x  y  5
5 5
3 4
 cos    ,sin     Line lies in the third quadrant.
5 5
Example 28 :- Write the line x + y = 3 in the parametric form with respect to a fixed point (2,1) on the line.
Solution :– Since slope = – 1 the equation of line is y – 1 = – 1 (x – 2) (from point slope form)
x  2 y 1 x  2 y 1 x2 y 1
or      r
1 1 1 1 cos 450
 sin 450
2 2
Example 29 :- Find the co-ordinates of two points on the line x + y = 3 which are situated at a distance 8 from
the point (2,1) on the line.
Solution :– The equation of the line can be written as
x 2 y1
0
  r
cos 45  s in 4 5 0
On putting r = 2 2 we get x = 4, y = –1

Similarly Putting r = 2 2 x=0, we get x =0, y =3  Required points are (4, –1) and (0, 3)
Example 30 :- If the straight line through the point P (3,4) makes an angle  / 6 with the x-axis and meets the line
12x + 5y + 10 = 0 at Q, find the length of PQ.
x3 y4
Solution :– Equation of line through p (3, 4) making an angle /6 with x-axis is cos 300  sin 300  r

d i b g
Let a point Q on it is r 3 / 2  3, r / 2  4 where PQ = r
if Q lies on then, 12x + 5y + 10 = 0
LMF r 3 I  3OP  5FG r  4IJ  10  0
 12
MNGH 2 JK PQ H 2 K d i
 12 3  5 r  132  0

MOMENTUM : Betiahata Chowk, GKP. PH. 0551- 2332808, 2205569 Page # 15


132 132
 r
12 b3g  5 
12 b3g  5
The above result could be written as

ax1  by1  c 12.3  5.4  10 132


r or r 
a cos  b sin  3 1 12 3  5
12.  5.
2 2
Example 31 :- The line joining two points A (2,0), B(3,1) is rotated about A in anti clockwise direction through an
angle of 150. Find the equation of the line in the new position. If B goes to C in the new position,
what will be the co-ordinates of C ?
Solution :– A is (2, 0), B is (3, 1)  AB  2 AC  2

Y C
B
2 5 0 (3, 1)
1
2
45 0

A (2, 0) X
Slope of AB = 1 (1–0)/(3–2)=1=tan , =45
= 450
Now AB is rolated through 150 in anti–clockwise direction and hence it makes an angle of
600 with x-axis. Since it passes through (2, 0) its equation in new position is
x2 y0
0
 r
cos 60 sin 600
Hence the coordinates or C are

2  2 cos 600 , . 0  2 sin 600 r  j2 or d4  2 i / 2 , 6/2


Example 32 :- The point P (1,1) is translated parallel to 2x = y in the first quadrant through a unit distance.
FG 1 2 IJ
Show that the coordinates of the new position of P are 1 
H 5
,1 
5 K
Solution :– Any line through p (1, 1) parallel to 2x=y whose tan 2 is
x 1 y 1
 r
cos sin 

x 1 y 1
hence for co-ordinates of P, r  1 ,   1
1/ 5 2 / 5
(x, y) is as given
Example 33 :- Find the direction on which a straight line must be drawn through the point (1,2) so that its point of
intersection with the line x + y = 4 may be at a distance 6 / 3
Solution :– Let the line through (1, 2) make an angle with x-axis so that the equation is
x 1 y  2
  r where r is the distance of any point on the line from the point (1, 2).
cos sin 
MOMENTUM : Betiahata Chowk, GKP. PH. 0551- 2332808, 2205569 Page # 16
Any point on it is (r cos + 1, r sin + 2),
If it lies on x + y = 4, then
[(r cos +1) + (r sin + 2) = 4]
1
r (cos + sin ) = 1. But r  6 given
3


1
3
b g
6 cos  sin   1 or cos  sin   3 / 6

1 1 3 3
 cos  sin   
2 2 b6g. b2g 2
0 0 0 0
or cos (– 450) = cos 300    45   30    75 or 15
Example 34 :- A line through A (–5,–4) meets the lines x +3y + 2 = 0, 2x + y + 4 = 0 and x –y –5 = 0 at the
points B, C and D respectively. If (15/AB)2 + (10/AC)2 = (6/AD)2, find the equation of the line.
x 5 y 4
Solution :– Any line through A (–5, –4) is  r
cos sin 
1( 5)  3( 4)  2 15 15
AB    or  cos  3 sin 
cos  3 sin  cos   3 sin  AB
10
Similarly  2 cos  sin 
AC
6
and  cos  sin 
AD
Hence from the given relation,
(cos + 3sin )2 + (2 cos + sin )2 = (cos – sin )2
or 4 cos2  + 12 sin  cos  + 9 sin2  = 0 or (2 cos  sin )2 = 0  tan = –2/3
2
Hence the equation of the line through A is y  4   ( x  5) or 2x + 3y + 22 = 0
3

Example 35 :- If A ( x1 , y1 ), B( x 2 , y 2 ) and C( x 3 , y 3 ) are the median through A is given by

x y 1 x y 1
x1 y1 1  x1 y1 10
x2 y2 1 x3 y3 1

FG x  x 3 y2  y3 IJ
H K
2
Solution :– If D is the mid-point of BC, its coordinates are ,
2 2
Therefore, eqution of the median AD is

x y 1 x y 1
x1 y1 1 0  x1 y1 1  0 [applying R 3  2 R 3 ]
x 2  x3 y2  y 3 x2  x3 y2  y 3 2
1
2 2

MOMENTUM : Betiahata Chowk, GKP. PH. 0551- 2332808, 2205569 Page # 17


x y 1 x y 1
x1 y1 1  x1 y1 10

x2 y2 1 x3 y3 1
Example 36 :- One diagonal of a square is the portion of the line x/a + y/b = 1 intercepted between the axes.
Show that the extremities of the other diagonal are
FG a  b , a  b IJ and FG a  b , b  a IJ
H 2 2K H 2 2K
Solution :– Let AB the line whose equation is x/a + y/b = 1. Y C

b a a FG IJ
Hence y 
2 b
x
2 H
........(1) K B(0,b)
E
FG b a
b gIJK
H
X
Let coordinates of C be x1 ,  x1  a 2 O A(a,0)
2 2

b g AB2 LM R F I U OP  a  b 2

MN ST GH JK VW PQ
 2 x12  a x1  a  b
2 D
Since BC  2 2
2 b 2 2

LMF a I 2a F a  b I ca  b h OP
2 2 2 2 2 2

MNGH1  b JK x  b x GH 2 b JK  4b PQ  a  b  (2x  a)  b
2 2 2 2 2
 2 1 1 2 1

x  1
a b. F a  b b  a I F a  b a  bI
 Extremities C & D are GH 2 , 2 JK and GH 2 , 2 JK
2
Example 37 :- A variable line is drawn through O to cut two fixed straight line L1 and L 2 in R and S. A point P is
m n m n
chosen on the variable line such that.   .
OP OR OS
Show that locus of P is a straight line passing through
the point of intersection of L1 and L 2 .

Solution :– Let equations of L1 and L 2 are y = c and ax + by = 1 respectively,,


Let L1 = y = C, L2 = ax + by = 1
x y
Let equation of the variable line is  r
cos  sin 
Let OP = r, OR = r1 and OS = r2 so that coordinates of P, R and S are respectively
( r cos , r sin ), ( r1 cos ,sin ) and ( r2 cos , r2 sin ).
Since R, S lies on L1, L2
 r1 sin = c and ar2 cos + br2 sin =1
c 1
or r1  and r2  (1)
sin  a cos   b sin 
mn m n
given  
OP OR OS

MOMENTUM : Betiahata Chowk, GKP. PH. 0551- 2332808, 2205569 Page # 18


mn m n m  n m sin 
     + n( a cos   b sin ) [ from(1)]
r r1 r2 r c
 ( m  n) c  m r sin   c n a r cos   cn b r sin 
Therefore locus of P ( r cos , r sin ) is cn (ax + by – 1) + m(y – c) = 0
which is a straight line passing through the intersection of y  x  0 and ax  by  1
Example 38:– Find equation of the line passing through the point (2,3) and making intercept of length 2 units
between the lines y + 2x = 3 and y + 2x = 5.
x2 y3
Solution :– Equation of any line through P(2,3) is  r
cos  sin 
Since this line makes an intercept of 2 between the given lines,
if (2  r cos ,3  r sin  ) y  2 x  3 ,
then ( 2  ( r  2) cos ,3  ( r  2) sin ) lies on y  2 x  5.
 3  r sin   2( 2  r cos )  3 and 3  ( r  2) sin   2(2  ( r  2) cos )  5
Subtacting, we get 2 sin   4 cos   2
 sin   2 cos   1  4 cos2   (1  sin ) 2
 (1  sin )[ 4  4 sin   1  sin ]  0  sin   1 or sin   3 / 5
Now if sin   3 / 5,cos   4 / 5 and the equation of the required line is
5( x  2) 5( y  3)
  3x  4 y  18
4 3
and sin   1,cos  0 therefor equation of line will be x–2=0
Example 39:– Find equation of the sides of the triangle having (3,–1) as a vertex, x – 4y + 10 = 0 and
6x + 10y –59 = 0 being the equations of an angle bisector and a median respectively drawn from
different vertices.
Solution :– Let the vertices of the triangle be A (3,–1), B ( x1 , y1 ) and C( x 2 , y 2 )
 x 2  4 y 2  10  0 (1) A(3,- 1)

FG x  3IJ  10FG y  1IJ  59  0


An
gle
bise
H 2 K H 2 K
2 2 c to r
 6
ia n
Med
 3x 2  5y 2  55  0 (2)
6x +
1 0y-
59=
0
x -4
GH
D x 2  3 , y2  1
2 2
JK
y+1
0 =
Solving (1) and (2) we get x 2  10, y 2  5 0
B(x1,y1) m C(x2,y2)
51
Equation of side AC is, y 1 ( x  3) or 6x – 7y = 25
10  3
Let the slope of BC be m, then since. BC and AC are equally inclined to the angle bisector x – 4y + 10 = 0.
1 6 1
m  1  4 m 17 2
4  7 4   m .
hence, 1 6 1 4  m 34 9
1 m 1 .
4 7 4
2
so the equation of BC is y  5   ( x  10) or 2x + 9y = 65.
9
MOMENTUM : Betiahata Chowk, GKP. PH. 0551- 2332808, 2205569 Page # 19
Now, 2 x1  9 y1  65
6x1  10y1  59.
Hence equation of AB is
81
y 1  ( x  3)  18x  13y  41.
7 / 2  3
ANGLE BETWEEN TWO LINES
The acute angle  between two lines with slope m1 and m2 is given by
m  m1
tan  2
1  m1m2
If lines are parallel   0  tan   0  m1  m 2
If lines are perpendicular then   900  tan   
 1  m1 m 2  0 or m1m 2  1

PROPERTIES OF STRAIGHT LINES

a c c
1. For line ax + by + c = 0, Slope =  , x - intercept = y-intercept = .
b a b
2. If the equation of straight lines L1, L2 are a 1x  b1 y  c1  0 and a 2 x  b 2 y  c 2  0
a1 a
then slope of lines L1 , L 2 are m1  , m 2  2 respectively
b1 b2

1 a 2 b1  a 1b 2
hence the acute angle between the lines,   tan
a 1a 2  b1b 2
a 1 b1
therefore lines are parallel if a 1b 2  a 2 b1  0  
a 2 b2
a b
and lines are perpendicular if a 1a 2  b1 b 2  0  1   2
b1 a2
a b c
The lines are identical if 1  1  1
a 2 b2 c2
3. A line parallel to ax + by + c = 0 is ax + by + k = 0 (where k is an arbitrary constant)
A line parallel to ax + by + c = 0 is bx – ay + k = 0 (where k is an arbitrary constant)
4. Length of perpendicular of a point from a straight line :
Let the equation of a line be Ax + By + C = 0 and P ( x1 , y1 ) be a point we have to find the perpendicular
distance of P ( x1 , y1 ) from the line Ax + By + C = 0.
Draw a line parallel to Ax + By + C = 0 from p ( x1 , y1 ) and draw OM perpendicular to the line from origin
and produce it to Q on this parallel line.
Now the normal form of the given line is
A B C C
x y  OM 
2 2 2 2 2 2
A B A B A B A 2  B2
The equation of the parallel line is Ax + By = Ax1 + By1
MOMENTUM : Betiahata Chowk, GKP. PH. 0551- 2332808, 2205569 Page # 20
A B Ax1  By1
OR x y
A 2  B2 A 2  B2 A 2  B2
Ax1  By1
 OQ 
A 2  B2

Ax1  By1  C
 Required perpendicular distance = QM = OQ – OM =
A 2  B2
Since this be negative therefore we can say the perpendicular distance of P ( x1 , y1 ) from the line

Ax1  By1  C
Ax + By + C = 0 is .
A 2  B2
5. Foot of perpendicular from a point to a line :
If the Foot of perpendicular from ( x1 , y1 ) to the line ax + by + c = 0 is ( ,  ) then
x  y   ( ax1by1  c )
 
a b a2  b2
6. Image of point in a line :
If the image of point ( x1 , y1 ) in the line ax + by + c = 0 is ( , ) then
  x1   y1 2( ax1  by1  c)
 
a b a2  b2
7. Distance between two parallel lines :
| c  d|
The distance between two parallel lines ax + by + c = 0 , ax + by + d = 0 is given by,
a 2  b2
8. Concurrent lines :
A set of lines is said to be concurrent if they have a single common point. Two unparallel lines are essentially
concurrent.
If three lines a 1x  b1 y  c1  0, a 2 x  b 2 y  c 2  0, a 3 x  b 3 y  c 3  0 are concurrent then point of inter-
section of any two (say last two) must lie on the remaining line.
This will easily lead to
a1 b1 c1
D  a2 b2 c2  0
a3 b3 c3
which is supposed to be a necessary condition for three lines to be concurrent. When the lines are parallel
then also D = 0.
The three distinct lienes L1  0, L 2  0, L 3  0 are concurrent are parallel if there exist
 ,  ,  (Not all zero) suct that L1  L 2  L 3  0

MOMENTUM : Betiahata Chowk, GKP. PH. 0551- 2332808, 2205569 Page # 21


TAP THE CONCEPTS - IV

Example 40:- Let PS be the median of the triangle with vertices P (2,2), Q (6, –1) and R (7,3). Find the equation
of the line passing through (1, – 1) and parallel to PS
1 2 2
Solution Slope of PS is 
(13 / 2)  2 9
2
Hence the line through (1, – 1) parallel to PS is, y + 1 = – ( x  1) or 2x + 9y + 7 = 0.
9
Example 41:- The vertices of a triangle OBC are O (0,0), B(– 3, –1), C(–1,–3). Find the equation of the line parallel to
BC and intersecting the sides OB and OC and whose perpendicular distance from the point (0,0) is 1/2.
3  ( 1)
Solution B (– 3, – 1) and C is (– 1, – 3). Slope of BC =  1.
1  ( 3)
Hence any line parallel to BC is, y = – x + c or x + y – c = 0.
1
given, (  c) / ( 1  1)   c  2 /2
2
Required equations is x  y  2 / 2  0  x  y  2 / 2  0 or 2 x  2 y  2  0.
Example 42:- Prove that the equation to the straight line passing through the point (a cos3 , a sin 3 ) and
perpendicular to the line x sec   y cos ec  a is x cos   y sin   a cos 2.
Solution : Equation of required line is, x cosec – y sec = c
 x cos –a cos4 – y sin + a sin4 =0
cosec (x–a cos3) – sec (y–a sin 3) = O
 cos4   sin 4   (cos2   sin 2 )(cos2   sin 2 )  (cos 2). 1  cos 2.
Example 43:- The line 2x + 3y = 12 meets the x-axis at A and y-axis at B. The line through (5,5) perpendicular
to AB meets the axes and the line AB at C, D, E respectively. If O is the origin of coordinates. find
the area of figure OCEB.
Solution : Equation of line perpendicular to AB is
(5,5)
3 (x–5) – 2 (y–5) = 0  3x – 2y =5 B(0, 4)
 C  (5/3, 0) and E  (3, 2)
E (3, 2)
Now, area of (OCEB) = area (OCE) + area (OBE)
O A
1 5 1 C(5/3,0)
D
. .2  .4.3  23
=
2 3 2
Example 44:- Find the coordinates of the foot of the perpendicular drawn from the point (2,3) to the line y = 3x+ 4.
Solution The given line is l  3 x  y  4  0
The foot of perpendicular to line l is given by
x2 y3 3.2  3  4 1 37
  2 2
x  ,y
3 1 3  ( 1) 10 10
Example 45:- If the equations (q – r) x + (r – p) y + (p – q) = 0 and (q 3  r 3 ) x  ( r 3  p 3 ) y  ( p 3  q 3 )  0
represent the same line, then prove that either p = q or q = r or r = p or p + q + r = 0.
q 3  r 3 r 3  p3 p 3  q 3
Solution    , say (For identical lines)
qr rp pq

MOMENTUM : Betiahata Chowk, GKP. PH. 0551- 2332808, 2205569 Page # 22


 m 2 2
r
q 3  r 3  ( q  r )  or (q  r ) (q  r  qr   )  0  either q = r or q 2  r 2  qr   .......(1)
Similarly, either r = p or r 2  p 2  rp   .......(2)
and p  q or p 2  q 2  pq   .......(3)
Hence either p = q or q = r or r = p. Again eliminating  from (2) , (3), we get
q 2  r 2  p( q  r )  0 or (q – r) (1 + r + p) = 0  q = r or p + q + r = 0
Hence either p = q or q = r or r = p or p + q + r = 0.
Example 46:- For what values of m are the lines x + y – 1 = 0, (m–1) x + (m2 –7)y – 5 = 0, (m–2) x + (2m – 5)y = 0
are concurrent ?
1 1 1
Solution : For concurrency (or all three parallel) , m  1 m2  7  5 = 0
m  2 2m  5 0
2
 m3  4 m2  5m  6  0 or (m–3) (m – m + 2) = 0  m=3
But for m = 3 the three lines become parallel.
 the given lines won’t be concurrent for any value of m.
Example 47 Prove that the lines ax + by + c = 0, bx + cy + a = 0 and cx + ay + b = 0 are concurrent
if a3 + b3 + c3 = 3abc or if a + b + c = 0.
a b c
Solution : The lines will be concurrent if b c a 0
c a b

or a ( bc  a 2 )  b( b 2  ac)  c(ab  c 2 )  0 or a 3  b 3  c 3  3abc  0


or (a  b  c)(a 2  b 2  c 2  ab  bc  ca )  0

 c
a + b + c = 0 [ a 2  b 2  c 2  ab  bc  ca 
1
2
h
 (a  b) 2 >0]
Example 48 The lines ax + 2y + 1 = 0, bx + 3y + 1 = 0 and cx + 4y + 1 = 0 are concurrent, show that
a, b, c are in A.P.
a 2 1 a  c  2b 0 0
Solution : for concurrency, b 3 1  0 orb 3 1  0 [ by R 1  R 3  2 R 2 ]
c 4 1 c 4 1
or (a + c – 2b) (3 – 4) = 0  a + c – 2b = 0  a, b, c are in A. P..
Example 49 Find the condition of concurrency of the lines y = m1x + c1,y = m2x + c2, and y = m3x + c3.
m1 1 c1
Solution : Given lines are concurrent if, m2 1 c 2  0  m1 ( c 2  c 3 )  0
m3 1 c 3
Example 50 The lines 2x + 3y – 8 = 0, 5x – 6y + 7 = 0 and px + qy – 1 = 0 are concurrent; then prove that
the line x + 2y – 1 = 0 passes through a fixed point.
Solution : The point of intersection of first two lines is (1, 2) which will lie in the third line
 p + 2q – 1 = 0. Hence the line x + 2y – 1 = 0 passes through the point (p, q)

MOMENTUM : Betiahata Chowk, GKP. PH. 0551- 2332808, 2205569 Page # 23


Example 51 A triangle has the lines y = m1x and y = m2 x as two of its sides, with m1 and m2 being roots of the
equation bx 2  2 hx  a  0. If H(a,b) is the orthocentre of the triangle, show that the equation of
the third side is (a + b) (ax + by) = ab(a + b – 2h).
b  m1 t O
 m 2  1 a  b m2
Solution :–  t
at 1  m1 m2

m x

y
1
a

=
 equation of line AB is, y  m1t  (x  t) H(a,b)

m
y=
b

2
x
m1 (a  b m2 )  a a ( a  b m2 )
or y  x 
1  m1m2 b b 1  m1 m2 A B
a m ( a  b m 2 ) a ( a  b m2 ) ab( m1  m2 )  a 2  b 2 m1m2
or y x 1 
b 1  m1 m2 b(1  m1m2 ) = b(1  m1m2 )
given m1 m 2 are roots of equation b x 2  2 hx  a  0
2 h a
 m1  m 2  , m1m 2 
b b
2ah  a 2  ab
equation of third side is,  y + a/b x = ( a  b)( by  ax)  ab(a  b  2 h )
 b(1  a / b) 
Example 52 Find the image of the line x – y –1 = 0 in the line 2x – 3y + 1 = 0.
Solution :– A point on x – y –1 = 0 is (,   1). Its image in the line 2x – 3y + 1 = 0 is given by
x   y  (  1) 2[2  3(  1)  1]
 
2 3 2 2  32
4 17  16
or x    (4   )  ........(1)
13 13
6 7  11
and y    1  (4   )  ........(2)
13 13
13x  16 17
Eliminating  from (1), (2) we get,  or 7x – 17y + 23 = 0
13y  11 7
Example 53 Find the value of k for which the lines.(k + a) x + by + c = 0, ax + (k + b) y + c = 0
ax + by + k + c = 0 are concurrent and also find the coordinates of the common point.
ka b c
Solution :– The lines are concurrent if a kb c  0 or k 2 ( k  a  b  c)  0
a b kc
If k = 0, the lines are identical.  k = – (a + b + c)
The lines become – (b + c) x + by + c = 0 etc  The common point is (1,1)
Example 54 Find the image of the curve x 2  y 2  1in the line x + y = 1
Solution :– Any point on the curve is, x = cos , y  sin . Its image is given by
x  cos  y  sin  2(cos   sin   1)
   x   sin  , y  1  cos
1 1 2
Eliminating , we get ( x  1) 2  ( y  1) 2  1

MOMENTUM : Betiahata Chowk, GKP. PH. 0551- 2332808, 2205569 Page # 24


2c 2 .
Example 55 Show that the lines ax  by  c  0 enclose a rhombus of area
| ab|
Solution :– One pair of opposite sides are ax + by + c = 0, ax + by – c = 0.
2c
Distance between them, h1  .
a 2  b2
The other pair is ax – by +c = 0, ax – by – c = 0
2c
The distance between them, h 2   h  h 2  Given lines enclose a rhombus.
a  b2
2 1

If  is the acute angle between two adjacent sides, then,

a a

tan   b b  2ab |2ab| h1h 2 2c 2
a2 b2  a 2  sin    Area = sin   | ab|
1 2 a  b2
2

b
9. Equation Of Line Equally Inclined To Two Given Lines

If there are two given lines whose slopes are m1 and m2 m2 m1

  m

m1  m m  m2
then the slope of a line making same angle with the given lines is given by 1  mm  1  mm
1 2

If we solve the above equation then we will get two values of m therefore there are two such lines which are
equally inclined to two given lines.
Special casses :
A line equally inclined to the both X and Y axes : A line with slope m is equally inclined to the X and
Y axes if m = tan ( 450 )  1

Two lines passing through a point and equally inclined to both co-ordinate axes or X and Y are equally
inclined to two given lines :
If the equation of two lines are a 1x  b1 y  c1  0, a 2 x  b 2 y  c 2

a1 a 2 a1 b1
In this case m1   m 2    [ tan(    )= –tan  ] or a   b
b1 b 2 2 2

Hence 2x + 3y + 7 = 0 and 2x – 3y + 9 = 0 are equally inclined to the axes of co-ordinates.


MOMENTUM : Betiahata Chowk, GKP. PH. 0551- 2332808, 2205569 Page # 25
10. Equation of line making an angle  with a given line
m

 M
. 

If a line makes an angle  with a given line whose slope is m1. then tan   b g m M
1  mM
m M m M M  tan  M  tan 
 tan   or  tan   ,  m or m 
1  mM 1  mM 1  M tan  1  M tan 
There will be two straight lines making some angle  with a given lines. Whose slopes are
M  tan  M  tan 
m1  and m2  .
1  M tan  1  M tan 
Hence equation of straight lines through a given point P ( x1 , y1 ) and inclined at an angle  to a given
m  tan  m  tan 
line y = mx + c are y  y1  ( x  x1 ) and y  y1  ( x  x1 )
1  m tan  1  m tan 

11. Angle bisector


If two lines are unparallel then they intersect. There will always be two lines bisecting the angle between the
given unparallel lines. These lines are called angular bisector of the given unparallel lines.

In the adjoining figure L1  a1x  b1y  c1  0, L2  a2 x  b2 y  c2  0 are two unparallel lines intersecting at S.
PSQ and RST are the lines bisecting the angle between the lines. If PSQ is bisecting  then RST is
bisecting
  . If P is any point (x,y) on PSQ (Or, R is any point (x,y) on RST) then from plane geometry..
Perpendicular PM = Perpendicular PM’
ax  by  c a ' x  b' y  c'
 
a 2  b2 a '2  b'2
ax  by  c a ' x  b' y  c'
Thus for one set such points 
a 2  b2 a '2  b'2
ax  by  c a ' x  b' y  c'
and for the other set 
a2  b2 a '2  b '2
ax  by  c a ' x  b' y  c'
The combined equation of two bisectors is sometimes written as 
2 2
a b a ' 2  b'2

MOMENTUM : Betiahata Chowk, GKP. PH. 0551- 2332808, 2205569 Page # 26


Angle bisector containing origin :
If the equation two lines are a 1x  b1y  c1  0 and a 2 x  b 2 y  c 2  0 are such that c1c 2  0
a 1 x  b 1 y  c1 a 2 x  b2 y  c2
Equation of angle bisector of the lines in which origin lies is given by 
2 2
a b
1 1 a 22  b 22
Acute and Obtuse Angle bisector :
If the equation two lines are a 1x  b1y  c1  0 and a 2 x  b 2 y  c 2  0 are such that c1c 2  0 then (i)
origin is contained in the acute angle if a 1a 2  b1b 2  0 and origin is contaned in obtuse angle if
a 1a 2  b1b 2  0
Hence the equation of angular bisector bisecting acute angle between the lines i.e. acute angle bisector
a 1 x  b 1 y  c1 a 2 x  b2 y  c2
is given by  if a 1a 2  b1 b 2  0 and ( c1c 2  0)
a 12  b12 a 22  b 22

a 1 x  b 1 y  c1 a 2 x  b2 y  c2
and it is  if a 1a 2  b1 b 2  0 and ( c1c 2  0)
a 12  b12 a 22  b 22
Similarly the equation of angular bisctor bisecting obtuse angle between the lines i.e. obtuse angle bisector
is given by,
a 1 x  b 1 y  c1 a 2 x  b2 y  c2
 if a 1a 2  b1b 2  0 and c1c 2  0
a 12  b12 a 22  b 22

a 1 x  b 1 y  c1 a 2 x  b2 y  c2
and it is  if a 1a 2  b1b 2  0 and c1c 2  0
a 12  b12 a 22  b 22

TAP THE CONCEPTS - V

Example 56:- Find the equation to the straight line which passes through the point (3, –2) and inclined at 3x  y  1.
Solution Required line are
F d 3i  tan 60 I 0

y2 G
GH 1  3 tan 60 JJK bx  3g 1  d 3i tan 60
0
0

F d 3i  tan 60 I 0

y2 G
and GH 1  3 tan 60 JJK bx  3g y  2  0 or 3 x  y  2  3
0
3

Example 57 If lines y = 3x + 1 and 2y = x + 3 are equally inclined to the line y = mx + 4, then evaluate m.
Solution Slope of the given lines are 3 and 1/2. Since these lines are qually inclined to line y = mx + 4, therefore
1
m
m3
 2 or (m – 3) (m + 2) = (1 + 3m) (1 –2m)
1  m . 3 1  m(1 / 2)

or m  [ 2  200 ] / 14  (1  5 2 ) / 7.
Example 58 A vertex of an equilateral triangle is (2,3) and the equation of the opposite side is x + y = 2.
Find the equations of the other sides of the triangle.
MOMENTUM : Betiahata Chowk, GKP. PH. 0551- 2332808, 2205569 Page # 27
Solution Here m  1, tan   tan 600  3

FG
m  tan 600 IJ bx  2g
Hence, equation of sides are y  3 
H
1  m tan 600 K  (y –3) = ( 2  3 ) (x – 2)

F m  tan 60 IJ bx  2g
and y  3  G
0

H 1  m tan 60 K0
 (y –3) = ( 2  3 ) ( x  2)

Example 59 The straight lines 3x + 4y = 5 and 4x – 3y = 15 interesect at the point A. On these lines, the
points B and C are chosen so that AB = AC. Find possible equation of the line BC passing
through the point (1,2).

3x+4y=5
Solution
Slop, m of the line BC is given by :
m4 3 3 4  m 1 C

1

b g
4 m 1  3 4 m
 m  or  7
7
450

3 m1 =4/3 900 450 4x–3y=15


Hence, the equation of line BC is, A B Sl
op
m2 = –3/4 e=
m
1
y2 
7
b g b g
x  1 or y  2  7 x  1  7 x  y  9  0 or x  7 y  13  0

Example 60 Show that the point (3, –5) lies between the parallel lines 2x+3y=7 and 2x+3y+12=0 and find the
equation of lines through (3, –5) cutting the above lines at an angle of 450.

Solution : Since (2x3+3(–5) –7) < 0 and 2(3)+3(–5)+12 > 0


 (3, –5) lies opposite sides of two given parallel lines.
 (3, –5) lies between the lines.
Equation of required lines are given by,

e 2 j  tan 45
3
b
0

x  3g and y  5 
2  tan 45
3 bx  3g
0

y5
F 2 I
1  G J  tan 45
F 2 I
1  G J tan 45
H 3K H 3K
0 0

 x–5y–28=0 or 5x+y–10=0
Example 61 A ray of linght is sent along the line x – 2y + 5 = 0; upon reaching the line 3x – 2y + 7 = 0, the ray
is reflected from it. Find the equation of the line containing the reflected ray.
Solution : Since normal (m = – 2/3) is equally inclined to incident ray (m1 = 1/2) and reflected ray m2.
( 2 / 3)  m1 ( 2 / 3)  m2 7 3m2  2 1
 or  on putting m1 
1  ( 2 / 3) m1 1  ( 2 / 3) m2 4 2 m2  3 2
or (14 – 12) m 2  8  21 or m 2  29 / 2
Hence equation of line containing reflected ray is given by,
y – 2 = (29/2) (x + 1)
or 29x – 2y + 33 = 0 .
Example 62 Which of the following points lie in the acute angle and which lie in the obtuse angle between the
lines 2x – y + 1 = 0, x + 3y – 2 = 0 ? A(0,0), B(–1,2), C(12,6)

MOMENTUM : Betiahata Chowk, GKP. PH. 0551- 2332808, 2205569 Page # 28


a 1a 2  b1b 2 2  3 1
Solution :   0  A is in the obtuse angle.
c1c 2 2 2
2x+3y–7=0
1 1 450 (3, –5)
Taking B,   0 B is in the obtuse angle.
( 3)(3) 9 450 m=–2/3
450 2x+3y+12=0
1
Taking C,  0  C is in the acute angle.
19.28
Example 63 Find the acute angle bisector between the lines 2x – y + 1 = 0, x – 2y – 2 = 0.

Solution : a 1a 2  b 1 b 2  (  4 )  0
2x  y  1 ( x  2 y  2)
 Acute angle bisector is 
5 5
Hence, obtuse angle bisector is
or, 3x – 3y –1 = 0
2x  y  1  x  2y  2
 or x + y + 3 = 0
5 5
Example 64 Find the obtuse angle bisector between the lines 3x + y + 1 = 0, x + 3y + 1 = 0.
Solution : a 1a 2  b1b 2  6  0
Obtuse angle bisector is
3x + y + 1 = x + 3y + 1 or x – y = 0
Hence, acute angle bisector is
3x + y + 1 = – (x + 3y + 1) or 2x + 2y + 1 = 0
Example 65 The vertices of a triangle are A(– 1, – 7), B(5,1) and C(1,4). Find the equation of the bisector of
angle ABC.
Solution : BC = 5, BA = 10
F 1 1I
A(-1,-7)

D is G , J .
Let D divide AC in the ratio 2 : 1. 
H 3 3K 10
D

y 1 1
The bisector is the line joining B and D.  or x  7 y  2  0 B C(1,4)
x5 7 (5,1) 5

Example 66 A vertex of an equilateral triangle is (2,3) and the equation of the opposite side is x + y = 2. Find
the equations of the other sides.
Solution : In equation of other sides are given by,
F 1  tan 60 I bx  2g 0
F 1  tan 60 I bx  2g x  y  2, m  1
0
y3 GH 1  b1g tan 60 JK 0
and y  3  GH 1  b1g tan 60 JK
0

 y  3  d2  3 i

MOMENTUM : Betiahata Chowk, GKP. PH. 0551- 2332808, 2205569 Page # 29


CENTRES IN A TRIANGLE
There are six centres in a triangle
(i) Centroid
(ii) Incentre
(iii) Excentre
(iv) Orthocentre
(v) Circumcentre
(vi) Nine point centre A (x1,y1)

(i) Centroid : It is the point of concurrency of three medians F 2 E


(line segment joining a vertex to mid point of its opposite side). 1 2 median
FG
x  x 2  x 3 y1  y 2  y 3 IJ 2
G 1
G 1
H3
,
3
(center of gravity)
K B D C
(X3,y )
(X2,y2 ) GH x 2 x , y 2 y JK
2 3 2 3

A median divide traingle into two triangles of equal area.


Hence, area ( AGF)  area ( BGF)  area ( AGE )  area ( CGE ) = area ( BGD)  area ( CGD)
therefore, area ( AGB)  area ( BGC)  area ( AGC)

(ii) Incentere : It is the point of concurancy of internal angle A (x1,y1)


bisector .It is the point inside the triangle which is
at equal distance from all the sides. E b Internal
F b+c

I
A 1 a ( c) 1 a a
Since, D  c  b x c  b  c C (x3,y3)
1 B (x2,y2) D a
FG ax  bx  cx ay1  by 2  cy 3 IJ GH bxb  ccx by 2  cy 3
JK BD C
H abc K
1 2 3
 I , 2 3
,  
abc bc DC b
a, b, c are length of BC, AC,AB

(iii) Excentre : It is the point of concurrancy of two external


and one internal angle bisector. There are three excentres
of a triangle. These are the points which are at equal
distances from all the three sides of a triangle
A
FG ax  bx  cx , ay  by  cy IJ
H a  b  c a  b  c K
1 2 3 1 2 3
I1 
B
F ax  bx  cx , ay  by  cy IJ
C

I G
H a  b  c abc K
1 2 3 1 2 3
2

F ax  bx  cx , ay  by  cy IJ
G
I1excentre
H abc abc K
1 2 3 1 2 3
& I3 Escribed Circle

MOMENTUM : Betiahata Chowk, GKP. PH. 0551- 2332808, 2205569 Page # 30


(iv) Orthocentre : A
It is the point of concurrancy of altitudes
(a perpendicular from a vertex to its opposite side)
In a right angle orthocentre is at the right angle H
altitude

B C
(v) Circumcentre :
It is the point concurrancy of perpendicular A
bisectors of three sides of a triangle.
It is the point at equal distances from
all the three vertices.
In a right angle  circumcentre is at the mid of
hypoteneous O
B C

A
(vi) Nine point centre : L
In a triangle ABC, three feet of altitudes from vertices to M G

opposite side K, L, M, three mid points of sides P,Q, R Q


R H
and three mid points of line segments joining vertices to
orthocentre E, F, G , all nine points lie on a cirlce called E F
nine point circle and its centrte is called nine point centre.
C
B K P

Example 67 Find the orthocentre of the triangle formed by the points (0,0), (4,0) and (3,4)
y

Solution : H(3,a) is orthocentre.


 Slope of BH. Slope of AC = – 1 Fig. 4 C (3,4)

4 3
= – .  1   
3 4 H

FG 3IJ A B

H 4K
x
 orthocentre is 3, (0, 0 ) D
(3,0)
(4, 0 )

Example 68 Find the orthocentre of the triangle formed by the lines x + y = 1, 2x + 3y = 6, 4x – y + 4 = 0

Solution : The altitude through the point of intersection of x + y – 1 = 0, 2x + 3y – 6 = 0 is


(2c+3y–6) +  (x+y–1) = 0 or (  +2) x+ (  +3) y–6–  =0
It is perpendicular to 4x – y + 4 = 0  4(2 +  ) – (3 +  ) = 0
5
  =  . The altitude is x + 4y – 13 = 0 ...........(1)
3
Similarly, the altitude through the point of intersection of

MOMENTUM : Betiahata Chowk, GKP. PH. 0551- 2332808, 2205569 Page # 31


x + y – 1 = 0, 4x – y + 4 = 0 is 3x – 2y + 5 = 0 ............(2)
FG 3 , 22 IJ
Solving (1) and (2), the orthocentre is H7 7 K
Example 69. The sides of a triangle are x = 2, y + 1 = 0 and x + 2y = 4. Find its circumcentre is

Y
Solution :

(2,1)
x+
2y Fig. 6
=4
x
0 y= –1
(6,-1)
x=2

The triangle is right angled and the midpoint of the hypotenuse is (4,0).

Example70 Two vertices of a triangle are at (3, –1) and (2, – 3). The orthocentre of the triangle is the origin.
Find the third vertex .

Solution : OA  C A (x,y)

FG y IJ FG 2 IJ  1
H x K H 1K or x + 2y = 0 ........(1) O

OB  CA
C (2,-3)
FG  1IJ FG y  3IJ  1 B (3,-1)
 H 3K H x  2 K or 3x – y = 9 .........(2)

FG 18 , 9 IJ
Solving (1), (2) we get A A 
H7 7K
Example 71 Find the orthocentre of the triangle formed by the lines 4x – 7y + 10 = 0, x + y = 5 and 7x + 4y = 15

Solution : The orthocentre is the point of intersection of the perpendicular sides


4x – 7y + 10 = 0 and 7x + 4y = 15. Solving we get (1,2).

Example 72 Find the nine point centre of a triangle whose vertices are (0, 3), (–6, 5), (6, 5)
(0,3)
Solution :
 nine point is the circum centre of triangle
whose vertices are (–3, 4), (0, 5), (3, 4).
 nine point centre is origin
(–3,4) (3,4)

(0,5)
(–6,5) (6,5)
MOMENTUM : Betiahata Chowk, GKP. PH. 0551- 2332808, 2205569 Page # 32
Example 73 The coordinates of the feet of the perpendiculoars from the vertices of a triangle on the opposite
sides are (20,25) (8, 16) and (8, 9). Find the coordinates of the vertices of the triangle and also,
the number of such triangles.
Solution : Since the orthocentre O of the triangle ABC is the incentre of the pedal triangle DEF. Let (h, k) be
the coordinates of O. A

Now ED  b20  8g  b25  16g


2 2
 15
(8,9)F
O
E(
8,1
6 )
FD = 20 and EF = 7
7 x20  20x8  15x8 7 x25  20x16  15x9 B D (20,25) C
Now, h  10 and k   15
7  20  15 7  20  15
Hence, O  (10, 15)
equation of AC is

y  16  
10  8
15  16
b g
x8

 y – 2x = 10 ...............(1)
Similarly equation of AB is, 3y + x – 35 = 0 ...............(2)
and equation of BC is, y + x – 45 = 0 ...............(3)
Solving (1) and (2) we get A (5, 10)
From (2) and (3) we get B (50, –5)
and from (3) and (1) we get C (15, 30)
Now we have four vertices, O, A, B and C, taking any three of them we get a triangle satisfying the given
conditions. Hence there are four such triangles.

FAMILY OF LINES

If there is a variable among the co-offcients of equation of a line, then the the equation represents an infinite
collection lines or family of lines.
example (i) ax + y = 0 represent all the lines passing through origin except y-axis (x=0).
(ii) y–3 = a (x–1) represent all the lines through (1,3) except the line x = 1.

(iii) The equation x  y  1 represent all lines whose x–intercepts are eequal to 5.
5 a
(iv) The equation x + y – 2 + a (3x – y – 2) = 0 represents all the lines through the point
of intersection of the lines x +y –2 = 0, 3x – y – 2 = 0 except the line 3x – y – 2 = 0
since for any value of a, the above family equation is satisfied by the point of
intersection i.e (1,1) of the two lines x + y – 2 = 0, 3x – y –2 = 0.

Hence t he equat ion of lines passing through the int ersectio n of lines
L1  a1x + b1y + c1 = 0, L 2  a2x + b2y + c2 = 0 must be of the form a1x + b1y + c1 + 
(a2x + b2y + c2) = 0, i.e. L1   L 2  0 where a1, b1,c1, a2, b2, c2, are fixed and  is a
parameter.

MOMENTUM : Betiahata Chowk, GKP. PH. 0551- 2332808, 2205569 Page # 33


TAP THE CONCEPTS - VII
Example 74:- Find the equation of a line which passes through the intersection of the lines x + y –3 = 0
and 2x + 4y – 10 = 0
(i) Which also passes through (0,2) (ii) which is parallel to 3x – 4y = 72
(iii) which has x-intercepts as –5 (iv) which makes an angle 450 with the line 3x + 4y = 7

Solution : The equation of lines passing through the intrsection of the lines x + y – 3 = 0, 2x + 4y –10 = 0 must
be of the form x + y –3 + k (2x + 4y –10)=0
1
(i) If it goes through (0,2) we have –1 + k (–2) = 0  k  
2
1
 required line is x  y  3  (2 x  4 y  10)  0 or y  2.
2
1  2k
(ii) The slope of the line given by x + y – 3 + k (2x + 4y –10) = 0 is 
1  4k
1  2k 3 7
Since this is parallel to the line 3x – 4y = 72 we have   k
1  4k 4 20

 Equation of required line is x  y  3 


7
20
b
2 x  4 y  10 g
(iii) The x-intercept of the line given by (obtained by putting y = 0)
3  10k
x  y  3  k ( 2 x  4 y  10)  0, is ,
1  2k
3  10k 2
Hence  5  k 
1  2k 5

b g 25 b2x  4 y  10g  0 .
required line is x  y  3 

3
(iv) Slope of the line 3x + 4y = 7 is
4

1  2k 3
 
1  4k 4
We must have tan 450 = 3(1  2 k )
1
4(1  4 k )

4k  1 4k  1 4k  1 4 3
 1   1,  1  k   ,k  
22 k  7 22 k  7 22 k  7 9 13
Thus the equations of required lines are
4 3
x+y–3– ( 2 x  4 y  10)  0 or x + y – 3 – (2 x  4 y  10)  0
9 13

MOMENTUM : Betiahata Chowk, GKP. PH. 0551- 2332808, 2205569 Page # 34


Example 75:- Show that the diagonal of a parallelogram formed by the lines ax + by + c = 0, a’x + b’y + c = 0
ax + by + c’ = 0, a’x + b’y + c’ = 0, will be at right angles if a2 + b2 = a’2 + b’2.

Solution : Equation of line AC is


ax  by  c'  ( a ' x  b' y  c')  0 ( i)
or ax  by  c   (a ' x  b' y  c)  0 (ii)
Hence equation (i) & (ii) are identical
a  a ' b  b' c  c'
 
a  a ' b  b' c  c'
On equating the first two ratios, we get
 ( ab' a ' b)   ( ab' a ' b)     (since ab’ – a’b  0
c '  c '
For    the first ratios = 1,   1    1  c  c '
c  c
Thus equation of diagonal AC is
a  a'
(ax + by + c) – (a’x + b’y + c)= 0  slope of AC = –
b  b'
Similarly  equation of the BD is ax + by + c’ + (a’x + b’y + c) = 0
a  a'
 slope of BD = – , given AC, BD
b  b'
a 2  a '2
 m1m2  1  2 2
 1  a 2  b 2  a ' 2  b ' 2 .
b  b'
Example 76:- If a and b are variable show that the lines (a + b) x + (2a–b) y = 7b pass through a fixed point.
Solution : Writing the above equation as
a(x + 2y) + b (x – y – 7) = 0
14 7
On solving x + 2y = 0, x – y –7 = 0 We get x = – ,y  
3 3
FG 14 7 IJ
Thus for all values of a and b, the lines given by pass through a fixed piont  H , .
3 3 K
FG 14 7 IJ
We can also say that the lines are concurrent at  H , .
3 3 K
Example.77 Find the image of the line L1  a1 x  b1y  c1  0 in the line L  ax  by  c  0.

Solution : Let the desired line be L 2  L  L1  0


L2=0
or L 2  ( a   a 1 ) x  ( b   b 1 ) y  c   c1  0
Let L1  0, L 2  0 be inclined at angle  with L = 0. L=0

0(a  a 1 ) a a a1 
 
( b  b1 ) b b b1 L1=0
tan   
a ( a  a 1 ) aa
1 1 1
b ( b  b1 ) bb1
MOMENTUM : Betiahata Chowk, GKP. PH. 0551- 2332808, 2205569 Page # 35
(ab1  a1 b)  (ab1  a 1 b)
i.e. 2 2

a  b   (aa 1  bb1 ) aa 1  bb1
(a 2  b 2 )
or   Equation of line L is  2(aa1  bb1 ) (ax  by  c )= (a 2  b 2 ) (a1x  b1y  c1 )
2 (aa 1  bb1 ) 2

Example 78 The sides BC, CA, AB of triangle ABC are respectively x + 2y = 1, 3x + y + 5 = 0, x – y + 2 = 0.


Find the equation of altitude through B.
Solution : The required line is
x + 2y – 1 +  (x – y + 2) = 0 ......(1)
It is perpendicular to 3x + y + 5 = 0
5
3(1 +  ) + 2 –  = 0   = –
2
5
Now (1)  x + 2y – 1 – (x – y + 2) = 0 or x – 3y + 4 = 0
2
Example 79 Prove that all lines represented by the equation ( 2 cos   3 sin ) x  ( 3 cos   5 sin )
y – (5 cos   2 sin )  0 pass through a fixed point for all values of  . Find the coordinates of
this and its reflection in the line x + y = 2.

Solution : The given equation can be written as


(2x + 3y – 5) cos  + (3x – 5y + 2) sin  = 0
or (2x + 3y – 5) + tan  (3x – 5y + 2) = 0
This passes through the point of intersection of the lines 2x + 3y – 5 = 0 and 3x – 5y + 2 = 0
for all values of  .
Now, co-ordinatery point of ref
F I
x 1 y 1
1

1
2
1 1 2
2
GH JK
 x  2  1, y  2  1

Example 80 A triangle is formed by the straight lines ax + by + c = 0,


lx + my + n = 0 and px + qy + r = 0. Show that the straight line
ax  by  c lx  my  n
 passes through the orthocenre.
ap  bq lp  mq

Solution Consider the side along the line px + qy + r = 0 ........(1)


The altitude from the opposite vertex is
ax + by + c +  (lx + my + n) = 0 ........(2)
(1) and (2) are perpendicular
 p(a +  l) + q (b +  m) = 0
(ap  bq )
or =–
lp  mq
ax  by  c lx  my  n
The altitudde (2) becomes  which passes through the orthocentre.
ap  bq lp  mq

MOMENTUM : Betiahata Chowk, GKP. PH. 0551- 2332808, 2205569 Page # 36


SHIFTING OF ORIGIN
Very often we come across a situation when change
of origin is required.
Let OX, OY be the original axes and now the
origin O be taken to a point (h,k) with reference to OX
and OY such that axes remain parallel to original axes.
Let P be a point whose co-ordinates with reference to
O be (x,y) and with reference to O’ be (X,Y)
then x=X + h,y = Y + k.
Thus if the origin is taken to a point (h,k)
and new axes remain parallel to the old axes we replace
x by X + h and y by Y + k.

ROTATION OF AXES

Let the axes OX and OY be rotated through an angle 


in the counter clockwise direction such that the new
axes become Ox’ and Oy’. Then if P has co-ordinates
(x,y) with reference to original axes and has
co-ordinates (x’,y’) with reference to new axes,
we can show that
x  x' cos   y' sin  and y  x' sin   y' cos 
x = OM = ON – MN = ON – RL = x' cos   y' sin ,
y = PM = MR + RP = LN + RP = x' sin   y' cos 
Also x'  x cos   y sin , y'   x sin   y cos 

TAP THE CONCEPTS - VIII

Example 81 Remove x and y term from the equation


2 x 2  4 xy  5y 2  4 x  22 y  7  0 by shifting origin.

Solution On putting x = X + h, y = Y + k the given equation becomes


2(X + h)2 + 4 (x +h) (Y + k) + 5(Y + k)2 – 4(X + h) – 22 (Y + k) + 7 = 0
 Coeff. of X = 4h + 4k – 4, Coeff. of Y = 4h + 10k – 22
Choosing h, k such that these coefficients become zero)
 4h + 4k – 4= 0, 4h + 10k – 22 = 0
from which we get h = – 2, k = 3
Thus x = X – 2, y = Y + 3
 Origin must be shifted to (–2,3), the new equation becomes
2(X–2)2 + 4(X–2) (Y+3) + 5(Y+3)2 – 4(X–2)–22(Y+3)+7 = 0
OR 2X2 + 4XY + 5Y2 – 22 = 0

MOMENTUM : Betiahata Chowk, GKP. PH. 0551- 2332808, 2205569 Page # 37


Example 82 Transform to parallel axes through the point (1, –2) the equations
(i) y2 –4x + 4y + 8 = 0 and (ii) 2x2 + y2 – 4x + 4y = 0
Solution : (i) Put x = x' + 1, y = y' – 2 to the given equation, we get
(y' – 2)2 – 4 (x' + 1) + 4 (y' – 2) + 8 = 0  y'2 = 4x'.
(ii) put x = x' +1, y = y' – 2 to the given equation, we get
2(x' + 1)2 + (y' – 2)2 – 4(x' + 1) +4(y' – 2) = 0  2x'2 + y'2 = 6.
Example 83 What does he equation (x –a)2 + (y – b)2 = c2 become when origin is shifted to
(i) the point (a – c,b); (ii) the point (a,b – c)?
Keeping the axes parallel.
Solution : Given equation is (x–a)2 + (y–b)2 = c2
(i) put x = x' + a – c, y = y' + b, to the given equation, we get
(x' + a – c – a)2 + (y' + b – b)2 = c2
 x'2 + y'2 – 2cx' = 0.
(ii) put x = x' + a and y = y' + b – c to the given equation, we get
(x' + a – a)2 + (y' + b – c – b)2 = c2
 x'2 + y'2 – 2cy' = 0.
Example 84 By transforming to parallel axes through a properly chosen point (h,k) prove that the equation
12x2 – 10xy + 2y2 + 11x – 5y + 2 = 0 can be reduced to one containing only terms of the second
degree.
Solution : put x = x' + h, y = y' + k to the given equation we get
12 (x' + h)2 –10(x' + h) (y' + k) + 2 (y' + k)2 + 11 (x' + h) –5 (y' + k') + 2 = 0
 12x'2 – 10x'y' + 2y'2 + (24h–10k+11) x' + (–10h+4k–5)y' + (12h2–10hk+2k2+11h–5k+2) = 0
3 5
If 24h – 10k + 11 = 0 and –10h + 4k – 5 = 0, solving we get, h  ,k .
2 2
Example 85 Transform to axes inclined at 450 to the original axes the equations
(i) x2 – y2 = a2, (ii) 17x2 – 16xy + 17y2 = 225, and (iii) y4 + x4 + 6x2y2 = 2.

Solution : put h  3 2 , k  5 2 to the constant form in equation, we get.

e je j FGH IJK e j FGH IJK


2

e j 5 5
2
12 3 2  10 3 2 5 2  2  11 3 2  5 2  a
2 2

Therefore, for h  3 2 , k  5 2 the given equation reduces to 12x’2 – 10x’y’ + 2y’2 = 0

x' y' x' y'


(i) put x=x’ cos 450–y’ sin 450 = and y=x’ sin450 + y’ cos 450= to the given
2 2
equation, we get

FG x'y' IJ  FG x'y' IJ
2 2
a 2
H 2K H 2K  a 2   x' y' 
2
x' y' x' y'
(ii) put x  and y  to the given equation, we get
2 2

F x'y' IJ
17 G
2
F x' y' IJ FG x' y' IJ  17 FG x' y' IJ
 16 G
2

H 2K H 2 KH 2 K H 2 K  225

MOMENTUM : Betiahata Chowk, GKP. PH. 0551- 2332808, 2205569 Page # 38


Example 86 If the axes be turned through an angle tan–12, what does the equation 4xy – 3x2 = a2 become?

Solution : 
17
2
c h
2 x' 2  2 y ' 2 
2
c
16 2 2
x'  y'  225 h
 9 x'2 25y'2  225
2 1
  tan 1 2  sin   and cos 
5 5
x'2 y' 2 x' y'
hence, put x  x' cos  y' sin   and y  x'sin   y' cos 
5 5
to the given equation, we get

FG x'2y' IJ FG 2 x' y'IJ  3 FG x'2 y' IJ 2

4
H 5 KH 5 K H 5 K  a2  x' 2  4 y'2  a 2

LOCUS AND ITS EQUATION


The locus of a moving point is the path traced out by it under certain geometrical condition or conditions.
Thus a curve C will be the locus of a point P under given condition or conditions, if
(i) whenever given conditions are satisfied then point P lies on curve C,
(ii) whenever point P lies on curve C, given condition or conditons are satisfied.
EQUATION OF A LOCUS :
An equation is said to be the equation of the locus of a moving point if the following two conditions are
satisfied.
(1) The coordinates of every point on the locus satisfy the equation.
(2) If the coordinates of any point satisfy the equation, then that point must lie on the locus.
EXAMPLE : If x 2  y 2  a 2 is the equation of a locus, and ( x1 , y1 ),( x 2 , y 2 ),( x 3 , y 3 ) are points on the
locus, then x12  y12  a 2 , x 22  y 22  a 2 and x23  y23  a2. If 2  2  a2 then point ( , ) will lie on thelocus.
Working rule to find the locus of a point :
(1) Let the co-ordinates of the moving point P be (h, k)
(2) Using the given geometrical conditions, find the relation in h and k. This relation must contain at
least one of h and k and known quantities.
(3) Express the given relation in h and k in the simplest form and then put x in place of h and y in place
of k. The equation thus obtained will be the required equation of the locus.

TAP THE CONCEPTS - IX

Example 87 The ends of rod of length l move on two mutually perpendicular lines. The locus of the point on the
rod which divides it in the ratio 1 : 2 is
x y
Solution :- Let the equation of the rod be  1
a b
a 2  b 2  2 .....(1)
Let C (h,k) divide AB in the ratio 2 : 1. So, by section formula,

MOMENTUM : Betiahata Chowk, GKP. PH. 0551- 2332808, 2205569 Page # 39


2.0  1. a a
h   a  3h
2 1 3
2. b  1.0 2 b 3k
k  b 
21 3 2

2 k2
Putting for a and b in (1), we get 9 h  9 P
4
x2 y 2 l 2
So, the required locus is  
1 4 9

Example 88:– The altitudes of a triangle are along the lines y = m1 x, y  m 2 x, y  m 3 x.


Show that the centroid lies on the line y( 3  m1m 2  m 2 m 3  m 3 m1 ) = x ( m1  m 2  m 3  3m1 m2 m3 ).

Solution O(0,0) is the orthocentre and let the vertices be A ( x1 , m1 x1 ), B( x 2 , m 2 x 2 ), C( x 3 , m3 x 3 ).


m2 x 2  m3x 3 1
OA  BC    x 2 (1  m1m 2 )  x 3 (1  m2 m3 )
x 2  x3 m1
x1 x2 x3
Likewise x 3 (1  m 2 m3 )  x1 (1  m2 m1 )     k , say.
1  m2 m3 1  m3 m1 1  m1 m2
If ( x , y ) is the centroid G, then
x1 ( 3  m1m 2 ) y1 m1 x1 ( m1  m1 m 2 )
x  k and y    k
3 3 3 3 3
Hence G lies on the straight line
y ( 3  m1 m 2 )  x( m1  3m1 m 2 m 3 ).

Example 89 A rod of length l moves such that its ends A and B always lie on the lines 3x – y + 5 = 0 and
y + 5 = 0 respectively. Find the locus of the point P which divides AB internally in the ratio 2 : 1
Solution : A point on 3x – y + 5 = 0 is A (, 3  5) .
A point on y + 5 = 0 is B (,5)
If P (h, k) divides AB in the ratio 2 : 1.
We have, 3h    2  , 3k  3  5
3k  5 9 h  3k  5
Solving   , 
3 6

FG 9k  9h  15IJ  b3k  15g


2

H 2 K
2
l2  AB2  (   ) 2  (3  10) 2 =

1
Hence, the locus of P (h, k) is l2  (3x  3y  5) 2  (3y  15) 2
4
Example 90 Through a fixed point O, a variable straight line L is drawn to cut n straight lines at points
P1 , P2 , P3 ,........ Pn . If P is a point on the line L such that OP is the harmonic mean of
OP1 , OP2 ,..... OPn , find the locus of P..

MOMENTUM : Betiahata Chowk, GKP. PH. 0551- 2332808, 2205569 Page # 40


Solution : Let the lines be L i  a i x  b i y  1, i = 1, 2, ....n and L be x = r cos , y  r sin .
1
L intersects Li at the point Pi, where r1  OP1 
a i cos   b i sin 
n
Since OP is the H.M of r1 , r2 ,... rn , We find,  ( a i ) cos   ( b i ) sin .
r
or, the locus of P is ax + by = n, where a = a i and b  b i .

Example 91 The vertices of a triangle lie on the lines y = x tan 1 , y  x tan  2 , y = x tan  3 .
The circumcentre is the origin. Show that the locus of the orthocentre is
x (sin 1  sin  2  sin  3 ) = y ( cos 1  cos  2  cos  3 )
Solution : If R is the circumradius, then the vertices are ( R cos  i , R sin  i ) i = 1, 2, 3.
The centroid is x  R (cos 1  cos  2  cos  3 ) / 3 , y  R (sin 1  sin  2  sin  3 ) / 3
Hence G lies on the line
x sin  i  y cos  i . Which will also be the can of orthocentre.
Since orthocentre and centroid remain in the same line.
Example 92 The equations of the altitudes AD, BE, CF of a triangle ABC are x + y = 0, x – 4y = 0, 2x – y = 0
respectively. The coordinates of A are (t, – t). Find the coordinates of B and C.
Find the locus of the centroid G as t varies.

Solution : O (0,0) is the orthocentre.Let B = (4, ) and C(, 2)


CF  AB ,
1 t t 2t t FG IJ
  
2 4  t
    and B is  , 
6 3 6 H K
2  t t t FG IJ
Further BE  AC ,  4 
t
   and C is , t
2 2 H K
Let ( h, k) be the centroid G..
2t t 5 t t
3h  t    t and 3k   t  t  
3 2 6 6 6
 h  5k  0 , The locus of G is x + 5y = 0.

Example 93 A rectangle PQRS has its side PQ parallel to the line y = mx and vertices P,Q, and S on the lines
y = a, x = b and x = – b respectively. Find the locus of the vertex R.
Solution : R (x, y)
P (–x, a) S R(x, y)
(-b, a + m(b + x))
Q (b, a + m (b + x))


y a m b x b g  1
x b m P Q (b, a + m(b + x))
(-x, a)
FG 1 IJ FG
1 IJ
or y  m  H m
x m
K H
m
ba
K
which is the locus of R.
MOMENTUM : Betiahata Chowk, GKP. PH. 0551- 2332808, 2205569 Page # 41
Example 94 The ends A and B of a line segment of constant length c slide upon x and y axes respectively.
If the rectangle OAPB is completed where O is the origin, then show that the locus of the foot
of perpendicular drawn from P on AB is the curve x 2 3  y 2 3
 c2 3
x y
Solution : Let the line AB be   1 . Then c2 = a2 + b2. Then P is (a, b). If R (h, k) is the foot of
a b
perpendicular from P on AB , we have

 
b
h  a k  b  1 1 1
 2
g
a 2 b 2
 h
a3
, k 
b3
1 1 1 1 a  b2 a 2  b2 a 2  b2
2
 2
a b a b

 c
h2 3  k 2 3  a 2  b2 h
13
 c2 3

Hence, locus of R (h, k) is x2 3  y2 3  c2 3

Example 95 Find the locus of centroid of triangle whose vertices are (a cos t, a sin t), (b sin t, – b cos t) and
(1, 0), where t is a variable parameter.

Solution : 3h = a cos t + b sin t + 1


3k = a sin t – b cos t
(3h – 1)2 + 9k2 = (a cos t + b sin t)2 + (a sin t – b cos t)2 = a2 + b2.
Hence, locus of (h, k) is (3x – 1)2 + 9y2 = a2 + b2.

Example 96 P is the point (–1, 2), a variable line through P cuts the co-ordinate axes in A and B respectively.
Q is a point on AB such that PA, PQ and PB are in H.P. Show that the locus of Q is the line y = 2x.
x 1 y  2
Solution :– Any line through P (–1, 2) is   r , Let PA = r1, PQ = r2, PB = r3
cos sin 

b g b
Hence A  r1 cos  1, r1 sin   2 and B  r2 cos  1, r2 sin   2 g
1  sin  1
As points A and B lie on X and Y axes  r1 sin +2 = 0 and r2 cos –1 = 0  r  2 and r  cos
1 3

Let the point Q be (h, k)  h = r2 cos – 1 and k = r2 sin + 2
2 1 1 sin 
It is given that r1, r2, r3 are in H.P.      cos ,
r2 r1 r3 2

or
2

b
1 k2

h 1 g b g or 2 = (–k/2) + 1+ h + 1
r2 2 r2 r2
Hence locus of Q (h, k) is y = 2x.

MOMENTUM : Betiahata Chowk, GKP. PH. 0551- 2332808, 2205569 Page # 42


PAIR OF STRAIGHT LINES
EQUATION OF PAIR OF STRAIGHT LINES
Case - I : L1 : a1x + b1y + c1 = 0
L2 : a2x + b2y + c2 = 0

Combined Equation of L1 & L2 is


( a 1 x  b 1 y  c1 ) ( a 2 x  b 2 y  c 2 )  0
 a 1a 2 x 2  (a 1b 2  a 2 b1 ) xy  b1b 2 y 2  (a 1c 2  a 2 c1 ) x  ( b1c 2  b 2 c1 ) y  c1c 2  0
 ax2  2 hxy  by 2  2gx  2 fy  c  0
(Assuming a 1a 2  a ,(a 1b 2  a 2 b1 )  2 h, b1b 2  b,
(a 1c 2  a 2 c1 )  2 g, ( b1c2  b 2 c1 )  2 f , c1c 2  c )
Hence, combined equation of pair of lines not passing through the origin gives non Homogeneous equation
of second degree.
Comparing above equations we can conclude that a non Homogeneous equation of second degree repre-
sents pair of straight lines if
a h g
h b f 0
  abc  2 fgh  af 2  bg2  ch 2  0 or
g f c
Case - II : L1 : a1x + b1y = 0
L2 : a2x + b2y = 0
Combined Equation of L1 & L2 is
( a 1 x  b1 y ) ( a 2 x  b 2 y )  0
a 1a 2 x 2  (a 1b 2  a 2 b1 ) xy  b1b 2 y 2  0
ax2  2 hxy  by 2  0
Hence, combined equation of two homogeneous equation of 1 degree gives homogeneous equation of second
degree.
Therefore, homogeneous equation of second degree represents combined equation of pair of lines passing
through origin.

Note : If we multiply equations of two lines then resulting algebraic equation will be satisfied by points on both
the lines. The equation so obtained is called equation of pair of lines.

Ex. -1 If we multiply the equation of line L1  x  y  0 and L2  x  2 y  0 then we get


( x  y) ( x  2 y)  0  y 2  3xy  2 x 2  0 ............(1)
which is called equation of pair of lines L1 & L2 because equation (1) will satisfy all the points on L1 and L2.

Ex. -2 The equation xy=0 is equivalent to x = 0 or y = 0. Hence it represent a co-ordinate axes.

MOMENTUM : Betiahata Chowk, GKP. PH. 0551- 2332808, 2205569 Page # 43


Ex. -3 The equation xy  4 x  5y  20  0 is equivalent to (x – 5) (y – 4) = 0 hence equation represent pair of
straight lines x = 5 and y = 4.

Ex. -4 x 2  5x  6  0 is pair of lines x  2 and x  3.

HOMOGENEOUS EQUATION OF SECOND DEGREE


An equation which is such that in each term the sum of the indices of x and y is same, is called a
homogeneous equation and sum of indices is called degree of homogeneous equation.
eg. 3x 3  4 x 2 y  5xy 2  9 y 3 = 0 is a homogeneous equation of third degree.
A homogeneous equation of second degree can be written as
ax2  2 hxy  by 2  0 .........(1)
On multiplying by a it may be written in the form
( a 2 x 2  2ahxy  h 2 y 2 )  ( h 2  ab) y 2  0

i.e. {(ax  hy)  y h 2  ab ) } {(ax  hy)  y h 2  ab }  0.


Therefore equation (1) represents the two straight lines whose equations are
ax  hy  y h 2  ab  0 ...........(2)

and ax  hy  y h 2  ab  0 ...........(3)
each of which passes through the origin.
Equation (1) is satisfied by all the points which satisfy (2) and also by all the points which satisfy (3).
These two straight lines are
(i) real and different if h2 > ab
(ii) real and coincident if h2 = ab and
(iii) imaginary if h2 < ab.
Note: In case when h2 < ab, the straight lines, though themselves imaginary, intersect in a real point.The origin lies
on the locus given by (1), since the equation (1) is always satisfied by the values x = 0 and y = 0.
The equation ax2 + 2hxy + by2 = 0 can be written as
by 2  2 hxy  ax 2  0 .............(1)
Dividing by x2 , we get
FG y IJ 2
FG y IJ  a  0
b
H xK  2h
H xK
y
put  m, we get bm2  2 hm  a  0 .................(2)
x
if equation (1) represent two lines with slopes m1, m2 then roots of equation (2) will be m1, m2.
2h a
Hence m1  m2   , m1m2  .................(3)
b b
Note: These relations are very useful in dealing with problems on homogeneous equation of second degree since
without actually factorizing the equation we can derive several important results on lines given by
ax 2  2 hxy  hy 2  0

MOMENTUM : Betiahata Chowk, GKP. PH. 0551- 2332808, 2205569 Page # 44


ANGLES BETWEEN TWO LINES
If  be angle between lines represented by ax 2  2 hxy  by 2  0 then
tan 1  tan  2
  1   2  tan   tan(1   2 )  tan  
1  tan 1 tan  2

FG 2h IJ  4 a
2

m1  m2 ( m1  m2 ) 2  4 m1m2 HbK b
tan    
tan  
F aI
1 G J
1  m1m2 1  m1m2
H bK
y  m1 x

4 h 2 4a 2 h 2  ab
  y  m2 x
b2 b  b 2 h 2  ab
tan  
ba ab FG 
IJ
(a  b)
b b H K 1
2

2 h 2  ab 2 h 2  ab
tan      tan 1
ab ab
if h2 > ab then lines are real and distinct
if h2 = ab then lines are real & coincedent
if h2 < ab then lies are imaginary
if a+b=0 then lines are perpendicular
Note (i) If lines are perpendicular   90o  tan     a  b  0 , i.e. coeff. of x2 + coeff. of y2 = 0
(ii) If lines are parallel then   0o .  tan   0  h 2  a b since lines pass through the origin,
they are therefore coincident in this case.

Pair of lines perpendicular to given pair of lines


The equation of pair of lines perpendicular to pair of lines ax2 + by2 + 2hxy = 0

1
FG y  1 xIJ FG y  1 xIJ  0 y
m2
x
y  m1 x

H m KH m K
1
y x
1 2 m1

( m1y  x ) ( m2 y  x )  0 y  m2 x

m1m2 y 2  ( m1  m2 ) xy  x 2  0
ay 2  2 hxy  bx 2  0

MOMENTUM : Betiahata Chowk, GKP. PH. 0551- 2332808, 2205569 Page # 45


Product of the length of the perpendiculars from (x1y1) to ax2 + 2hxy + by2 = 0 :
m1x1  y1 ( x1y1 )
p1  y  m1x
2
m1  1 p1
p2
y  m2 x
m2 x1  y1
p2 
m2 2  1

m1x1  y1 m2 x1  y1
p1 p 2  
2 2
m1  1 m2  1

a 2 2h
m1m2 x12  ( m1  m2 ) x1y1  y12 x1  x1y 2  y12
p1 p 2   b b
m12  m22  ( m1m2 ) 2  1 ( m1  m2 ) 2  2 m1m2  ( m1m2 ) 2  1

a 2 2h ax12  2 hx1y1  by12


x1  x1y1  y12
p1 p 2  b b b ax12  2 hx1y1  by12
FG IJ
2 h
2
2a a 2

4 h 2  2ab  a 2 b 2

4 h 2  ( a  b) 2
H K
b
  2 1
b b b

Angle bisector
The equation of pair of angle bisectors of given pair of lines ax2 +2hxy + by2 = 0
1       2 y  m1x
2   1   2
(1  )
tan 2  tan(1  2 )
2 tan  tan 1  tan  2 (  2 )
2
 y  m2 x
1  tan  1  tan 1.tan  2
1
2m m  m2 2 
2
 1
1 m 1  m1m2

2y 2 h 2y 2h
x  b x xy h
 b
1  y2 a FG IJ  x 2  y 2 (a  b)
 x2  y2  a  b
x 2
1
b H K x2 b

x 2  y 2 xy

ab h

MOMENTUM : Betiahata Chowk, GKP. PH. 0551- 2332808, 2205569 Page # 46


Note: if two pair of lines are given
P1 : a 1x 2  2 h1xy  b1y 2  0 P2 : a 2 x 2  2 h 2 xy  b 2 y 2  0 P1
P2 P2
P1
(I) if each pair bisects the angle between the other pair then

Angle bisecter of one coinside with the other.

P2
P1
(II) if angle between the two lines one from each pair is equal
P2
to angle between the remaining two lines then angle bisectors
of both the pair of lines are same. P1

HOMOGENISATION
If a line lx + my + n = 0 intersects a curve in two distinct points then combined equation of pair of lines
joining the origin to the point of intersection of curve is
A
ax2  2 hxy  by 2  2gx  2 fy  c  0
Equation of OA & OB is Homogeneous equation of second degree
to make the non Homogeneous equation of second degree Homogeneous
F x  myIJ  1
x  my   n  G
B

H n K
O
ax2  2 hxy  by 2  2gx(1)  2 fy(1)  C(1) 2  0

F x  my IJ  2fyFG x  my IJ  cFG x  my IJ
ax  2 hxy  by  2gxG
2

H n K H  n K H  n K
2 2
0

On simplification we get Homogeneous equation of 2nd degree.

Example-1 : Find the equation of lines joining origin and point of intersection of the line 3x + 4y = 7 with the
conic 2x2 + y2 + xy + 5x – 9 = 0.
3x  4 y
Solution: we have  1 on line.
7
FG
3x  4 y IJ FG
3x  4 y IJ 2
 The equation of required lines is 2x2 + y2 + xy + 5x
H 9
7 K H 7 K 0

 98x2 + 49y2 + 49xy + 105x2 + 140xy – 81x2 – 144y2 – 216xy = 0


 122x2 – 27xy – 95y2 = 0

Example-2 : Show that all chords of the curve 2x2 + 3y2 – 5x = 0 subtending 900 at origin pass through a
fixed point. Find co-ordinates of this point.
Solution: Let y = mx + c .......(1)
be any chord AB of 2x2 + 3y2 – 5x = 0 subtending 900 at origin.
Then equation of OA and OB (O origin) are given by
MOMENTUM : Betiahata Chowk, GKP. PH. 0551- 2332808, 2205569 Page # 47
FG y  mx IJ  0
2x2 + 3y2 – 5x H c K
5m
Since these lines are at right angle so,  2  3 0  C=–m
c
The equation of chords (1) now becomes y = mx – m  y = m(x – 1)
For any value of m this line passes through the fixed point (1, 0 ).

TAP THE CONCEPTS - I

Example-1 Find joint equation of the diagonals of the square formed by the pairs of lines xy + 4x – 3y – 12 = 0
and xy – 3x + 4y – 12 = 0
Solution. xy + 4x – 3y – 12 = (x – 3) (y + 4) = 0  x = 3, y = –4 .................(i)
and xy – 3x + 4y – 12 = (x + 4) (y – 3) = 0  x = –4, y = 3 .................(ii)
From (i) and (ii) we find that the vertices of the square are A(3, 3), B(–4, 3), C(–4, –4) and D(3, –4)
3 4
Equation of the diagonal AC is y = x and of BD is, y  3  ( x  4 )  x  y  1  0.
4  3
Hence the required equation of the diagonals is (x – y) (x + y + 1) = 0  x2  y2  x  y  0
Example-2 : Calculate the distance betweent the parallel lines given by ( x  7 y ) 2  4 2 ( x  7 y)  42  0
Solution. lines are ( x  7 y  3 2 ) ( x  7 y  7 2 )  0  x  7 y  3 2  0 and x  7 y  7 2  0

7 2  ( 3 2 ) 10 2
distance between these lines   2
2 2 5 2
1 7

Example-3 : If the pairs of lines x2 + 2xy + ay2 = 0 and ax2 + 2xy + y2 = 0 have exactly one line in common
then find the joint equation of the other two lines
Solution : Let y = mx be a line common to the given pairs of lines, then
m2 m 1
am2  2am  1  0 and m2  2 m  a  0   2 
2(1  a ) a  1 2(1  a )
a 1
 m2  1 and m    (a  1) 2  4  a  1 or  3
2
FG 3  1IJ  1
But for a = 1, the two pairs have both the lines common so a = –3 So, m = 
H 2 K
Now x2 + 2xy + ay2 = x2 + 2xy – 3y2 = (x – y) (x + 3y)
ax2 + 2xy + y2 = –3x2 + 2xy + y2 = – (x – y) (3x + y)
So the equation of the required lines is
( x  3y) ( 3x  y)  0  3x 2  10xy  3y 2  0

Example-4 : If one of the lines given by equation 2x2 + axy + 3y2 = 0 coincide with one of those given by
2x2 + bxy – 3y2 = 0 and the other lines represented by them be perpendicular, then find a & b

MOMENTUM : Betiahata Chowk, GKP. PH. 0551- 2332808, 2205569 Page # 48


2 2 a
Solution : Let x  xy  y 2  ( y  mx) ( y  mx)
3 3
2 2 b 1
x  xy  y 2  ( y  x ) ( y  mx )
3 3 m
a 2
then m  m   , mm  ................(i)
3 3
1 b m 2
and  m  ,   ................(ii)
m 3 m 3
 m2  1  m   1
2 2
If m  1, m   a  5, b  1 & If m  1, m    a  5, b  1 .
3 3

Example-5 : If the product of the perpendicular drawn from the point (1, 1) on the lines ax2 + 2hxy + by2 = 0 is 1,
h (a  b)
then calculate
ab

a .12  2 h .1.1  b .12


Solution : 1  ( a  b  2 h ) 2  ( a  b) 2  4 h 2
(a  b) 2  4 h2

 ( a  b) 2  4 h ( a  b)  ( a  b ) 2  0  4 ab  4 b (a  b)  0
h (a  b)
  1
ab
Example-6 : If two of the lines given by 3x3  3x2 y  3xy2  dy3  0 are at right angles then calculate slope
of the lines
Solution : Put y = mx in the given equation weget, dm3 – 3m2 + 3m + 3 = 0
m1 m2 m3 = –3/d. If the lines y = m1x, y = m2x are perpendicular, then m1 m2 = –1
 m3 = 3/d  d(3/d)3 –3 (3/d)2 + 3 (3/d) + 3 = 0  d = –3
3 2 2 3 2 2
The given becomes x + x y – xy – y = 0 or (x + y) (x – y ) = 0
hence the slopes of the lines 1, 1, –1.
Example-7: If the lines ax2 + 2h xy + by2 = 0 are two sides of a parallelogram and the line lx + my = 1 is one
of its diagonals, show that the equation of the other diagonals is y (b l – h m) = x (am – hl ).
Solution : Let y = m1 x and y = m2 x be the lines OA and OB given by ax2 + 2h xy + by2 = 0.
Then m1 + m2 = –2hlb and m1 m2 = a/b.
FG 1 m1 IJ
solving y = m1x and lx + my = 1, we get the point A as
H 1  mm 1
,
mm 1 K
FG 1 m2 IJ
Similarly, point B is
H 1  mm 2
,
1  mm2 K
FG 2  m(m  m ) , 2mm m  ( m  m ) IJ
H 2(  mm )(  mm ) 2(  mm )(  mm ) K
1 2 1 2 1 2
Hence D  (mid point of AB)
1 2 1 2

MOMENTUM : Betiahata Chowk, GKP. PH. 0551- 2332808, 2205569 Page # 49


Therefore equation of diagonal OD given by
FG 2mm m  ( m  m ) IJ x FG 2m(a / b)  (2h / b) IJ x  FG am  h IJ x
H 2  m( m  m ) K H 2  m(2 h / b) K H b  hm K
1 2 1 2
y 
1 2

 y ( b  hm)  x (am  h )

Example-8: The equation (x + y – 6) (xy – 3x – y + 3) represents sides of a triangle then find the coordinates
of the circumcentre of the triangle
Solution : Equation of lines are (x + y – 6) (xy – y – 3x + 3) = 0  (x + y) and y(x – 1) – 3(x – 1) = 0
Equation of the sides of the triangle are
x+y=6 .........(1)
y=3 .........(2)
x=1 .........(3)
Triangle is right angled at (1, 3) .
Since circum centre of right angled triangle is mid point of hypoteneous, Hence (2, 4)
Example-9 : If 1 and  2 be the angles which the lines (x2 + y2) (cos2  sin2  + sin2  ) = (x tan  – y sin  )2
make with the axis of x, if  =  / 6 , then prove that tan  1 + tan  2 = –8/3 cosec 2
Solution: The given equation can be written as
(x2 + y2) (cos2  sin2  + sin2  ) = x2 tan2  – 2xy tan  sin  + y2 sin2 
or (cos2  sin2  + sin2  – tan2  ) x2 + 2 (tan  sin  ) xy + cos2  sin2 y2 = 0
2 tan  sin 
sum of the slopes, (tan 1  tan 2 ) =
cos2  sin 2 
2 tan 
 tan  1 + tan  2 = 2  (3 / 4)  sin 2  (   / 6) = – (8/3) cosec 2  .

Example-10 : If two of the lines represented by x4 + x3 y + cx2 y2 – xy3 + y4 = 0 bisect the angle between the
other two, then calculate the value of c
Solution: Let the equation of one pair be ax2 + 2hxy – ay2 = 0
x 2  y2 xy
then equation of its bisectors is  .
2a h
Now x4 +x3y + cx2y2 – xy3 +y4 = (ax2 +2hxy – ay2) (hx2 – 2axy – hy2) = ah(x4 +y4) + 2(h2 – a2) (x3y – xy3) – 6ahx2y2
Comparing the co-efficients, we get ah = 1 and c = – 6ah = –6
Example-11 : If y = mx bisects the angle between the lines x2 (tan2  + cos2  ) + 2xy tan  – y2 sin2  = 0
when  =  / 3 calculate the value of m
Solution: Equation of the bisectors of the angles between the given lines is
x 2  y2 xy x 2  y2 xy x 2  y 2 xy
2 2 2
  1  tan 2   tan    when  =  / 3
tan   cos   sin  tan  1 3 3

1 m2 m
which is satisfied by y = mx if   3 m 2  4 m  3.
4 3

 4  16  12 2  7
 3 m2  4 m  3  0  m= 
2 3 3

MOMENTUM : Betiahata Chowk, GKP. PH. 0551- 2332808, 2205569 Page # 50


Example-12 : From a point A (1,1) straight lines AL and AM are drawn at right angles to the pair of straight lines
3x2 + 7xy –2y2 = 0. Find the equation of pair of straight lines AL and AM.
Solution: Equation of the lines through the origin perpendicular to the given lines 3x2 + 7xy – 2y2 = 0 is

–2x2 – 7xy + 3y2 = 0 i.e. 2x2 + 7xy – 3y2 = 0


So, the equation of the lines through (1,1) perpendicular to the given lines is
2(x – 1)2 + 7(x – 1) (y – 1) –3(y – 1)2 = 0 or 2x2 + 7xy – 3y2 – 11x – y + 6 = 0
Example-13 : Show that the orthocentre of the triangle formed by the lines ax2 + 2hxy + by2 = 0 and l x + my = 1 is
x y ab
given by   2
1 m am  2 h l m  bl 2
Solution: Let OA, OB and AB be y = m1 x, y = m2 x and lx + my = 1 respectively.
Equation of OD is mx – ly = 0
x y
or   k (say) ........(1)
1 m
FG 1 m1 IJ
Coordinates of A are
H l  mm 1
,
l  mm 1 K
m1 1 1 FG
1  m1m2 IJ
1 2 1 H
Equation of AE is y – l  mm   m x  l  mm  m2y + x = l  mm
1 K ......(2)

Orthocentre H of the triangle is point common to AD and BE.


1  m1m2
Any point on (1) is (Kl, Km) and if it lies on (2)  k (l + mm2 ) = l  mm
1

1  m1m 2 (a  b)
 k = (l  mm ) (l  mm2 ) = [ m1m2 = a/b, m1+ m2 = –2h/b]
1 am  2 h l m  bl 2
2

Hence the orthocentre of the triangle is given by x  y  ab


.
l m am  2 hl m  b l 2
2

GENERAL EQUATION OF SECOND DEGREE

An equation of the type S  ax2 + 2hxy + by2 + 2gx + 2fy + c = 0 is called general equation of second
degree. It can be proved that S = 0 admits two linear factors of the type lx + my + n and l  x + my + n  if
a hg
2 2 2
 = abc + 2fgh – af – bg – ch = 0 or h b f  0 (Condition for pair of straight lines)
g f c
Nature of lines :
(i) The lines become coincident if   0, h 2  ab and f 2  bc
(ii) If   0, h 2  ab and f 2  bc then the lines are parallel.
(iii) Again the lines will be imaginary if   0, h 2  ab  0.

MOMENTUM : Betiahata Chowk, GKP. PH. 0551- 2332808, 2205569 Page # 51


Point of Intersection :
We can find the point of intersection of two lines given by ax2 + 2hxy + by2 + 2gx + 2fy + c = 0 by
F I
solving the equation:
G
dx
 ax  hy  g  0 and
G
dy
, GH
bg  hf af  gh
 hx  by  f  0 . So it is 2
h  ab h 2  ab
JK
Angle of Intersection :
Equation ax2 + 2h xy + b y2 = 0 represents a pair of straight lines both passing through origin parallel to
the lines given by the general equation of second degree (S = 0) and hence the angle between the lines given by S = 0 is
2 h 2  ab
  ta n  1
ab
Hence if a + b = 0, the lines are perpendicular and if h2 = ab, the lines are parallel
Angle Bisector :

Equations of the bisectors of the angles between the lines represented by ax2 + 2hxy + by2 + 2gx + 2fy + c = 0
( x  x1 )2  ( y  y1 ) 2 ( x  x1 ) ( y  y1 )
are given by  where (x1, y1) is the point of intersection of the
ab h
lines represented by the given equation.

Distance between parallel straight lines :

If ax2 + 2hxy + by2 + 2gx + 2fy + c = 0 represents two parallel straight lines, then distance between
g 2  ac
them is 2 .
a (a  b)

Seperation :

A general Equation of the form ax2 + 2hxy + by2 + 2gx + 2fy + c = 0 represent two lines say y = m1x + c1 and
y = m2x + c2 to find them first factorise ax2 + 2h xy + by2 = 0 to get y = m1 x and y = m2 x then write
ax2 + 2bxy + by2 + 2gx + 2fy + c = (y – m1x – c1) (y – m2x – c2 ) by equating co-efficicients of x, y,
and constant we get c1 and c2
2nd method:- Express the second degree equation as the quadratic equation in terms of x or y then get
b  d
x or y, using formula
2a

TAP THE CONCEPTS - II

Example-1 : For what value of  does the equation 12x2 – 10xy + 2y2 + 11x – 5y +  = 0 represent two
lines. Find the point of intersection of two lines and also find the angle between them.
11 5
Solution: Here, a = 12, b = 2, h = –5, g = , f=– , c= 
2 2
2 2 2
The given equation will represent lines if abc  2 fgh  af  bg  ch  0
FG 5 IJ FG 11IJ (5)  12FG  5IJ  2FG 11IJ
2 2
 12 (2) (  )  2 
H 2K H 2 K H 2K H 2 K   ( 5) 2  0 which easily gives  = 2

MOMENTUM : Betiahata Chowk, GKP. PH. 0551- 2332808, 2205569 Page # 52


for  = 2 let us write the given equation as 2y2 + y (–5 –10x) + 12x2 + 11x + 2 = 0

5  10x  ( 5  10x) 2  8(12 x 2  11x  2)


 y=  4y = 5 +10x  (2x + 3)
4
1
 Lines are y = 3x + 2 and y = 2x +
2
FG
3 5 IJ
H
The point of intersection of these lines is  , 
2 2 K
1 1
 1
and the angle  between them is given by tan  = 2 1 3 1 or   tan 1
1  7
2 3
2nd method:- 12x2 – 10xy + 2y2 = (6x – 2y) (2x – y)
Hence 12x2 – 10xy + 2y2 + 11x – 5y +  = (6x – 2y + c1) (2x – y + c2)
comparing the co-effcients of x and y and constant we get 6c2 + 2c1 = 11, – 2c2 – c1 = – 5 , c1 c2 = 
1
solving we get c1 = 4, c2 = ,  = 2
2
1
Hence the equation the lines are y = 3x + 2 and y = 2x +
2
Example-2 : If  ia an angle between the lines given by the equation 6x2 + 5xy – 4y2 + 7x + 13y – 3 = 0, then
find the equation of the line passing through the point of intersection of these lines and making an
angle  with the positive x-axis.
Solution: writing the given equation as a quadratic in x, we get 6x2 + (5y + 7)x – (4y2 – 13y + 3) = 0
 (5y  7 )  (5y  7 ) 2  24 ( 4 y 2  13y  3)
 x=
12
(5y  7)  11( y  1) 6y  18 16y  4
 =  ,  2x – y + 3 = 0 and 3x + 4y – 1 = 0
12 12 12
 point of intersection is (–1, 1),

2 h 2  ab 2 (5 / 2) 2  6( 4) 11
Also tan  =  = 
ab 64 2
11
So the equation of the required line is y–1 = 
(x + 1)  11x – 2y + 13 = 0, 11x + 2y + 9 = 0
2
Example-3 : Calculate the value of f, h so that the equation 9x2 + 2hxy + 4y2 + 6x + 2fy – 3 = 0 represent
two parallel line.
Solution: Since the given equation represents a pair of parallel lines, therefore
9h 3
2
h = 9 4 and h 4   0.
3  3

 3h2  6h   9  2  144  0  108  36   9 2  144  0 (h   6)


   2 if h  6 and    2 if h   6

MOMENTUM : Betiahata Chowk, GKP. PH. 0551- 2332808, 2205569 Page # 53


EXERCISE-1
ELEMENTARY EXERCISE

1. Prove that the points (2a, 4a), (2a, 6a) and (2a + 3a , 5a) are the vertices of an equilateral triangle whose
side is 2a.
 1 
2. In what ratio does the point  , 6  divide the line segment joining the points (3, 5) and (– 7, 9) ?
2 
3. Find the third vertex of a triangle if two of its vertices are at (– 2, 4) and (7, – 3) and the centroid at
(3, 2).
4. Find circumcentre of triangle whose vertices are (–2, –3) (–1, 0) (7, –6).
5. A and B are the points (3, 4) and (5, – 2). Find the coordinates of a point P such that PA = PB and the area
of the triangle PAB = 10.
6. Find the area of the quadrilateral with vertices as the points given in each of the following :
(i) (0, 0) (6, 0) (4, 3) (0, 3) (ii) (0, 0) (a, 0) (a, b), (0, b)
7. Find the equation of the locus of the point whose distance from x-axis is twice that from y-axis.
8. Find the locus of the centroid of a triangle whose vertices are (a cos t, a sin t), (b sin t, –b cos t) and (1, 0)
where ‘t’ is the parameter
y 2  y3 y 3  y1 y1  y 2
9. If the points (x1, y1), (x2, y2) and (x3, y3) be collinear, show that x2x3 + x 3 x1 + x 1x 2 = 0.
10. Find the equation to the straight line cutting off an intercept – 5 from the axis of y and being equally inclined
to the axes.
11. The coordinates of the mid-points of the sides of a triangle ABC are D(2, 1), E(5, 3) and
F(3, 7). Find the lengths and equations of its sides.
12. Find the slope of the lines which make an angle of 45° with the line x – 2y = 3.
13. Find the coordinates of the othocentre of the triangle whose sides are 3x – 2y = 6, 3x + 4y + 12 = 0 and
3x – 8y + 12 = 0.
14. Write down parametric equation of line passing through the points (2, 1) and (1, 2)
15. Find the distance between the parallel lines 3x – 4y + 5 = 0 and 3x – 4y + 7 = 0.
16. If p and p' be the perpendiculars from the origin upon the straight lines whose equations are
x sec  + y cosec  = a and x cos – y sin  = a cos 2, prove that 4p2 + p'2 = a2.
17. Find the area of parallelogram whose two sides are y = x + 3 and 2x – y + 1 = 0 also remaining two sides
are passing through (0, 0).
18. Find the equation of the bisector of the angle between the lines 4x + 3y – 7 = 0 and 24x + 7y – 31 = 0
which contains the origin.
19. Prove that x(a + 2b) + y (a – 3b) = (a – b) passes through a fixed point for all a, b  R
20. Find the value of k so that the following equations may represent pairs of straight lines :
12x2 – 10xy + 2y2 + 11x – 5y + k = 0
21. The variable line x cos + y sin = 2 cuts the x and y axes at A and B respectively. Find the locus of the
vertex P of the rectangle OAPB, O being the origin.
x y x y
22. A variable line, drawn through the point of intersection of the straight lines  = 1 &  = 1,
a b b a
meets the coordinate axes in A & B. Show that the locus of the mid point of AB is the curve
2xy(a + b) = ab(x + y).

MOMENTUM : Betiahata Chowk, Gorakhpur PH. 0551-2332808, 2205569 Page # 54


23. If A(x1, y1), B(x2, y2), C(x3, y3) are the vertices of the triangle then show that :
x y 1 x y 1
(i) The median through A can be written in the form x1 y1 1  x 1 y1 1 = 0.
x2 y2 1 x3 y3 1

x y 1 x y 1
(ii) the line through A & parallel to BC can be written in the form ; x1 y1 1  x1 y1 1 = 0.
x2 y2 1 x3 y3 1

x y 1 x y 1
(iii) equation to the angle bisector through A is b x1 y1 1  c x1 y1 1 = 0.
x2 y2 1 x3 y3 1

where b = AC & c = AB.


24. If the straight lines, ax + by + p = 0 & x cos  + y sin  - p = 0 enclose an angle /4 between them, and meet
the straight line x sin  – y cos  = 0 in the same point, then find the value of a2 + b2 .
25. Drive the conditions to be imposed on  so that (0, ) should lie on or inside the triangle having sides
y + 3x + 2 = 0, 3y – 2x – 5 = 0 & 4y + x – 14 = 0.
26. Let the co-ordinates of the two points A & B be (1, 2) and (7, 5) respectively. The line AB is rotated
through 45º in anti clockwise direction about the point of trisection of AB which is nearer to B. Find the
equation of the line in new position.
27. Find the equations of the straight lines passing through the point (1, 1) and parallel to the lines represented
by the equation, x2 - 5 xy + 4 y2 + x + 2 y - 2 = 0.
28. Show that all the chords of the curve 3x² - y² - 2x + 4y = 0 which subtend a right angle at the origin are
concurrent. Does this result also hold for the curve, 3x² + 3y² - 2x + 4y = 0 ? If yes, what is the point of
concurrence & if not, give reasons.
29. Find the coordinates of the vertices of a square inscribed in the triangle with vertices
A (0, 0), B (2, 1), C (3, 0); given that two of its vertices are on the side AC.
30. Two consecutive side of a parallelogram are 4x + 5y = 0 and 7x + 2y = 0. If the equation to one diagonal
is 11x + 7y = 9. Find the equation of the other diagonal.
31. Two ends A & B of a straight line segment of common length c slide upon the fixed
rectangular axes Ox & Oy respectively. If the rectangle OAPB is completed show that the locus of the foot
of the perpendicular drawn from P to AB is,x2/3 + y2/3 = c2/3.
32. Find the equations of the sides of a triangle having (4, –1) as a vertex, if the lines x – 1 = 0 and
x – y – 1 = 0 are the equations of two internal bisectors of its angles.
x y
33. Line   1 intersects the x and y axes at M and N respectively. If the coordinates of the point P lying
6 8
inside the triangle OMN (where ‘O’ is origin) are (a, b) such that the areas of the triangle POM, PON and
PMN are equal. Find
(a) the coordinates of the point P and
(b) the radius of the circle escribed opposite to the angle N.
34. Find the co-ordinates of the orthocentre of the triangle, the equations of whose sides are x + y = 1,
2x + 3y = 6, 4x – y + 4 = 0, without find the co-ordinates of its vertices.
35. Two vertices of a triangle are (4, –3) & (–2, 5). If the orthocentre of the triangle is at (1, 2), find the
coordinates of the third vertex.
36. The point A divides the join of P (–5, 1) & Q (3, 5) in the ratio K : 1. Find the two values of K for which
the area of triangle ABC, where B is (1, 5) & C is (7, –2), is equal to 2 units in magnitude.

MOMENTUM : Betiahata Chowk, Gorakhpur PH. 0551-2332808, 2205569 Page # 55


37. Determine the ratio in which the point P(3, 5) divides the join of A(1, 3) & B(7, 9). Find the harmonic
conjugate of P w.r.t. A & B.
38. A line is such that its segment between the straight lines 5x – y – 4 = 0 and 3x + 4y – 4 = 0 is bisected at
the point (1, 5). Obtain the equation.
39. A line through the point P(2, –3) meets the lines x – 2y + 7 = 0 and x + 3y – 3 = 0 at the points A and B
respectively. If P divides AB externally in the ratio 3 : 2 then find the equation of the line AB.
40. The area of a triangle is 5. Two of its vertices are (2, 1) & (3, –2). The third vertex lies on y = x + 3. Find
the third vertex.
x y x y
41. A variable line, drawn through the point of intersection of the straight lines   1 &   1 , meets
a b b a
the coordinate axes in A & B. Show that the locus of the mid point of AB is the curve 2xy(a + b) = ab(x + y).
42. Two consecutive sides of a parallelogram are 4x + 5y = 0 & 7x + 2y = 0. If the equation to one diagonal
is 11x + 7y = 9, find the equation to the other diagonal.
43. The line 3x + 2y = 24 meets the y-axis at A & the x-axis at B. The perpendicular bisector of AB meets the
line through (0, –1) parallel to x-axis at C. Find the area of the triangle ABC.

44. If the straight line drawn through the point P( 3 ,2) & inclined at an angle with the x-axis, meets the line
6
3x  4 y  8  0 at Q. Find the length PQ.
45. Find the condition that the diagonals of the parallelogram formed by the lines
ax + by + c = 0; ax + by + c' = 0; a'x + b'y + c = 0 & a'x + b'y + c' = 0 are at right angles. Also find the
equation to the diagonals of the parallelogram.
46. A triangle has side lengths 18, 24 and 30. Find the area of the triangle whose vertices are the incentre,
circumcentre and centroid of the triangle.
47. The points (1, 3) & (5, 1) are two opposite vertices of a rectangle. The other two vertices lie on the line
y = 2x + c. Find c & the remaining vertices.
48. A straight line L is perpendicular to the line 5x – y = 1. The area of the triangle formed by the line L & the
coordinate axes is 5. Find the equation of the line.
49. Two equal sides of an isosceles triangle are given by the equations 7x – y + 3 = 0 and x + y – 3 = 0 & its
third side passes through the point (1, –10). Determine the equation of the third side.
50. The vertices of a triangle OBC are O(0, 0), B(–3, –1), C(–1, –3). Find the equation of the line parallel to
BC & intersecting the sides OB & OC, whose perpendicular distance from the point (0,0) is half.
51. Starting at the origin, a beam of light hits a mirror (in the form of a line) at the point A(4, 8) and is reflected
at the point B(8, 12). Compute the slope of the mirror.
52. Given vertices A(1, 1), B(4, –2) & C(5, 5) of a triangle, find the equation of the perpendicular dropped
from C to the interior bisector of the angle A.
53. Triangle ABC lies in the Cartesian plane and has an area of 70 sq. units. The coordinates of B and C are
(12, 19) and (23, 20) respectively and the coordinates of A are (p, q). The line containing the median to the
side BC has slope –5. Find the largest possible value of (p + q).
54. A straight line is drawn from the point (1, 0) to the curve x2 + y2 + 6x – 10y + 1 = 0, such that the intercept
made on it by the curve subtends a right angle at the origin. Find the equations of the line.
55. Determine the range of values of  {0, 2 } for which the point (cos ,sin ) lies inside the triangle
formed by the lines x + y = 2 ; x – y = 1 & 6x  2 y  10  0 .
56. The points (–6, 1), (6, 10), (9, 6) and (–3, –3) are the vertices of a rectangle. If the area of the portion of
this rectangle that lies above the x axis is a/b, find the value of (a + b), given a and b are coprime.
57. The two line pairs y2 – 4y + 3 = 0 and x2 + 4xy + 4y2 – 5x – 10y + 4 = 0 enclose a 4 sided convex polygon
find
(i) area of the polygon; (ii) length of its diagonals.

MOMENTUM : Betiahata Chowk, Gorakhpur PH. 0551-2332808, 2205569 Page # 56


58. The equations of perpendiculars of the sides AB & AC of triangle ABC are x – y – 4 = 0 and 2x – y – 5 = 0
FG 3 , 5IJ , find
respectively. If the vertex A is (–2, 3) and point of intersection of perpendiculars bisectors is
H 2 2K
the equation of medians to the sides AB & AC respectively.
59. The interior angle bisector of angle A for the triangle ABC whose coordinates of the vertices are A(–8, 5);
B(–15,–19) and C(1, –7) has the equation ax + 2y + c = 0. Find ‘a’ and ‘c’.
60. Show that all the chords of the curve 3x2 – y2 – 2x + 4y = 0 which subtend a right angle at the origin are
concurrent. Does this result also hold for the curve 3x2 + 3y2 – 2x + 4y = 0 ? If yes, what is the point of
concurrency & if not, give reasons.
61. The coordinates of the vertices of a quadrilateral are A(0, 0); B(16, 0), C(8, 8), D(0, 8). Find the equation
of the line parallel to AC that halves the area of the quadrilateral in the form of y = mx + c.
62. Find the equation of the straight lines passing through (–2, –7) & having an intercept of length 3 between
the straight lines 4x + 3y = 12, 4x + 3y = 3.
63. Without finding the vertices or angles of the triangle, show that the three straight lines au + bv = 0; au – bv
= 2ab and u + b = 0 from an isosceles triangle where u  x  y  b & v  x  y  a & a , b  0.
64. Two sides of a rhombous ABCD are parallel to the lines y = x + 2 & y = 7x + 3. If the diagonals of the
rhombous intersect at the point (1, 2) & the vertex A is on the y-axis, find the possible coordinates of A.
65. The equations of the perpendicular bisectors of the sides AB & AC of a triangle ABC are x – y + 5 = 0 &
x + 2y = 0, respectively. If the point A is (1, –2) find the equation of the line BC.
66. A triangle is formed by the lines whose equations are AB : x + y – 5 = 0, BC : x + 7y – 7 = 0 and
CA : 7x + y + 14 = 0. Find the bisector of the interior angle at B and the exterior angle at C. Determine the
nature of the interior angle at A and find the equation of the bisector.
67. A point P is such that its perpendicular distance from the line y – 2x + 1 = 0 is equal to its distance from the
origin. Find the equation of the locus of the point P. Prove that the line y = 2x meets the locus in two points
Q & R, such that the origin is the mid point of QR.
68. Find the area of the triangle formed by the straight lines whose equations are x + 2y – 5 = 0; 2x + y – 7 = 0
ans x – y + 1 = 0 without determining the coordinates of the vertices of the triangle. Also compute the
tangent of the interior angles of the triangle and hence comment upon the nature of triangle.
69. Find the equation of the two straight lines which together with those given by the equation
6x2 – xy – y2 + x + 12y – 35 = 0 will make a parallelogram whose diagonals intersect in the origin.
70. Find the equations of the sides of a triangle having (4, –1) as a vertex, if the lines x – 1 = 0 and x – y – 1 = 0
are the equations of two internal bisectors of its angles.
71. Equation of a line is given by y + 2at = t(x – at2), t being the parameter. Find the locus of the point of
intersection of the lines which are at right angles.
72. P is the point (–1, 2), a variable line through P cuts the x & y axes at A & B respectively Q is the point on
AB such that PA, PQ, PB are H.P. Show that the locus of Q is the line y = 2x.

MOMENTUM : Betiahata Chowk, Gorakhpur PH. 0551-2332808, 2205569 Page # 57


EXERCISE-2
MAIN EXERCISE

STRAIGHT LINES

1. The three points (–2, 2), (8, –2) and (– 4, –3, ) are the vertices of
(a) An isosceles triangle (b) An equilateral triangle
(c) A right angled triangle (d) none of these
2. Find a point on the line joining points (0, 4) and (2, 0) dividing the line segment externally in ratio 3 : 2
3 8 8 3
(a) (3, –4) (b) (6, – 8) (c)  5 , 5  (d)  5 , 5 
   
3. The points A(–4, –1), B (–2, –4), C (4, 0) and D(2, 3) are the vertices of
(a) parallelogram (b) rectangle (c) rhombus (d) none of these
4. Find the area of the triangle formed by the mid points of sides of the triangle whose vertices are
(2, 1), (– 2, 3), (4, – 3)
(a) 1.5 sq. units (b) 3 sq. units (c) 6 sq. units (d) 12 sq. units
5. Given the points A (0, 4) and B (0, –4), the equation of the locus of the point P (x, y) such that
| AP – BP | = 6 is :
(a) 9x2 - 7y2 + 63 = 0 (b) 9x2 - 7y2 - 63 = 0 (c) 7x2 - 9y2 + 63 = 0 (d) 7x2 - 9y2 - 63 = 0
6. Slope of line joining points (5, 3) abd (k2, k + 1) is 1/2 then k is
(a) 1 (b) 1 + 2 (c) 2 – 1 (d) – 1 – 2
7. Find the equation to the straight line which passes through the point (– 4, 3) and is such that the portion of
it between the axes is divided by the point in the ratio 5 : 3.
(a) 9x – 20y + 96 = 0 (b) 2x – y + 11 = 0 (c) 2x + y + 5 = 0 (d) 3x – 2y + 7 = 0
8. The equations of the perpendicular bisector of the sides AB and AC of a ABC are x – y + 5 = 0 and
x + 2y = 0 respectively. If the point A is (1, –2) then the equation of the line BC is :
(a) 14x + 23y = 40 (b) 14x – 23y = 40 (c) 23x + 14y = 40 (d) 23x – 14y = 40
9. The distance of the point (2, 3) from the line 2 x - 3 y + 9 = 0 measured along a line x - y + 1 = 0 is :
(a) 5 3 (b) 4 2 (c) 3 2 (d) 2 2
10. Which pair of points lie on the same side of 3x – 8y – 7 = 0
(a) (0, –1) and (0, 0) (b) (4, –3) and (0, 1)
(c) (– 3, – 4) and (1, 2) (d) (–1, –1) and (3, 7)
11. The reflection of the point (4, –13) in the line 5x + y + 6 = 0 is
(a) (–1, –14) (b) (3, 4) (c) (1, 2) (d) (–4, 13)
12. Find the equation of the bisector of the acute angle between the lines 3x – 4y + 7 = 0 and
12x + 5y – 2 = 0.
(a) 11x – 3y + 9 = 0 (b) 3x + 11y – 13 = 0
(c) 3x + 11y – 3 = 0 (d) 11x – 3y + 2 = 0
13. The lines ax + by + c = 0, where 3a + 2b + 4c = 0 are concurrent at the point :
1 3  3 1
(a)  2 , 4  (b) (1, 3) (c) (3, 1) (d)  4 , 2 
   

MOMENTUM : Betiahata Chowk, Gorakhpur PH. 0551-2332808, 2205569 Page # 58


14. If the slope of one line of the pair of lines represented by ax2 + 10xy + y2 = 0 is four times, the slope of the
other line, then a =
(a) 1 (b) 2 (c) 4 (d) 16
15. The combined equation of the bisectors of the angle between the lines represented by
(x2 + y2) 3 = 4xy is

x 2  y2 xy
(a) y2 – x2 = 0 (b) xy = 0 (c) x2 + y2 = 2xy (d) =
3 2

16. The equation of second degree x2 + 2 2 xy + 2y2 + 4x + 4 2 y + 1 = 0 represents a pair of straight lines.
The distance between them is
4
(a) 4 (b) (c) 2 (d) 2 3
3

17. The straight lines joining the origin to the points of intersection of the line 2x + y = 1 and curve
3x2 + 4xy – 4x + 1 = 0 include an angle :
   
(a) (b) (c) (d)
2 3 4 6
18. The orthocentre of the triangle ABC is 'B' and the circumcentre is 'S' (a, b). If A is the origin then the
co-ordinates of C are :
 a b
(a) (2a, 2b) (b)  ,  (c)  a 2  b 2 , 0 (d) none
 2 2
19. A triangle ABC with vertices A (- 1, 0),B (- 2, 3/4) & C (- 3, - 7/6) has its orthocentre H. Then the
orthocentre of triangle BCH will be :
(a) (–3, –2) (b) (1, 3) (c) (–1, 2) (d) none of these
20. In a triangle ABC, co-ordinates of A are (1, 2) and the equations to the medians through B and C are
x + y = 5 and x = 4 respectively. Then the co-ordinates of B and C will be
(a) (–2, 7), (4, 3) (b) (7, –2), (4, 3) (c) (2, 7), (–4, 3) (d) (2, –7), (3, –4)
21. Equation of a straight line passing through the origin and making with x - axis an angle twice the size of the
angle made by the line y = 0.2 x with the x - axis, is :
(a) y = 0.4 x (b) y = (5/12) x (c) 6y – 5x = 0 (d) none of these
22. A variable straight line passes through a fixed point (a, b) intersecting the co-ordinates axes at A& B. If 'O'
is the origin then the locus of the centroid of the triangle OAB is :
(a) bx + ay - 3xy = 0 (b) bx + ay - 2xy = 0 (c) ax + by - 3xy = 0 (d) ax + by - 2xy = 0
23. Area of the quadrilateral formed by the lines | x | + | y | = 2 is :
(a) 8 (b) 6 (c) 4 (d) none
24. The set of values of 'b' for which the origin and the point (1, 1) lie on the same side of the straight line,
a2x + a by + 1 = 0  a  R, b > 0 are :
(a) b  (2, 4) (b) b  (0, 2) (c) b  [0, 2] (d) (2, )
2
25. The point (a , a + 1) is a point in the angle between the lines 3x – y + 1 = 0 and x + 2y – 5 = 0 containing
the origin if :
(a) a  1 or a  - 3 (b) a  (–3, 0)  (1/3, 1)
(c) a  (0, 1) (d) none of these

MOMENTUM : Betiahata Chowk, Gorakhpur PH. 0551-2332808, 2205569 Page # 59


26. Drawn from the origin are two mutually perpendicular straight lines forming an isosceles triangle together
with the straight line, 2x + y = a. Then the area of the triangle is :
a2 a2 a2
(a) (b) (c) (d) none
2 3 5
27. The image of the point A (1, 2) by the line mirror y = x is the point B and the image of B by the line mirror
y = 0 is the point (, ) then :
(a)  = 1, = –2 (b)  = 0, = 0 (c)  = 2, = - 1 (d) none of these
28. The line x + 3y - 2 = 0 bisects the angle between a pair of straight lines of which one has equation
x - 7y + 5 = 0. The equation of the other line is :
(a) 3x + 3y - 1 = 0 (b) x - 3y + 2 = 0 (c) 5x + 5y - 3 = 0 (d) none
29. On the portion of the straight line, x + 2y = 4 intercepted between the axes, a square is constructed on the
side of the line away from the origin. Then the point of intersection of its diagonals has
co-ordinates :
(a) (2, 3) (b) (3, 2) (c) (3, 3) (d) none
30. A light beam emanating from the point A(3, 10) reflects from the straight line 2x + y - 6 = 0 and then passes
through the point B(4, 3). The equation of the reflected beam is :
(a) 3x - y + 1 = 0 (b) x + 3y - 13 = 0 (c) 3x + y - 15 = 0 (d) x - 3y + 5 = 0
31. The equation of the bisector of the angle between two lines 3 x - 4 y + 12 = 0 and 12 x - 5 y + 7 = 0 which
contains the points (- 1, 4) is :
(a) 21x + 27y - 121 = 0 (b) 21x - 27y + 121 = 0
 3x  4y  12 12x  5y  7
(c) 21x + 27y + 191 = 0 (d) =
5 13
32. The equation of bisectors of two lines L1 & L2 are 2 x – 16 y – 5 = 0 and 64 x + 8 y + 35 = 0. If the line
L1 passes through (–11, 4), the equation of acute angle bisector of L1 & L2 is :
(a) 2 x – 16 y – 5 = 0 (b) 64 x + 8 y + 35 = 0
(c) data insufficient (d) none of these
33. AB is a variable line sliding between the coordinate axes is such a way that a lies on x-axis and B lies on
y-axis. If P is a variable point on AB such that PA = b, PB = a and AB = a + b, then equation of locus of P is
x2 y2 x2 y2
(a) 2 + 2 =1 (b) – =1 (c) x2 + y2 = a2 + b2 (d) none of these
a b a2 b2
34. Equations of the line pair through the origin and perpendicular to the line pair xy – 3y2 + y – 2x + 10 = 0 is:
(a) xy – 3y2 = 0 (b) xy + 3x2 = 0 (c) xy + 3y2 = 0 (d) x2 – y2 = 0
35. If pairs of straight lines, x2 – 2p xy – y2 = 0 & x2 – 2q xy – y2 = 0 be such that each pair bisects the angles
between the other pair then :
(a) pq = –1/2 (b) pq = –2 (c) pq = –1 (d) p/q = –1
36. If the straight lines joining the origin and the points of intersection of the curve
5x2 + 12xy – 6y2 + 4x – 2y + 3 = 0 and x + ky – 1 = 0 are equally inclined to the x-axis then the value of
k is equal to :
(a) 1 (b) –1 (c) 2 (d) 3
37. Let triangle ABC have vertices A(–4, –3), B(6, –1) and C(2, 5). the length of the median from C to AB, is
(a) 50 (b) 53 (c) 89 (d) 104
38. The length of a line segment AB is 10 units. if the coordinates of one extermity are (2, –3) and the abscissa
of the other extremity is 10 then the sum of all possible values of the ordinate of the other extremity is
(a) 3 (b) –4 (c) 12 (d) –6

MOMENTUM : Betiahata Chowk, Gorakhpur PH. 0551-2332808, 2205569 Page # 60


39. If P(1, 2), Q(4, 6), R(5, 7) & S(a, b) are the vertices of a parallelogram PQRS, then :
(a) a = 2, b = 4 (b) a = 3, b = 4 (c) a = 2, b = 3 (d) a = 3, b = 5
40. the four points whose co-ordinates are (2, 1), (1, 4), (4, 5), (5, 2) form :
(a) a rectangle which is not a square (b) a trapezium which is not a parallelogram
(c) a square (d) a thombus which is not a square
41. If A and B are the points (–3, 4) and (2, 1), then the co-ordinates of the point C on AB produced such that
AC = 2BC are :
FG 1 5 IJ
(a) (2, 4) (b) (3, 7) (c) (7, –2) (d)  2 , 2 H K
42. The orthocentre of the triangle ABC is 'B' and the circumcentre is 'S' (a, b). If A is the origin then the
co-ordinates of C are :
FG a b IJ e j
H K
2 2
(a) (2a, 2b) (b) 2 , 2 (c) a  b , 0 (d) none
43. A particle begins at the origin and moves successively in the following manner as shown,
1 unit to the right, 1/2 unit up, 1/4 unit to the right,
1/8 unit down, 1/16 unit to the right etc.
The length of each move is half the length of the previous move and movement continues in the 'zigzag'
manner indefinitely. the co-ordinates of the point to which the 'zigzag' converges is
(a) (4/3, 2/3) (b) (4/3, 2/5) (c) (3/2, 2/3) (d) (2, 2/5)
44. Coordinates of the vertices of a triangle ABC are (12, 8), (–2, 6) and (6, 0) then the coorect statement is
(a) triangle is right but not isosceles (b) triangle is isosceles but not right
(c) triangle is obtuse
(d) the product of the abscissae of the centroid, orthocentre and circumcentre is 160
45. The area of the quadrilateral ABCD with vertices A(–2, 0), B(0, –4), C(4, –2) and D(2, 2) is
(a) 12 sq. units (b) 16 sq. units (c) 20 sq. units (d) 32 sq. units
46. The medians of a triangle meet at (0, –3) and its two vertices are at (–1, 4) and (5, 2). Then the third vertex
is at
(a) (4, 15) (b) (–4, –15) (c) (–4, 15) (d) (4, –15)
47. If he two vertices of a triangle are (7, 2) and (1,6) and its centroid is (4, 6) then the coordinate of the third
vertex are (a, b). The value of (a + b), is
(a) 13 (b) 14 (c) 15 (d) 16
FG 1 2 IJ FG 11 , 4 IJ
48. If in triangle ABC, A  (1, 10), circumcentre   ,
H 3 3 K and orthoentre 
H 3 3K then the

co-ordinates of mid-point of side opposite to A is


(a) (1, –11/3) (b) (1, 5) (c) (1, –3) (d) (1, 6)
49. Suppose ABC is a triangle with 3 acute angle A, B and C. The point whose coordinates are
(cos B – sin A, sin B – cos A) can be in the
(a) first and 2nd quadrant (b) second the 3rd quadrant
(c) third and 4th quadrant (d) second quandrnt only
50. Consider the points P(2, –4); Q(4, –2) and R = (7, 1). The points P, Q, R
(a) form an equilateral triangle (b) form a right angled triangle
(c) form an isosceles triangle which is not equilateral (d) are collinear
51. The line x = c cuts the triangle with corners (0, 0), (1, 1) and (9, 1) into two regions. For the area of the two
regions to be the same c must be equal to
(a) 5/2 (b) 3 (c) 7/2 (d) 3 or 15
52. A triangle has two of its vertices at (0, 1) and (2, 2) in the Cartesian plane. Its third vertex lies on the x-axis.
If the area of the traingle is 2 square units then the sum of the possible abscissae of the third vertex, is
(a) –4 (b) 0 (c) 5 (d) 6

MOMENTUM : Betiahata Chowk, Gorakhpur PH. 0551-2332808, 2205569 Page # 61


53. A point P(x, y) moves so that the sum of the distances from P to he coordinate axes is equal to the distance
from P to the point A(1, 1). The equation of the locus of P in the first quadrant is
(a) (x + 1) (y + 1) = 1 (b) (x + 1) (y + 1) = 2 (c) (x – 1) (y – 1) = 1 (d) (x – 1) (y – 1) = 2
54. Let A(2, –3) and B(–2, 1) be vertices of a  ABC. If the centroid of  ABC moves on the line
2x + 3y = 1, then the locus of the vertex C is
(a) 2x + 3y = 9 (b) 2x – 3y = 7 (c) 3x + 2y = 5 (d) 3x – 2y = 3
55. A stick of length 10 units rests against the floor and a wall or a room. If the stick begins to slide onthe floor
then the locus of its middle point is :
(a) x2 + y2 = 2.5 (b) x2 + y2 = 25 (c) x2 + y2 = 100 (d) none
56. AB is the diameter of a semicircle k, C is an arbitrary point on the
semicircle (other than A or B) and S is the centre of the crcle inseribed
intotriangle ABC, then measure of
(a) angle ASB changes as C moves on k.
(b) angle ASB is the same for all positions of C but it cannot be determined without knowing the radius
(c) angle ASB = 135o for all C (d) angle ASB = 150o for all C
57. Given the points A (0, 4) and B (0, – 4) the equation of the locus of the point P (x, y) such that |AP – BP | = 6 is
(a) 9x2 – 7y2 + 63 = 0 (b) 9x2 – 7y2 – 63 = 0 (c) 7x2 – 9y2 + 63 = 0 (d) 7x2 – 9y2 – 63 = 0
58. Each member of the family of parabolas y = ax2 + 2x + 3 has a maximum or a minimum point depending
upon the value of a. The equation to the locus of the maxima or minima for all possible values of ‘a’ is
(a) a straight line with slope 1 and y intercept 3.
(b) a straight line with slope 2 and y intercept 2.
(c) a straight line with slope 1 and x intercept 3.
(d) a straight line with slope 2 and y intercept 3.
59. A line passes through (2, 2) and cuts a triangle of area 9 square units from the first quadrant. The sum of all
possible values for the slope of such a line, is
(a) – 2.5 (b) – 2 (c) – 1.5 (d) – 1
60. A variable straight line passes through the points of intersection of the lines, x + 2y = 1 and 2x – y = 1 and
meets the co-ordinate axes in A and B. The locus of the middle point of AB is :
(a) x + 3y – 10xy = 0 (b) x – 3y + 10xy = 0 (c) x + 3y + 10 xy = 0 (d) none
61. A variable straight line passes through a fixed point (a, b) intersecting the co-ordinates axes at A and B. If
‘O’ is the origin then the locus of the centroid of the triangle OAB is :
(a) bx + ay – 3xy = 0 (b) bx + ay – 2xy = 0 (c) ax + by – 3xy = 0 (d) none
62. The equations of L1 and L2 are y = mx and y = nx, respectively. Suppose L1 makes twice as large of an
angle with the horizontal (measured counterclockwise from the positive x-axis) as does L2 and that L1 has
4 times the slope of L2. If L1 is not horizontal, then the value of the product (mn) equals
2 2
(a) (b)  (c) 2 (d) – 2
2 2
63. The extremities of the base of an isosceles triangle ABC are the points A(2, 0) and B(0, 1). If the equation
of the side AC is x = 2 then the slope of the side BC is
3 4 3
(a) (b) (c) (d) 3
4 3 2
64. The graph of the function, y = cos x cos (x + 2) – cos2 (x + 1) is :
(a) a straight line passing through (0, – sin21) with slope 2
(b) a straight line passing through (0, 0)
(c) a parabola with vertex (1, – sin21)
 
(d) a straight line passing through the point  ,  sin 2 1 and parallel to the x-axis.
2 
MOMENTUM : Betiahata Chowk, Gorakhpur PH. 0551-2332808, 2205569 Page # 62
65. A and B are any two points on the positive x and y axis respectively satisfying 2(OA) + 3(OB) = 10. If P
is the middle point of AB then the locus of P is
(a) 2x + 3y = 5 (b) 2x + 3y = 10
(c) 3x + 2y = 5 (d) 3x + 2y = 10
66. A line with gradient 2 intersects a line with gradient 6 at the point (40, 30). The distance between x-
intercepts of these lines, is
(a) 6 (b) 8 (c) 10 (d) 12
67. Locus of a point which is equidistant from the point (3, 4) and (5, – 2) is a straight line whose x-intercept
is
(a) 1/3 (b) 2/3 (c) 1 (d) –1/3
68. The diagonals of a parallelogram PQRS are along the lines x + 3y = 4 and 6x – 2y = 7. Then PQRS must
be a :
(a) rectangle (b) square
(c) cyclic quadrilateral (d) rhombus
69. The sides of a triangle ABC lie on the lines 3x + 4y = 0; 4x + 3y = 0 and x = 3. Let (h, k) be the centre of
the circle inscribed in ABC . The value of (h + k) equals
(a) 0 (b) 1/4 (c) – 1/4 (d) 1/2
70. If m and b are real number and mb>0, then the line whose equation is y = mx + b cannot contain the point
(a) (0, 2009) (b) (2009, 0) (c) (0, – 2009) (d) (20, – 100)
71. If the vertices P and Q of a triangle PQR are given by (2, 5) and (4, –11) respectively, and the point R
moves along the line N : 9x + 7y + 4 = 0, then the locus of the centroid of the triangle PQR is a straight line
parallel to
(a) PQ (b) QR (c) RP (d) N
72. The co-ordinates of the orthocentre of the triangle bounded by the lines, 4x – 7y + 10 = 0; x + y = 5 and
7x + 4y = 15 is :
(a) (2, 1) (b) (–1, 2) (c) (1, 2) (d) (1, –2)
73. If the x intercept of the line y = mx + 2 is greater than 1/2 then the gradient of the line lies in the interval
(a) (–1, 0) (b) (–1/4, 0) (c) ( , 4) (d) (– 4, 0)
74. Let the co-ordinates of the two points A and B be (1, 2) and (7, 5) respectively. The line AB is rotated
through 45o in anti clockwise direction about the point of trisection of AB which is nearer to B. The
equation of the line in new position is :
(a) 2x – y – 6 = 0 (b) x – y – 1 = 0 (c) 3x – y – 11 = 0 (d) none of these
75. The greatest slope along the graph represented by the equation 4x2 – y2 + 2y – 1 = 0, is
(a) – 3 (b) – 2 (c) 2 (d) 3
76. Let PS be the median of the triangle with vertices, P (2, 2), Q (6, –1) and R (7, 3). The equation of the line
passing through (1, –1) and parallel to PS is
(a) 2x – 9y – 7 = 0 (b) 2x – 9y – 11 = 0 (c) 2x + 9y – 11 = 0 (d) 2x + 9y + 7 = 0
77. A ray of light passing through the point A(1, 2) is reflected at a point B on the x-axis and then passes
through (5, 3). Then the equation of AB is :
(a) 5x + 4y = 13 (b) 5x – 4y = – 3 (c) 4x + 5y = 14 (d) 4x – 5y = – 6
78. In a triangle ABC, side AB has the equation 2x + 3y = 29 and the side AC has the equation, x + 2y = 16.
If the mid – point of BC is (5, 6) then the equation of BC is :
(a) x – y = – 1 (b) 5x – 2y = 13 (c) x + y = 11 (d) 3x – 4y = – 9
79. ABC is an isosceles triangle. If the co-ordinates of the base are (1, 3) and (– 2, 7), then co-ordinates of
vertex A can be
 1   1  5   1
(a)   ,5  (b)   ,5  (c)  , 5  (d)  7, 
 2   8  6   8

MOMENTUM : Betiahata Chowk, Gorakhpur PH. 0551-2332808, 2205569 Page # 63


80. Co-ordinates of the orthocentre of the triangle whose vertices are A(0, 0), B(3, 4) and C(4, 0) is
 3
(a) (3, 1) (b) (3, 4) (c) (3, 3) (d)  3, 
 4
81. Number of lines that can be drawn through the point (4, – 5) so that its distance from (– 2, 3) will be equal
to 12 is equal to
(a) 0 (b) 1 (c) 2 (d) 3
82. Two mutually perpendicular straight lines through the origin from an isosceles triangle with the line
2x + y = 5. Then the area of the triangle is :
(a) 5 (b) 3 (c) 5/2 (d) 1
83. Let the lines (y – 2) = m1(x – 5) and (y + 4) = m2(x – 3) intersect at right angles at P (where m1 and m2 are
parameters). If locus of P is x2 + y2 + gx + fy + 7 = 0, then (f – g) equals
(a) 1 (b) 2 (c) 8 (d) 10
84. P lies on the line y = x and Q lies on y = 2x. The equation for the locus of the mid point of PQ,
if | PQ | = 4, is
(a) 25x2 + 36xy + 13y2 = 4 (b) 25x2 – 36xy + 13y2 = 4
2 2
(c) 25x – 36xy – 13y = 4 (d) 25x2 + 36xy – 13y2 = 4
85. The vertex of a right angle of a right angled triangle lies on the straight line 2x + y – 10 = 0 and the two other
vertices, at points (2, –3) and (4, 1) then the area of triangle in sq. units is
33
(a) 10 (b) 3 (c) (d) 11
5
86. Point ‘P’ lies on the line l {(x, y) | 3x + 5y = 15}. If ‘P’ is also equidistant from the coordinate axes, then
P can be located in which of the four quadrants.
(a) I only (b) II only (c) I or II only (d) IV only
87. If each of the points (a, 6) and (3, b) lies on the line joining the points (3, 2) and (5, 1) then the point (a, b)
lies on the line :
(a) 3x + 4y + 7 = 0 (b) 2x + 3y – 7 = 0
(c) 4x – 3y – 7 = 0 (d) 3x – 2y – 7 = 0

88. The line L1 given by x  y  1 passes through the point M(13, 32). The line L2 is parallel to L1 and has the
5 b
x y
equation   1 . Then the distance between L1 and L2 is
c 3
17 23 23
(a) 17 (b) (c) (d)
15 17 15
89. The area of the parallelogram formed by the lines 3x + 4y = 7a; 3x + 4y = 7b; 4x + 3y = 7c and
4x + 3y = 7d is
| (a  b)(c  d) |
(a) (b) 49 | (a  b)(c  d) |
7
| (a  b)(c  d) |
(c) (d) 7 | (a  b)(c  d) |
49
90. If x1, y1 are the roots of x2 + 8x – 20 = 0, x2, y2 are the roots of 4x2 + 32x – 57 = 0 and x3, y3 are the roots
of 9x2 + 72x – 112 = 0, then the points, (x1, y1), (x2, y2) and (x3, y3)
(a) are collinear (b) form an equilateral triangle
(c) form a right angled isosceles triangle (d) are concyclic

MOMENTUM : Betiahata Chowk, Gorakhpur PH. 0551-2332808, 2205569 Page # 64


91. Let (x1, y1) ; (x2, y2) and (x3, y3) are the vertices of a triangle ABC respectively. D is a point on BC such that
BC = 3BD. The equation of the line through A and D, is
x y 1 x y 1 x y 1 x y 1
(a) x1 y1 1  2 x1 y1 1  0 (b) 3 x1 y1 1  x1 y1 1  0
x2 y2 1 x3 y3 1 x2 y2 1 x3 y3 1

x y 1 x y 1 x y 1 x y 1
(c) x1 y1 1  3 x1 y1 1  0 (d) 2 x1 y1 1  x1 y1 1  0
x2 y2 1 x3 y3 1 x2 y2 1 x3 y3 1
92. Equation of a straight line passing through the origin and making with x-axis an angle twice the size of the
angle made by the line y = 0.2 x with the x-axis, is :
(a) y = 0.4 x (b) y = (5/12) x
(c) 6y – 5x = 0 (d) none of these
93. A triangle ABC is formed by the lines 2x – 3y – 6 = 0 ; 3x – y + 3 = 0 and 3x + 4y – 12 = 0. If the points
P(,0) and Q(0, ) always lie on or inside the ABC, then :
(a)   [ 1, 2]and   [ 2, 3] (b)   [ 1, 3]and   [ 2, 4]
(c)   [2, 4]and   [ 3, 4] (d)   [1, 3]and   [2,3]
94. The co-ordinates of a point P on the line 2x – y + 5 = 0 such that | PA – PB | is maximum where A is
(4, – 2) and B is (2, – 4) will be :
(a) (11, 27) (b) (–11, –17) (c) (–11, 7) (d) (0, 5)
95. If the lines
x  (sin )y  cos   0 
x  (cos )y  sin   0  pass through the same point where   R then  lies in the interval
x  (sin )y  cos   0 

(a) [–1, 1] (b) [ 2, 2] (c) [–2, 2] (d) (, )


96. Two points A(x1, y1) and B(x2, y2) are chosen on the graph of f(x) = ln x with 0 < x1 < x2. The points C and
D trisect line segment AB with AC < CB. Through C a horizontal line is drawn to cut the curve at E(x3, y3).
If x1 = 1 and x2 = 1000 then the value of x3 equals
(a) 10 (b) 10 (c) (10)2/3 (d) (10)1/3
97. Area of the quadrilateral formed by the lines | x | + | y | = 2 is :
(a) 8 (b) 6 (c) 4 (d) none
98. The number of possible straight lines, passing through (2, 3) and forming a triangle with coordinate axes,
whose area is 12 sq. units, is
(a) one (b) two (c) three (d) four
99. Let A  (3, 2) and B  (5,1) . ABP is an equilateral triangle is constructed on the side of AB remote from
the origin then the orthocentre of triangle ABP is
 1 3   1 3 
(a)  4  3,  3  (b)  4  3,  3 
 2 2   2 2 
 1 3 1   1 3 1 
(c)  4  3,  3 (d)  4  3,  3
 6 2 3   6 2 3 

MOMENTUM : Betiahata Chowk, Gorakhpur PH. 0551-2332808, 2205569 Page # 65


100. Family of lines represented by the equation (cos   sin )x  (cos   sin )y  3(3cos   sin )  0
passes through a fixed point M for all real values of  . The reflection of M in the line x – y = 0, is
(a) (6, 3) (b) (3, 6) (c) (–6, 3) (d) (3, – 6)
4
101. A is a point on either of two lines y  3 | x | 2 at a distance ofunits from their point of intersection.
3
The co-ordinates of the foot of perpendicular from A on the bisector of the angle between them are
 2   2 
(a)   ,2 (b) (0, 0) (c)  ,2 (d) (0, 4)
 3   3 
102. The line (k + 1)2x + ky – 2k2 – 2 = 0 passes through a point regardless of the value k. Which of the
following is the line with slope 2 passing through the point ?
(a) y = 2x – 8 (b) y = 2x – 5 (c) y = 2x – 4 (d) y = 2x + 8
103. Given the family of lines, a(2x + y + 4) + b(x – 2y – 3) = 0. Among the lines of the family, the number of
lines situated at a distance of 10 from the point M(2, – 3) is :
(a) 0 (b) 1 (c) 2 (d) 
104. The co-ordinates of the point of reflection of the origin (0, 0) in the line 4x – 2y – 5 = 0 is
 4 2
(a) (1, – 2) (b) (2, – 1) (c)  ,   (d) (2, 5)
5 5
105. m, n are integer with 0 < n < m. A is the point (m, n) on the cartesian plane. B is the r eflection of A in the
line y = x. C is the reflection of B in the y-axis, D is the reflection of C in the x-axis and E is the reflection
of D in the y-axis. The area of the pentagon ABCDE is
(a) 2m(m + n) (b) m(m + 3n) (c) m(2m + 3n) (d) 2m(m + 3n)
106. Consider a parallelogram whose sides are represented by the lines 2x + 3y = 0; 2x + 3y – 5 = 0; 3x – 4y
= 0 and 3x – 4y = 3. The equation of the diagonal not passing through the origin, is
(a) 21x – 11y + 15 = 0 (b) 9x – 11y + 15 = 0
(c) 21x – 29y – 15 = 0 (d) 21x – 11y – 15 = 0
107. In a triangle ABC, if A(2, – 1) and 7x – 10y + 1 = 0 and 3x – 2y + 5 = 0 are equations of an altitude and
an angle bisector respectively drawn from B, then equation of BC is
(a) x + y + 1 = 0 (b) 5x + y + 17 = 0
(c) 4x + 9y + 30 = 0 (d) x – 5y – 7 = 0
108. A variable line L = 0 is drawn through O(0, 0) to meet the lines L1 : x + 2y – 3 = 0 and L2 : x + 2y + 4 = 0 at
1 1 1
points M and N respectively. A point P is taken on L = 0 such that 2
 2
 . Locus of P is
OP OM ON 2
2 2 144 2 144
(a) x  4y  (b) (x  2y) 
25 25
2 2 144 2 144
(c) 4x  y  (d) (x  2y) 
25 25
109. If the straight lines, ax + amy + 1 = 0, bx + (m + 1) by + 1 = 0 and cx + (m + 2)cy + 1 = 0, m  0 are
concurrent then a, b, c are in :
(a) A.P. only for m = 1 (b) A.P. for all m (c) G.P. for all m (d) H.P. for all m
110. P is a point inside the triangle ABC. Lines are drawn through P, parallel to the sides of the triangle. The three
resulting triangles with the vertex at P have areas 4, 9 and 49 sq. units. The area of the triangle ABC is
(a) 2 3 (b) 12 (c) 24 (d) 144

MOMENTUM : Betiahata Chowk, Gorakhpur PH. 0551-2332808, 2205569 Page # 66


 11 2 
111. The position vectors of vertices of ABC are (1, – 2), (– 7, 6) and  ,  respectively. The measure
 5 5
of the interior angle A of the ABC , is
(a) acute and lies in (75o, 90o) (b) acute and lies in (60o, 75o)
o o
(c) acute and lies in (45 , 60 ) (d) obtuse and lies (120o, 150o)
112. The area of the triangular region in the first quadrant bounded on the left by the y-axis, bounded above by
the line 7x + 4y = 168 and bounded below by the line 5x + 3y = 121, is
50 52 53
(a) (b) (c) (d) 17
3 3 3
113. Let A(5, 12), B(13cos ,13sin ) and C(13sin , 13cos ) are angular points of ABC where
 R . The locus of orthocentre of ABC is
(a) x – y + 7 = 0 (b) x – y – 7 = 0 (c) x + y – 7 = 0 (d) x + y + 7 = 0
114. Number of straight line(s) passing through the point P(4, 3) whose x-intercept is a prime number and
whose y-intercept is a positive integer, is equal to
(a) 0 (b) 1 (c) 2 (d) more than 2 but finite
115. Let PQR be a right angled isosceles triangle, right angled at P(2, 1). If the equation of the line QR is
2x + y = 3, then the equation representing the pair of lines PQ and PR is
(a) 3x2 – 3y2 + 8xy + 20x + 10y + 25 = 0 (b) 3x2 – 3y2 + 8xy – 20x – 10y + 25 = 0
(c) 3x2 – 3y2 + 8xy + 10x + 15y + 20 = 0 (d) 3x2 – 3y2 – 8xy – 10x – 15y – 20 = 0
116. If the straight lines joining the origin and the points of intersection of the curve
5x2 + 12xy – 6y2 + 4x – 2y + 3 = 0 and x + ky – 1 = 0
are equally inclined to the co-ordinate axes then the value of k :
(a) is equal to 1 (b) is equal to – 1
(c) is equal to 2 (d) does not exist in the set of real numbers.
117. The angles between the straight lines joining the origin to the points common to
7x2 + 8y2 – 4xy + 2x – 4y – 8 = 0 and 3x – y = 2 is :
  
(a) tan  2 (b) (c) (d)
3 4 2
118. A pair of perpendicular straight lines is drawn through the origin forming with the line 2x + 3y = 6 an
isosceles triangle right angled at the origin. The equation to the line pair is :
(a) 5x2 – 24xy – 5y2 = 0 (b) 5x2 – 26xy – 5y2 = 0
(c) 5x2 + 24xy – 5y2 = 0 (d) 5x2 + 26xy – 5y2 = 0
119. If the line y = mx bisects the angle between the lines ax2 + 2h xy + by2 = 0 then m is a root of the quadratic
equation :
(a) hx2 + (a – b) x – h = 0 (b) x2 + h(a – b) x – 1 = 0
(c) (a – b) x2 + hx – (a – b) = 0 (d) (a – b) x2 – hx – (a – b) = 0
120. If the equation ax – 6xy + y + 2gx + 2fy + c = 0 represents a pair of lines whose slopes are m and m2,
2 2

then sum of all possible values of a is


(a) 17 (b) – 19 (c) 19 (d) – 17
121. Through a point A on the x-axis a straight line is drawn parallel to y-axis so as to meet the pair of straight
lines ax2 + 2hxy + by2 = 0 in B and C. If AB = BC then
(a) h2 = 4ab (b) 8h2 = 9ab (c) 9h2 = 8ab (d) 4h2 = ab
122. Suppose that a ray of light leaves the point (3, 4), reflects off the y-axis
towards the x-axis, reflects off the x-axis, and finally arrives at the point
(8, 2). The value of x, is
1 1 2 1
(a) x  4 (b) x  4 (c) x  4 (d) x  5
2 3 3 3
MOMENTUM : Betiahata Chowk, Gorakhpur PH. 0551-2332808, 2205569 Page # 67
123. If A(1, p2) ; B(0, 1) and C(p, 0) are the coordinates of three points then the value of p for which the area
of the triangle ABC is minimum, is
1 1 1 1
(a) (b)  (c) or  (d) none
3 3 3 3
2 2
124. Let S = {(x, y) | x + 2xy + y – 3x – 3y + 2 = 0}, then S
(a) consists of two coincident lines.
(b) consists of two parallel lines which are not coincident.
(c) consists of two intersecting lines.
(d) is a parabola.

PAIR OF STRAIGHT LINES

1. The separate equations of the straight lines whose joint equaton is x2 – 5xy + 6y2 = 0, are
(a) x + 2y = 0, x – 3y = 0 (b) x – 2y = 0, x – 3y = 0
(c) x – 2y = 0, x + 3y = 0 (d) x – 2y = 0, x – 3y = 0
2 2
2. If x - 3xy + y + 3x - 5y + 2 = 0 represents a pair of straight lines, then the value of  is
(a) 1 (b) 4 (c) 3 (d) 2
2 2
3. If x – 10xy + 12y + 5x – 16y – 3 = 0 represents a pair of straight lines, then the value of  is
(a) 4 (b) 3 (c) 2 (d) 1
2 2
4. The value of K such that 3x – 11xy + 10y – 7x + 13y + K = 0 may represent a pair of st. lines, is
(a) 3 (b) 4 (c) 6 (d) 8
2 2
5. The equation ax + by + cx + cy = 0, c  0 represents a pair of st. lines if
(a) a + b = 0 (b) b + c = 0 (c) c + a = 0 (d) a + b + c = 0
2 2
6. If ax – y + 4x – y = 0 represens a pair of lines, then a =
(a) –16 (b) 16 (c) 4 (d) – 4
2 2
7. The equation 3x + 2hxy + y = 0 represents a pair of st. lines passing thro’ the origin. The two lines are
(a) real and distinct if h2 > 3 (b) real and distinct if h2 > 9
(c) real and coincident if h2 = 3 (d) real and coincident if h2 > 3.
8. If h2 = ab, then the lines represented by ax2 + 2hxy + by2 = 0 are
(a) parallel (b) perpendicular (c) coincident (d) none
2 2
9. The angle between the pair of st. lines x + 4y – 7xy = 0 is
F 33 I F 33 I
(a) tan–1
1
3
(b) tan–1 3 (c) tan–1 GH 5 JK (d) tan–1 GH 10 JK
10. The angle between the lines 3xy  y 2  0 is-
(a) 30o (b) 450 (c) 600 (d) 900
11. If the angle between the two lines represented by 2x2 + 5xy + 3y2 + 7y + 4 = 0 is tan–1 m. then m =
(a) 1/5 (b) 1 (c) 7/5 (d) 7
2 2
12. The lines represented by the equation Ax + 2Bxy + Hy = 0 are  if
(a) A + B = 0 (b) A + H = 0 (c) B + H = 0 (d) AH = – 1
13. If the equation 3x2 – 8xy + y2 = 0 repersents two perpendicular lines, then the value of  is
(a) 3 (b) – 3 (c) 2 (d) None of these
14. If the lines represented by 2x2 + 8xy + ky2 = 0 are coincident, then the values of k is
(a) 8 (b) – 8 (c) 4 (d) None of these

MOMENTUM : Betiahata Chowk, Gorakhpur PH. 0551-2332808, 2205569 Page # 68


15. The pair of st. lines  to the pair ax2 + 2hxy + by2 = 0 has the equation
(a) ax2 – 2hxy + by2 = 0 (b) ay2 + 2hxy + bx2 = 0
(c) bx2 + 2hxy + ay2 = 0 (d) bx2 – 2hxy + ay2 = 0
16. If ax2 + 2hxy + by2 + 2gx + 2fy + c = 0 represents parallel st. lines, then
(a) hf = bg (b) h2 = bc (c) a2f = b2g (d) none of these
2 2
17. If one of the lines given by 6x – xy + 4cy = 0 is 3x + 4y = 0, then c equals
(a) 3 (b) – 1 (c) 1 (d) –3
2 2
18. Out of two st. lines represented by an equation ax + 2hxy + by = 0 if one will be y = mx, then
(a) a + 2hm + bm2 = 0 (b) b + 2hm + am2 = 0
(c) h + 2am + bm2 = 0 (d) h + 2hm + am2 = 0
19. The angle between the lines represented by x + 2xy sec  + y2 = 0 is
2

(a) 4 (b) 2 (c)  (d) None of these


20. c h
The angle between the pair of straight lines y 2 sin 2   xy sin 2   x 2 cos2   1  0 is
(a)  / 3 (b)  / 4 (c) 2 / 3 (d) none of these
2 2
21. If the equation 3x + 6xy + my = 0 represents a pair of straight lines inclined at an angle , then m is
equal to
(a) 3 (b) 6 (c) 9 (d) any real number
2 2
22. One of the lines ax + 2hxy + by = 0 will bisect an angle between the coordinates axes if
(a) (a + b)2 = 4h2 (b) (a + b)2 = 2h2 (c) (a + b)2 = h2 (d) None of these
2 2 2 2
23. If the slope of one of the lines given by a x + 2hxy + b y = 0 be three times of the other then h is equal to
2 2
(a) 2 3 ab (b) 2 3 ab (c) ab (d)  ab
3 3
24. The gradient of one of the lines x2 + hxy + 2y2 = 0 is twice that of the other, then h =
3
(a)  3 (b)  (c)  2 (d)  1
2
25. If the slope of one of the lines given by ax2 + 2hxy + by2 = 0 be the square of the other, then
(a) ab ( a + b) + 6abh + 8h3 = 0 (b) ab ( a + b) – 6abh + 8h3 = 0
3
(c) ab ( a + b) + 3abh + 4h = 0 (d) None of these
26. If one of the straight lines given by the equation ax2 + 2hxy + by2 = 0 coincides with one of those given
by a ' x2 + 2h ' xy + b ' y2 = 0, then
(a) (ab ' – a ' b)2 + 4 (ah ' – a ' h) (bh ' – b ' h) = 0
(b) (ab ' – a ' b)2 + 4 (a ' h– ah ' ) (bh ' – b ' h) = 0
(c) (ab ' + a ' b)2 + 4 (a ' h – ah ' ) (bh ' – b ' h) = 0
(d) None of these
27. The two straight line x2 (tan2 + cos2) – 2xy tan  + y2 sin2= 0 make with the axis of x angles such
that the difference of their tangents is
(a) 4 (b) 3 (c) 2 (d) None of these
2 2
28. If the lines px – qxy – y = 0 make the angle  and  with x-axis, then the value of tan ( + ) is
q q p p
(a) (b) (c) (d)
1 p 1 p 1 q 1 q
29. The coordinates of the centroid of the triangle whose sides are
12x2 – 20xy + 7y2 = 0 and 2x – 3y + 4 = 0 are
FG 8 ,  8IJ FG
8 8 IJ FG 8 , 8IJ FG
8 8 IJ
(a) H3 3 K H
(b)  ,
3 3 K (c) H 3 3K H
(d)  ,  ,
3 3 K

MOMENTUM : Betiahata Chowk, Gorakhpur PH. 0551-2332808, 2205569 Page # 69


30. The equation 3x2 - 8xy - 3y2 = 0 and x + 2y = 3 represent the sides of a triangle, which is
(a) equilateral (b) right angled (c) isosceles (d) isosceles rt. angled
3 2
31. The three lines given by y - 9x y = 0 form a triangle which is
(a) equilateral (b) isoceles (c) right angled (d) none
3 2 2 3
32. The equation x + x y - xy = y represents -
(a) three real straight lines
(b) lines in which two of them are perpendicular to each other
(c) lines in which two of them are coincident
(d) none of these
33. The equation m (x3 – 3xy2) + y3 – 3x2y = 0 represents three straight lines
(a) which are equally inclined to one another (b) two of which are at right angles
(c) two of which are coincident (d) which pass through origin
34. The orthocentre of the triangle whose three sides are given by the combined equation
(15x2 + 16xy – 48y2) (y – 15) = 0, is
(a) (0, – 33) (b) (0, 33) (c) (33, 0) (d) (–33, 0)
35. The combined equation of two sides of an equilateral triangle is x2 - 3y2 - 2x + 1 = 0. If the length of a
side of the triangle is 4 then the equation of the third side is -
(a) x  2 3  1 (b) y  2 3  1 (c) y  2 3  1 (d) x  2 3
36. If xy + x + y + 1 = 0, x + ay - 3 = 0 are concurrent, then a = 0
(a) 3 (b) –4 (c) 2 (d) none
37. The diagonals of a square are along the pair of lines whose equation is 2x2 - 3xy - 2y2 = 0. If (2, 1) is a
vertex of the square then another vertex consecutive to it can be -
(a) (1, -2) (b) (1, 4) (c) (-1, 2) (d) (-1, -4)
2 2
38. The image of the pair of lines represented by ax + 2hxy + by = 0 by the line mirror y = 0 is -
(a) ax2 - 2hxy - by2 = 0 (b) bx2 - 2hxy + ay2 = 0
(c) bx2 + 2hxy + ay2 = 0 (d) ax2 - 2hxy + by2 = 0
39. Given the equation 4x2 + 2 3 xy + 2y2 = 1, the angle through which the axes be rotated so that the
trem in xy by wanting from the transformed equation, is
  
(a) (b) (c) (d) None of these
6 3 3
40. The two sets of pair of lines given by ax2 - 2xy + by2 = 0 and bx2 - 2xy + ay2 = 0 be such that each
pair bisects the angle between the other pair, then
(a) a - b = 2 (b) a + b = 0 (c) a + b = 2 (d) a - b = -2
2 2 2 2
41. If pairs of straight lines x – 2pxy – y = 0 and x – 2qxy – y = 0 be such that each pair bisects the
angle between the other pair, then
(a) pq = 1 (b) pq = – 1 (c) pq = 2 (d) None of these
42. The lines bisecting the angle between the bisectors of the angles between the lines ax2 + 2hxy + by2 = 0
are given by
(a) (a – b) (x2 – y2) – 4hxy = 0 (b) (a – b) (x2 + y2) + 4hxy = 0
(c) (a – b) (x2 – y2) + 4hxy = 0 (d) None of these
43. The straight lines represented by the equation 135x2 – 136xy + 33y2 = 0 are equally inclined to the line
(a) x – 2y = 7 (b) x + 2y = 7 (c) x – 2y = 4 (d) 3x + 2y = 4
2 2
44. If the lines represented by x - 2pxy - y = 0 are rotated about the origin through an angle  one in
clockwise direction and other in anti-clockwise direction, then the equation of the bisectors of the angle
between the lines in the new position is -
(a) px2 + 2xy - py2 = 0 (b) px2 + 2xy + py2 = 0
2 2
(c) x - 2pxy + y = 0 (d) None of these

MOMENTUM : Betiahata Chowk, Gorakhpur PH. 0551-2332808, 2205569 Page # 70


45. The area bounded by the angle bisector of the lines x2 – y2 + 2y = 1 and the line x + y = 3 is
(a) 2 (b) 3 (c) 4 (d) 6
2 2
46. If ax + 2hxy + by + 2gx + 2f y + c = 0 represents two parallel straight lines, then
(a) h2 = ab (b) g2 = ac (c) bg2 = af2 (d) ag2 = bf2
47. If the equation 12x2 + 7xy – py2 – 18x + qy + 6 = 0 represents a pair of prependicular straight lines,
then
(a) p = 12, q = 1 (b) p = 1, q = 12 (c) p = – 1, q = 12 (d) p = 1, q = – 12
48. The equation x2 - 3xy + y2 + 3x - 5y + 2 = 0 when  is a real number, represents a pair of straight
lines. If O is the angle between the lines, then cosec2  =
(a) 3 (b) 9 (c) 10 (d) 100
49. The equation of second degree x 2  2 2 xy  2 y 2  4 x  4 2 y  1  0 represents a pair of straight
lines, the distance between them is
(a) 4 (b) 4 / 3 (c) 2 (d) 2 3
50. The equation of two straight lines through the point (x1, y1) and  to the lines given by
ax2 + 2hxy + by2 = 0 is
(a) b (x – x1)2 – 2h (x – x1) (y – y1) + a (y – y1)2 = 0
(b) b (x – x1)2 + 2h (x – x1) (y – y1) + a (y – y1)2 = 0
(c) a (x – x1)2 – 2h (x – x1) (y – y1) + b (y – y1)2 = 0
(d) None of these
51. The equation of two straight lines through the point (x1, y1) and parallel to the lines given by
ax2 + 2hxy + by2 = 0 is
(a) a (y – y1)2 + 2h (x – x1) (y – y1) + b (x – x1)2 = 0
(b) a (y – y1)2 – 2h (x – x1) (y – y1) + b (x – x1)2 = 0
(c) b (y – y1)2 + 2h (x – x1) (y – y1) + a (x – x1)2 = 0
(d) None of these
52. One of the bisectors of the angle between the lines a(x - 1)2 + 2h (x - 1) (y - 2) + b (y - 2)2 = 0 is
x + 2y - 5 = 0 The other bisector is -
(a) 2x - y = 0 (b) 2x + y = 0 (c) 2x + y - 4 = 0 (d) x - 2y + 3 = 0
53. If the lines joining the origin to the points of intersection of y = mx + 1 with x2 + y2 = 1 are  , then m is
equal to
(a) 2 (b) 1 (c) – 1 (d) – 2
54. The equations of lines joining the origin to the point of intersection of circle x2 + y2 = 3 and the line
x + y = 2 are
(a) y – (3 + 2 2 ) x = 0 (b) x – (3 + 2 2 ) y = 0
(c) y – (3 – 2 2 ) x = 0 (d) x – (3 – 2 2 ) y = 0
2 2
55. If the curve x + y + 2gx + 2fy + c = 0 intercepts on the line lx + my = 1, a length which subtneds a
right angle at the origin, then
(a) c (l2 + m2) + 2 (gl + fm + 1) = 0 (b) c (l2 + m2) – 2 (gl + fm + 1) = 0
2 2
(c) c (l + m ) + 2 (gl + fm – 1) = 0 (d) None of these
56. The pair of lines joining the origin to the points of intersection of the curves ax2 + 2hy + by2 + 2gx = 0
and a’x2 + 2h’xy2 + b’y2 + 2g’x = 0 will be at right angles to one another if -
(a) g (a’ + b’) = g’ (a + b) (b) g (a + b) = g’ (a’ + b’)
(c) gg’ = (a + b) (a’ + b’) (d) none of these
57. If two of the straight lines represented by ax3 + bx2y + cxy2 + dy3 = 0 ae at right angles, then
(a) a2 + ac + bd – d2 = 0 (b) a2 + ac – bd + d2 = 0
2 2
(c) a – ac + bd + d = 0 (d) a2 + ac + bd + d2 = 0

MOMENTUM : Betiahata Chowk, Gorakhpur PH. 0551-2332808, 2205569 Page # 71


58. The equation x4 + bx3y + cx2y2 + dxy3 + ey4 = 0 representes two pairs of perpendicular straight lines if
(a) b + d = 1 and e = – 1 (b) b + d = 0 and e = – 1
(c) b + d = 0 and e = 1 (d) None of these
59. All chords of the curve 3x2 – y2 – 2x + 4y = 0 which subtend a right angle at the origin, pass through the
fixed point
(a) (1, 2) (b) (1, – 2) (c) (–1, 2) (d) None of these
60. The four sides of a quadrilateral are given by the equation xy (x - 2) (y - 3) = 0. The equation of the line
parallel to x - 4y = 0 that divides the quadrilateral in two equal areas is -
(a) x - 4y + 5 = 0 (b) x - 4y - 5 = 0 (c) 4y = x + 1 (d) 4y + 1 = x
61. If the line y = tan  x cut the curve x3 + xy2 + 2y2 + 3x + 1 = 0 at the points A, B and C. If OA, OB,
OC and in H.P. then tan  is -
(a) 1 (b) –1 (c) 2 (d) –2
62. If the line y  x 3 cuts the curve x3 + y3 + 3xy + 5x2 + 3y2 + 4x + 5y + 1 = 0 at the points A, B, C then
OA . OB . OC is equal to

(a)
4
13
d3 3 1 i (b) 3 3  1 (c)
1
3
d
27 3 i (d) none of these
63. The combined equation of three sides of a triangle is x2(2x + 3y - 6) - y2(2x + 3y - 6) = 0. If (-2, a) is
an interior point and (b, 3) is an exterior point of the triangle (opp. to O with respect to AB), then -
10
(a) 2  a  (b) 2  a  2 (c)  3  b  3 (d) 3  b  1
3 2
64. A pair of straight lines, drawn through the origin, form with the line 2x + 3y = 6 an isosceles triangle right
angled at the origin. The equation of the pair of straight lines is
(a) 5x2 – 24xy + 5y2 = 0 (b) 5x2 – 24xy – 5y2 = 0
(c) 5x2 + 24xy – 5y2 = 0 (d) None of these
65. If the distance of a point (x1, y1) from each of two straight lines, which pass through the origin of coordi-
nates, is , then the two lines are given by
(a) (x1y – xy1)2 = 2 (x2 + y2) (b) (x1y + xy1)2 = 2 (x2 + y2)
2 2 2 2
(c) (x1y – xy1) =  (x + y ) (d) None of these
66. The equation of the image of the lines y = x by the line x = 2 is
(a) y = x  4 (b) y = x + 4 (c) y + 4 = x (d) None of these

67. LM OP
The curve y  1 x 2  2 x  27 where [x] is the greatest integer less than or equal to x, 6  x  9
N6 Q
represents -
(a) A parabola (b) part of the parabola
(c) A straight line (d) two straight line segment

MOMENTUM : Betiahata Chowk, Gorakhpur PH. 0551-2332808, 2205569 Page # 72


EXERCISE-3
ADVANCED EXERCISE
OBJECTIVE (SINGLE CORRECTS) :
1. If the straight lines joining the origin and the points of intersection of the curve 5x2 + 12xy – 6y2 + 4x – 2y
+ 3 = 0 and x + ky – 1 = 0 are equally inclined to the x-axis then the value of k is :
(a) 1 (b) –1 (c) 2 (d) 3
2. Drawn from the origin are two mutually perpendicular straight lines froming an isosceles together with the
straight line, 2x + y = a. Then the area of triangle is :
a2 a2 a2
(a) (b) (c) (d) None
2 3 5
3. Equation of bisector of the angle between two lines 3x – 4y + 12 = 0 and 12x – 5y + 7 = 0 which contains
point (–1, 4) in its region is :
(a) 21x + 27 y – 121 = 0 (b) 21x – 27y + 121 = 0
3x  4y  12 12x  5y  7
(c) 21x + 27y + 191 = 0 (d) 
5 13
2
4. The point (a , a + 1) lies in the angle between the lines 3x – y + 1 = 0 and x + 2y – 5 = 0 containing the
origin if :
1  1 
(a) a  ( 3, 0)   ,1 (b) a  (  ,3)   ,1
3  3 
 1 1 
(c) a   3,  (d) a   , 
 3 3 
5. A ray of light through (2, 1) is reflected at a pint A on the y-axis and then passes through the point (5, 3).
Then co-ordinates of A are :
 11  5  11  3
(a)  0,  (b)  0,  (c)  0,  (d)  0, 
 7  11  5  5
6. The combined equation of the pair of lines through (3, –2) and parallel to the lines x2 – 4xy + 3y2 = 0 is :
(a) x2 – 4xy + 3y2 – 14x – 24y – 45 = 0 (b) x2 + 4xy + 3y2 + 14x – 24y – 45 = 0
(c) x2 – 4xy +3y2 – 14x + 24y + 45 = 0 (d) x2 + 4xy – 3y2 – 14x + 24y + 45 = 0
7. If (–2, 6) is the image of the point (4, 2) with respect to the line L = 0, then L is equal to :
(a) 3x – 2y + 11 = 0 (b) 2x – 3y + 11 = 0 (c) 3x – 2y + 5 = 0 (d) 6x – 4y + 1 = 0
8. A man starts from the point P(–3, 4) and reaches point Q (0, 1) touching x axis at R such that PR + RQ is
minimum, then the point R is :
(a) (3/5, 0) (b) (–3/5, 0) (c) (–2/5, 0) (d) (–2, 0)
9. The equation of line segment AB is y = x. If A & B lie on same side of line mirror 2x – y = 1, then the
equation of image of AB with respect to line mirror 2x – y = 1 is :
(a) y = 7x – 5 (b) y = 7x – 6 (c) y = 3x – 7 (d) y = 6x – 5
10. ABC is a variable triangle such that A is (1, 2) B and C lie on y = x +  (where  is variable), then locus
of the orthocentre of triangle ABC is :
(a) (x – 1)2 + y2 = 4 (b) x + y = 3 (c) 2x – y = 0 (d) x + 2y = 0
11. The line 2x + y = 4 meet x-axis at A and y-axis at B. The perpendicular bisector of AB meets the
horizontal line through (0, –1) at C. Let G be the centroid of ABC. The perpendicular distance from
G to AB equals
5
(a) 5 (b) (b) 2 5 (d) 3 5
3
MOMENTUM : Betiahata Chowk, Gorakhpur PH. 0551-2332808, 2205569 Page # 73
12. Let ABC be a triangle. Let A be the point (1, 2), y = x is the perpendicular bisector of AB and x – 2y + 1
= 0 is the angle bisector of angle C. If the equation of BC is given by ax + by – 5 = 0, then the value of a
+ b is :
(a) 1 (b) 2 (c) 3 (d) 4
13. I (1, 0) is the centre of in circle of triangle ABC, the equation of BI is x – 1 = 0 and equation of CI is x –
y – 1 = 0, then angle BAC is :
   2
(a) (b) (c) (d)
4 3 2 3
14. If the points where the lines 3x – 2y – 12 = 0 and x + ky + 3 = 0 intersect both the coordinate axes are
concyclic, then number of possible real values of k is :
(a) 1 (b) 2 (c) 3 (d) 4
15. In the figure shown, OABC is a rectangle with dimensions OA = 3 units and OC = 4 units. If AD = 1.5
units then slope of diagonal OB will be :

1 2 1 1
(a) (b) (c) (d)
2 3 2 3
16. In a ABC, the equations of right bisectors of sides AB and CA are 3x + 4y = 20 and 8x + 6y = 65
respectively. If the vertex A be (10, 10), then the area of ABC will be :
(a) 14 (b) 21 (c) 42 (d) 63
17. In the adjacent figure ABC is right angled at B. If AB = 4 and BC = 3 and side AC slide along the
coordinate axes in such a way that ‘B’ always remains in the first quadrant, then B always lie on straight
line :

(a) y = x (b) 3y = 4x (c) 4y = 3x (d) x + y = 0


18. Vertices of a variable triangle are (3, 4), (5 cos , 5 sin , ) and (5 sin , – 5 cos  ) where   R, then
locus of its orthocenter is :
(a) (x + y – 1)2 + (x – y – 7)2 = 100 (b) (x + y – 7)2 + (x – y – 1)2 = 100
2 2
(c) (x + y – 7) + (x + y – 1) = 100 (d) (x + y – 7)2 + (x – y + 1)2 = 100
19. Consider the triangle OAB where O = (0, 0) B (3, 4). If orthocenter of triangle is H (1, 4), then coordi-
nates of ‘A is :
15 17 21 19
(a)  0,  (b)  0,  (c)  0,  (d)  0, 
 4  4  4  4

MOMENTUM : Betiahata Chowk, Gorakhpur PH. 0551-2332808, 2205569 Page # 74


20. On the portion of the straight line, x + 2y = 4 intercepted between the axes, a square is constructed on the
side of the line away from the origin. Then the point of intersection of its diagonals has co-ordinates :
(a) (2, 3) (b) (3, 2) (c) (3, 3) (d) (2, 2)
21. Through a point A on the x-axis a straight line is drawn parallel to y-axis so as to meet the pair of straight
lines ax2 + 2hxy + by2 = 0 in B and C. If AB = BC then :
(a) h2 = 4ab (b) 8h2 = 9ab (c) 9h2 = 8ab (d) 4h2 = ab
22. Suppose that a ray of light leaves the point (3, 4), reflects off the y-axis towards the x-axis, reflects off the
x-axis, and finally arrives at the point (8, 2). The value of x, is :

1 1 2 1
(a) x  4 (b) x  4 (c) x  4 (d) x  5
2 3 3 3
23. Given a triangle whose vertices are at (0, 0), (4, 4) and (10, 0). A square is drawn in it such that its base is
on the x-axis and its two corners are on the 2 sides of the triangle. The area of the square is equal to :
400 400 625 625
(a) (b) (c) (d)
49 25 16 49
24. A, B and C are points in the xy-plane such that A (1, 2); B (5, 6) and AC = 3BC. Then :
(a) ABC is a unique triangle
(b) There can be only two such triangles
(c) No such triangle is possible
(d) There can be infinite number of such triangles
25. A ray of light passing through the point A (1, 2) is reflected at a point B on the x-axis and then passes
through (5, 3). Then the equation of AB is :
(a) 5x + 4y = 13 (b) 5x – 4y = –3 (c) 4x + 5y = 14 (d) 4x – 5y = –6
26. Vertices of a parallelogram ABCD are A (3, 1), B (13, 6), C (13, 21) and D (3, 16). If a line passing
through the origin divides the parallelogram into two congruent parts then the slope of the line is :
11 11 25 13
(a) (b) (c) (d)
12 8 8 8
27. If the vertices P and Q of a triangle PQR are given by (2, 5) and (4, –11) respectively, and the point R
moves along the line N : 9x + 7y + 4 = 0, then the locus of the centroid of the triangle PQR is a straight line
parallel to :
(a) PQ (b) QR (c) RP (d) N
28. In a triangle ABC, if A (2, –1) and 7x – 10y + 1 = 0 and 3x – 2y + 5 = 0 are equations of an altitude and
an angle bisector respectively drawn from B, then equation of BC is :
(a) x + y + 1 = 0 (b) 5x + y + 17 = 0 (c) 4x + 9y + 30 = 0 (d) x – 5y – 7 = 0
29. In an isosceles right angled triangle, a straight line drawn from the mid-point of one of equal sides to the
opposite angle. It divides the angle into two parts,  and ( / 4  ). Then tan  and tan [( / 4)  ] are
equal to :
1 1 1 1 1 1
(a) , (b) , (c) , (d) None of these
2 3 3 4 5 6

MOMENTUM : Betiahata Chowk, Gorakhpur PH. 0551-2332808, 2205569 Page # 75


30. The line (p + 2q) x + (p – 3q) y = p – q, for different values of p and q, passes through the fixed point :
 3 5  2 2  3 2  2 3
(a)  ,  (b)  ,  (c)  ,  (d)  , 
 2 2  5 5  5 5  5 5

31. The orthocentre of a triangle whose vertices are (0, 0), ( 3, 0) and (0, 6 ) is :
(a) (2, 1) (b) (3, 2) (c) (4, 1) (d) None of these
32. If the line y = mx meets the lines x + 2y – 1 = 0 and 2x – y + 3 = 0 at the same point, then m is equal to :
(a) 1 (b) –1 (c) 2 (d) –2
33. The distance of any point (x, y) from the origin is defined as d = max {P | x |, | y |}, then the distance of the
common point for the family of lines x (1 +  ) + y + 2 +  = 0 (  being parameter) from origin is :
(a) 1 (b) 2 (c) 5 (d) 0
34. Let ax + by + c = 0 be a variable straight line, where a, b and c are 1 , 3 and 7th terms of some increasing
st rd

A.P. Then the variable straight line always passes through a fixed point which lies on :
(a) x2 + y2 = 13 (b) x2 + y2 = 5 (c) y2 = 9x (d) 3x + 4y = 9
35. Area of the triangle formed by the line x + y = 3 and angle bisector of the pair of straight lines x2 – y2 + 2y
– 1 = 0 is :
(a) 2 sq. units (b) 4 sq. units (c) 6 sq. units (d) 8 sq. units
36. The number of integral values of m, for which the x-coordinate of the point of intersection of the lines 3x +
4y = 9 and y = mx + 1 is also an integer is :
(a) 2 (b) 0 (c) 4 (d) 1
37. A line passes through (1, 0). The slope of the line, for which its intercept between y = x – 2 and y = –x +
2 subtends a right angle at the origin, is :
(a)  2/3 (b)  3/2 (c)  1 (d) None of these
38. A variable straight line passes through a fixed point (a, b) intersecting the coordinate axes at A & B. If ‘O’
is the origin, then the locus of centroid of triangle OAB is :
(a) bx + ay – 3xy = 0 (b) bx + ay – 2xy = 0
(c) ax + by – 3xy = 0 (d) ax + by – 2xy = 0
39. Two vertices of a triangle are (5, –1) and (–2, 3). If orthocenter of the triangle is origin, then the co-
ordinates of third vertex is :
(a) (4, 7) (b) (3, 7) (c) (–4, –7) (d) (4, –7)
40. The straight line y = x – 2 rotates about a point where it cuts the x-axis and becomes perpendicular to the
straight line ax + by + c = 0. Then its equation is :
(a) ax + by + 2a = 0 (b) ax – by – 2a = 0
(c) bx + ay – 2b = 0 (d) ay – bx + 2b = 0
41. It is desired to construct a right angled triangle ABC ( C   / 2) in xy-plane so that its sides are parallel
to co-ordinates axes and the medians through A and B lie on the lines y = 3x + 1 and y = mx + 2
respectively. The values of ‘m’ for which such a triangle is possible is/are :
(a) –12 (b) 3/4 (c) 4/3 (d) 1/12
42. The medians AD and BE of a triangle ABC with vertices A (0, b), B (0, 0) and C (a, 0) are perpendicular
to each other if :
(a) b   2 a (b) a   2 b (c) b   3 a (d) b   3 b
o
43. The equations of the lines through (–1, –1) and making angle 45 with the line x + y = 0 are given by :
(a) x2 – xy + x – y = 0 (b) xy – y2 + x – y = 0
(c) xy + x + y = 0 (d) xy + x + y + 1 = 0
44. If the lines x + y + 1 = 0; 4x + 3y + 4 = 0 and x + y    0, where  2   2  2, are concurrent
then :
(a)   1,   1 (b)   1,   1 (c)   1,   1 (d)   1,   1
MOMENTUM : Betiahata Chowk, Gorakhpur PH. 0551-2332808, 2205569 Page # 76
45. The straight line, ax + by = 1 makes with the curve px2 + 2axy + qy2 = r a chord which subtends a right
angle at the origin. Then :
(a) r (a2 + b2) = p + q (b) r (a2 + p2) = q + b
2 2
(c) r (b + q ) = p + a (d) none of these
46. Given the family of lines, a (2x + y + 4) + b (x – 2y – 3) = 0. Among the lines of the family, the number of
lines situated at a distance of 10 from the point M (2, –3) is :
(a) 0 (b) 1 (c) 2 (d) 
47. The area enclosed by the graphs of | x + y | = 2 and | x | = 1 is :
(a) 2 (b) 4 (c) 6 (d) 8
48. The ends of the base of an isosceles triangle are at (2, 0) and (0, 1) and the equation of one side is x = 2
then the orthocentre of the triangle is :
 3 3 5  3  4 7 
(a)  ,  (b)  ,1 (c)  ,1 (d)  , 
 4 2 4  4   3 12 
49. The equation of the pair of bisectors of the angles between two straight lines is, 12x2 – 7xy – 12y2 = 0. If
the equation of one line is 2y – x = 0 then the equation of the other line is :
(a) 41x – 38y = 0 (b) 11x + 2y = 0 (c) 38x + 41y = 0 (d) 11x – 2y = 0
50. A piece of cheese is located at (12, 10) in a coordinate plane. A mouse is at (4, –2) and is running up the
line y = –5x + 18. At the point (a, b), the mouse starts getting farther from the cheese rather than closer to
it. The value of (a + b) is :
(a) 6 (b) 10 (c) 18 (d) 14
51. If L is the line whose equation is ax + by = c. Let M be the reflection of L through the y-axis, and let N be
the reflection of L through the x-axis. Which of the following must be true about M and N for all choices
of a, b and c ?
(a) The x-intercepts of M and N are equal
(b) The y-intercepts of M and N are equal
(c) The slopes of M and N are equal
(d) The slopes of M and N are reciprocal
52. The line x = c cuts the triangle with corners (0, 0), (1, 1) and (9, 1) into two regions. For the area of the
two regions to be the same c must be equal to :
(a) 5/2 (b) 3 (c) 7/2 (d) 3 or 15
53. The distance between the two parallel lines is 1 unit. A point ‘A’ is chosen to lie between the lines at a
distance ‘d’ from one of them. Triangle ABC is equilateral with B on one line and C on the other parallel
line. The length of the side of the equilateral triangle is :

2 2 d2  d  1
(a) d  d 1 (b) 2 (c) 2 d 2  d  1 (d) d 2  d  1
3 3
54. If m and b are real numbers and mb > 0, then the line whose equation is y = mx + b cannot contain the
point :
(a) (0, 2008) (b) (2008, 0) (c) (0, –2008) (d) (20, –100)
55. Given A (0, 0) and B (x, y) with x  (0,1) and y > 0. Let the slope of the line AB equals m1. Point C lies
on the line x = 1 such that the slope of BC equals m2 where 0 < m2 < m1. If the area of the triangle ABC
can be expressed as (m1 – m2) f (x), then the largest possible value of f (x) is :
(a) 1 (b) 1/2 (c) 1/4 (d) 1/8
56. What is the y-intercept of the line that is parallel to y = 3x, and which bisects the area of a rectangle with
corners at (0, 0) (4, 0), (4, 2) and (0, 2) ?
(a) (0, –7) (b) (0, –6) (c) (0, –5) (d) (0, –4)

MOMENTUM : Betiahata Chowk, Gorakhpur PH. 0551-2332808, 2205569 Page # 77


57. The vertex of right angle of a right angled triangle lies on the straight line 2x + y – 10 = 0 and the two other
vertices, at points (2, –3) and (4, 1) then the area of triangle in sq. units is :
33
(a) 10 (b) 3 (c) (d) 11
5
58. Given A = (1, 1) and AB is any line through it cutting the x-axis in B. If AC is perpendicular to AB and
meets the y-axis in C, then the equation of locus of mid-point P of BC is :
(a) x + y = 1 (b) x + y = 2 (c) x + y = 2xy (d) 2x + 2y = 1
59. The number of possible straight lines, passing through (2, 3) and forming a triangle with coordinate axes,
whose area is 12 sq. Units, is :
(a) one (b) two (c) three (d) four
60. Let A = (3, 2) and B = (5, 1). ABP is an equilateral triangle is constructed one the side of AB remote from
the origin then the orthocentre of triangle ABP is :
 1 3   1 3 
(a)  4  3,  3  (b)  4  3,  3 
 2 2   2 2 

 1 3 1   1 3 1 
(c)  4  3,  3 (d)  4  3,  3
 6 2 3   6 2 3 

OBJECTIVE (MORE THAN ONE) :


1. The area of triangle ABC is 20 cm2. The co-ordinates of vertex A are (– 5, 0) and B are (3, 0). The vertex
C lies on the line, x – y = 2. The co-ordinates of C are
(a) (5, 3) (b) (– 3, – 5) (c) (– 5, – 7) (d) (7, 5)
2. Three vertices of a triangle are A(4, 3); B(1, –1) and C(7, k). Value(s) of k for which centroid, orthocentre,
incentre and circumcentre of the ABC lie on the same straight line is/are :
(a) 7 (b) – 1 (c) – 19/8 (d) none
3. The equation of the altitude of the ABC whose vertices are A(–4, 2); B(6, 5) and C(1, – 4) can be
(a) 10x + 3y + 2 = 0 (b) 5x + 9y + 2 = 0
(c) 6x – 5y = 0 (d) 5x – 6y = 0
4. Two vertices of the ABC are at the points A(–1, –1) and B(4, 5) and the third vertex lines on the straight
line y = 5(x – 3). If the area of the  is 19/2 then the possible co-ordinates of the vertex C are :
(a) (5, 10) (b) (3, 0) (c) (2, – 5) (d) (5, 4)
x y x y
5. Line   1 cuts the co-ordinate axes at A(1, 0) and B(0, b) and the line   1 at A'(–a', 0) and
a b a  b
B'(0, –b'). If the points A, B, A', B' are concyclic then the orthocentre of the triangle ABA' is
 aa    bb 
(a) (0, 0) (b) (0, b') (c)  0,  (d)  0, 
 b   a 
6. A line passes through the origin and makes an angle of  / 4 with the line x – y + 1 = 0. Then :
(a) equation of the line is x = 0
(b) the equation of the line is y = 0
(c) the point of intersection of the line with the given line is (– 1, 0)
(d) the point of intersection of the line with the given line is (0, 1)

MOMENTUM : Betiahata Chowk, Gorakhpur PH. 0551-2332808, 2205569 Page # 78


1
7. Equation of a straight line passing through the point (2, 3) and inclined at an angle of arc tan with the line
2
y + 2x = 5 is
(a) y = 3 (b) x = 2
(c) 3x + 4y – 18 = 0 (d) 4x + 3y – 17 = 0
8. Consider the equation y – y1 = m (x – x1). If m and x1 are fixed and different lines are drawn for different
values of y1, then :
(a) the lines will pass through a fixed point (b) there will be a set of parallel lines
(c) all the lines intersect the line x = x1 (d) all the lines will be parallel to the line y = x1.
9. If one vertex of an equilateral triangle of side ‘a’ lies at the origin and the other lies on the line x  3 y  0
then the co-ordinates of the third vertex are :
 3a a   3a a 
(a) (0, a) (b)  2 ,  2  (c) (0, – a) (d)  2 , 2 
 
   
x y x y x y
10. If   1 is a line through the intersection of   1 and   1 and the lengths of the perpen-
c d a b b a
diculars drawn from the origin to these lines are equal in lengths then :
1 1 1 1 1 1 1 1
(a) 2
 2  2 2 (b) 2
 2  2 2
a b c d a b c d
1 1 1 1
(c)    (d) none
a b c d
11. The sides of a triangle are the straight lines x + y = 1 ; 7y = x and 3 y + x = 0. Then which of the following
is an interior point of the triangle ?
(a) circumcentre (b) centroid (c) incentre (d) orthocentre
12. If a2 + 9b2 – 4c2 = 6ab then the family of lines ax + by + c = 0 are concurrent at :
(a) (1/2, 3/2) (b) (– 1/2, – 3/2) (c) (– 1/2, 3/2) (d) (1/2, – 3/2)
13. The straight lines x + y = 0, 3x + y – 4 = 0 and x + 3y – 4 = 0 form a triangle which is
(a) isosceles (b) right angled (c) obtuse angled (d) equilateral
14. The x-coordinates of the vertices of a square of unit area are the roots of the equation x2 – 3 | x | + 2 = 0
and the y-coordinates of the vertices are the roots of the equation y2 – 3y + 2 = 0 then the possible vertices
of the square is/are :
(a) (1, 1), (2, 1), (2, 2), (1, 2) (b) (– 1, 1), (– 2, 1), (– 2, 2) (– 1, 2)
(c) (2, 1), (1, – 1), (1, 2), (2, 2) (d) (– 2, 1), (–1, –1), (– 1, 2) (– 2, 2)
15. P(x, y) moves such that the area of the triangle formed by P, Q(a, 2a) and R(– a, – 2a) is equal to the area
of the triangle formed by P, S(a, 2a) and T(2a, 3a). The locus of 'P' is a straight line given by
(a) 3x – y = a (b) 5x – 3y + a = 0 (c) 5x – 5y + a = 0 (d) 2y = ax
16. Let u  ax  by  a 3 b  0 v  bx  ay  b 3 a  0 a, b  R be two straight lines. The equation of the
bisectors of the angle formed by k1u – k2v = 0 and k1u + k2v = 0 for non zero real k1 and k2 are
(a) u = 0 (b) k2u + k1v = 0 (c) k2u – k1v = 0 (d) v = 0
2m 2m
17. The bisectors of angle between the st. lines, y  b  (x  a) and y  b  (x  a) are
1 m 2
1  m 2
(a) (y – b) (m + m') + (x – a) (1 – m m') = 0 (b) (y – b) (m + m') – (x – a) (1 – m m') = 0
(c) (y –b) (1 – m m') + (x – a) (m + m') = 0 (d) (y – b) (1 – m m') – (x – a) (m + m') = 0

MOMENTUM : Betiahata Chowk, Gorakhpur PH. 0551-2332808, 2205569 Page # 79


18. If the vertices, P, Q, R of a triangle PQR are rational points, which of the following points of the triangle
PQR is/are always rational point(s) ?
(a) centriod (b) incentre (c) circumcentre (d) orthocentre
19. Two equal sides of an isosceles triangle are given by the equations 7x – y + 3 = 0 and x + y – 3 = 0 and its
third side passes through the point (1, – 10). The equation of the third side can be
(a) x + 3y + 29 = 0 (b) x – 3y = 31 (c) 3x – y = 13 (d) 3x + y + 7 = 0
20. Straight lines 2x + y = 5 and x – 2y = 3 intersect at the point A. Points B and C are chosen on these two
lines such that AB = AC. Then the equation of a line BC passing through the point (2, 3) is
(a) 3x – y – 3 = 0 (b) x + 3y – 11 = 0 (c) 3x + y – 9 = 0 (d) x – 3y + 7 = 0
21. The lines L1 and L2 denoted by 3x2 + 10xy + 8y2 + 14x + 22y + 15 = 0 intersect at the point P and have
gradients m1 and m2 respectively. The acute angles between them is  . Which of the following relations
hold good?
(a) m1 + m2 = 5/4
(b) m1m2 = 3/8
 2 
(c) acute angle between L1 and L2 is sin 1  
5 5
(d) sum of the abscissa and ordinate of the point P is – 1.
22. Leb B (1, – 3) and D (0, 4) represent two vertices of rhombus ABCD in (x, y) plane, then coordinates of
vertex A if  BAD  60o can be equal to

1 7 3 1 3  1 7 3 1 3 
(a)  ,  (b)  , 
 2 2   2 2 

 1  14 3 1  2 3   1  14 3 1  2 3 
(c)  ,  (d)  , 
 2 2   2 2 
23. Two sides of a triangle have the joint equation (x – 3y + 2) (x + y – 2) = 0, the third side which is variable
always passes through the point (–5, –1), then possible values of slope of third side such that origin is an
interior point of triangle is/are :
4 2 1 1
(a) (b) (c) (d)
3 3 3 6
24. The equations of lines passing through point (2, 3) and having an intercept of length 2 units between the
lines 2x + y = 3 and 2x + y = 5 are :
(a) y = 3 (b) x = 2 (c) y = x + 1 (d) 4y + 3x = 18
25. Two sides of a rhombus ABCD are parallel to lines y = x + 2 and y = 7x + 3. If the diagonals of the
rhombus intersect at point (1, 2) and the vertex A is on the y-axis is, then the possible coordinates of A
are :
 5
(a)  0,  (b) (0, 0) (c) (0, 5) (d) (0, 3)
 2
26. Possible values of  for which the point (cos ,sin ) lies inside the triangle formed by lines x + y = 2 ;
x – y = 1 and 6x + 2y = 10 are :
  3 
(a) (b) (c) (d)
8 4 8 2

MOMENTUM : Betiahata Chowk, Gorakhpur PH. 0551-2332808, 2205569 Page # 80


27. Two equal sides of an isosceles triangle are given by the equations 7x – y + 3 = 0 and x + y – 3 = 0 and its
third side passes through the point, (1, –10), then equations of the third side can be :
(a) x – 3y – 31 = 0 (b) y – 3x + 13 = 0 (c) 3x + y + 7 = 0 (d) y – 2x + 12 = 0
2m 2m '
28. The bisectors of angle between the straight lines y  b  (x  a) and y  b  (x  a) are :
1 m 2
1 m ' 2
(a) (y – b) (m + m’) + (x – a) (1 – m m’) = 0 (b) (y – b) (m + m’) – (x – a) (1 – m m’) = 0
(c) (y – b) (1 – m m’) + (x – a) (m + m’) = 0 (d) (y – b) (1 – m m’) – (x – a) (m + m’) = 0
29. Straight lines 2x + y = 5 and x – 2y = 3 intersect at point A. Points B and C are chosen on these two lines
such that AB = AC. Then the equation of a line BC passing through the point (2, 3) is :
(a) 3x – y – 3 = 0 (b) x + 3y – 11 = 0 (c) 3x + y – 9 = 0 (d) x – 3y + 7 = 0
30. The sides of a triangle are the straight lines x + y = 1, 7y = x and 3y  x  0. Then which of the following
is an interior point of the triangle :
(a) circumcentre (b) centroid (c) incentre (d) orthocentre
31. If one vertex of an equilateral triangle of side ‘a’ lies at origin and the other lies on the line x – 3y  0, then
the coordinates of the third vertex are :
 3a a   3a a 
(a) (0, a) (b)  ,  (c) (0, –a) (d)   , 
 2 2  2 2
32. The area of triangle ABC is 20 cm2. The coordinates of vertex A are (–5, 0) and B are (3, 0). The vertex
C lies on the line x – y = 2. The coordinates of C are :
(a) (5, 3) (b) (–3, –5) (c) (–5, –7) (d) (7, 5)
33. Let B(1, –3) and D(0, 4) represent two vertices of rhombus ABCD in (x, y) plane, then coordinates of
vertex A if BAD  60o can be equal to :
 1 7 3 1 3   1 7 3 1 3 
(a)  , (b)  ,
 2 2   2 2 

 1  7 3 1  3   1  7 3 1  3 
(c)  , (d)  ,
 2 2   2 2 
34. Let L1 : 3x + 4y = 1 and L2 : 5x – 12y + 2 = 0 be two given lines. Let image of every point on L1 with
respect to a line L lies on L2 then possible equation of L can be :
(a) 14x + 112y – 23 = 0 (b) 64x – 8y –3 = 0
(c) 11x – 4y = 0 (d) 52y – 45x = 7
x y
35. If one diagonal of a square is the portion of line   1 intercepted by the axes, then the extremities of
a b
the other diagonal of the square are :
 a  b a  b  a  b a  b  a  b b a  a  b b a
(a)  , (b)  , (c)  , (d)  ,
 2 2   2 2   2 2   2 2 
36. The points A(0, 0), B(cos  ,sin  ) and C(cos ,sin ) are the vertices of a right angled triangle if :
    1     1
(a) sin    (b) cos   
 2  2  2  2

    1     1
(c) cos    (d) sin   
 2  2  2  2

MOMENTUM : Betiahata Chowk, Gorakhpur PH. 0551-2332808, 2205569 Page # 81


37. Let x1 and y1 be the roots of x2 + 8x – 2009 = 0; x2 and y2 be the roots of 3x2 + 24x – 2010 = 0 and x3
and y3 be the roots of 9x2 + 72x – 2011 = 0. The points A(x1, y2) B(x2, y2) and C(x3, y3) :
(a) can not lie on a circle (b) form a triangle of area 2 sq. units
(c) form a right-angled triangle (d) are collinear

COMPREHENSIONS :
Comprehenssion # 1
Let ABC be an acute angled triangle and AD, BE and CF are its medians, where E and F are the points
E(3, 4) and F(1, 2) respectively and centroid of  ABC is G(3, 2), then answer the following questions :

1. The equation of side AB is


(a) 2x + y = 4 (b) x + y – 3 = 0 (c) 4x – 2y = 0 (d) none of these
2. Coordinate of D are
(a) (7, –4) (b) (5, 0) (c) (7, 4) (d) (– 3, 0)
3. Height of altitude drawn from point A is (in units)
(a) 4 2 (b) 3 2 (c) 6 2 (d) 2 3

Comprehension # 2

Given two straight lines AB and AC whose equations are 3x + 4y = 5 and 4x – 3y = 15 respectively. Then
the possible equation of line BC through (1, 2), such that ABC is isosceles, is L1 = 0 or L2 = 0, then
answer the following questions
4. If L1  ax + by + c = 0 & L2  dx + ey + f = 0 where a, b, c, d, e, f  I, and a, d > 0, then c + f =
(a) 1 (b) 2 (c) 3 (d) 4
5. A straight line through P(2, c + f – 1), inclined at an angle of 60° with positive y axis. The coordinates of
one of the points on it at a distance (c + f) units from point P is (c, f obtained from previous question)
(a) (2 + 2 3 , 5) (b) (3 + 2 3 , 3) (c) (2 + 3 2 , 4) (d) (2 + 3 2 , 3)
6. If (a, b) is the coordinate of the point obtained in previous question, then find the equation of line which is
at the distance |b – 2a – 1| units from origin and make equal intercept on coordinate axis in first quadrant.
(a) x + y + 4 6 = 0 (b) x + y + 2 6 = 0 (c) x + y – 4 6 = 0 (d) x + y – 2 6 = 0

Comprehension # 3
Let ABCD is a square with sides of unit length. Points E and F are taken on sides AB and AD respectively
so that AE = AF. Let P be a point inside the square ABCD.
7. The maximum possible area of quadrilaterial CDFE is
1 1 5 3
(a) (b) (c) (d)
8 4 8 8
2 2 2 2
8. The value of (PA) – (PB) + (PC) – (PD) is equal to
(a) 3 (b) 2 (c) 1 (d) 0
9. Let a line passing through point A divides the square ABCD into two parts so that area of one portion is
double the other, then the length of portion of line inside the square is
10 13 11 2
(a) (b) (c) (d)
3 3 3 3
MOMENTUM : Betiahata Chowk, Gorakhpur PH. 0551-2332808, 2205569 Page # 82
Comprehension # 4
 13 
Let M  2,  is the circumcentre of  PQR whose sides PQ and PR are represented by the straight
 8
lines 4x – 3y = 0 and 4x + y = 16 respectively.
10. The orthocentre of  PQR is
7 4 4 7  3 3 
(a)  ,  (b)  ,  (c)  3,  (d)  ,3 
3 3 3 3  4 4 
11. If A, B and C are the midpoint of the sides PQ, OR and PR of  PQR respectively, then the are of ABC
equals
(a) 1 (b) 2 (c) 3 (d) 4
12. If PB be the median of the PQR , then the equation of the straight line passing through N(–2, 3) and
perpendicular to PB is
(a) 4x + y + 5 = 0 (b) x – 4y + 14 = 0 (c) 4x – y + 11 = 0 (d) x + 4y – 10 = 0

Comprehension # 5
In the diagram, a line is drawn through the points A(0, 16) and B(8, 0). Point
P is chosen in the first quadrant on the line through A and B. Points C and D
are chosen on the x and y-axis respectively, so that PDOC is a rectangle.
13. Perpendicular distance of the line AB from the point (2, 2) is
(a) 4 (b) 10
(c) 20 (d) 50
14. Sum of the coordinates of the point P if PDOC is a square is
32 16
(a) (b) (c) 16 (d) 11
3 3
15. Number of possible ordered pair(s) of all positions of the point P on AB so that the area of the rectangle
PDOC is 30 sq. units, is
(a) three (b) two (c) one (d) zero

Comprehension # 6
Consider a ABC whose sides BC, CA and AB are represented by the straight lines x – 2y + 5 = 0,
x + y + 2 = 0 and 8x – y – 20 = 0 respectively.
16. The are of ABC equals
41 43 45 47
(a) (b) (c) (d)
2 2 2 2
17. If AD be the median of the ABC then the equation of the straight line passing through (2, – 1) and
parallel to AD is
(a) 4x – 3y – 11 = 0 (b) 13x – 4y – 30 = 0 (c) 4x + 13y + 5 = 0 (d) 13x + 4y – 22 = 0
18. The orthocentre of the ABC is
 1 2  2 4
(a) (–3, 1) (b)   ,  (c) (–2, 4) (d)   , 
 3 3  3 3

MOMENTUM : Betiahata Chowk, Gorakhpur PH. 0551-2332808, 2205569 Page # 83


Comprehension # 7
An equilateral triangle ABC has its centroid at the origin and the base BC lies along the line x + y = 1
19. Area of the equilateral ABC is
3 3 3 3 3 2 2 3
(a) (b) (c) (d)
2 4 2 4
20. Gradient of the other two lines are
1
(a) 3, 2 (b) 3, (c) 2  1, 2  1 (d) 2  3, 2  3
3

Comprehension # 8
Consider two points A  (1, 2) and B  (3, 1) . Let M be a point on the straight line L  x  y  0 .
21. If M be a point on the line L = 0 such that AM + BM is minimum, then the reflection of M in the line x = y
is
(a) (1, – 1) (b) (– 1, 1) (c) (2, – 2) (d) (– 2, 2)
22. If M be a point on the line L = 0 such that |AM – BM| is maximum, then the distance of M from N  (1,1)
is
(a) 5 2 (b) 7 (c) 3 5 (d) 10
23. If M be a point on the line L = 0 such that |AM – BM| is maximum, then the area of AMB equals
13 13 13 13
(a) (b) (c) (d)
4 2 6 8

Comprehension # 9
Consider a family of lines (4a + 3)x – (a + 1)y – (2a + 1) = 0 where a  R
24. The locus of the foot of the perpendicular from the origin on each member of this family, is
(a) (2x – 1)2 + 4(y + 1)2 = 5 (b) (2x – 1)2 + (y + 1)2 = 5
2 2
(c) (2x + 1) + 4(y – 1) = 5 (d) (2x – 1)2 + 4(y – 1)2 = 5
25. A member of this family with positive gradient making an angle of  / 4 with the line 3x – 4y = 2, is
(a) 7x – y – 5 = 0 (b) 4x – 3y + 2 = 0
(c) x + 7y = 15 (d) 5x – 3y – 4 = 0
26. Minimum area of the triangle which a member of this family with negative gradient can make with the
positive semi axes, is
(a) 8 (b) 6 (c) 4 (d) 2

Comprehension # 10
Consider a trapezoid ABCD, one of whose non parallel sides AB which is 8cm long is perpendicular to the
base. The base BC and AD of trapezoid are 6cm and 10cm in lengths respectively. Let L1, L2, L3, L4
represent the lines AB, BC, CD and DA respectively and d(P, L) denote the perpendicular distance of
point P from line L.

27. Find the area of region inside the trapezoid ABCD in which the point Q can lie satisfying
d(Q, L4)  d(Q, L3) :
(a) 3(3 5  3) (b) 24( 3  1) (c) 4(5  5) (d) 25( 5  1)
28. Distance of the point R lying on line AD from vertex A so that perimeter of triangle RBC is minimum is :
(a) 2 (b) 3 (c) 4 (d) 5

MOMENTUM : Betiahata Chowk, Gorakhpur PH. 0551-2332808, 2205569 Page # 84


29. The maximum possible area of rectangle inscribed in the trapezoid so that one of its sides lies on the larger
base of trapezoid is :
(a) 36 (b) 54 (c) 42 (d) 48

Comprehension # 11
Consider a variable line ‘L’ which passes through the point of intersection ‘P’ of the lines 3x + 4y – 12 = 0
and x + 2y – 5 = 0 meeting the coordinate axes at points A and B.

30. Locus of the middle point of the segment AB has the equation :
(a) 3x + 4y = 4xy (b) 3x + 4y = 3xy (c) 4x + 3y = 4xy (d) 4x + 3y = 3xy
31. Locus of the feet of the perpendicular from the origin on the variable line ‘L’ has the equation :
(a) 2(x2 + y2) – 3x – 4y = 0 (b) 2(x2 + y2) – 4x – 3y = 0
(c) x2 + y2 – 2x – y = 0 (d) x2 + y2 – x – 2y = 0
32. Locus of the centroid of the variable triangle OAB has the equation (where ‘O’ is the origin) :
(a) 3x + 4y + 6xy = 0 (b) 4x + 3y – 6xy = 0
(c) 3x + 4y – 6xy = 0 (d) 4x + 3y + 6xy = 0

Comprehension # 12
Consider 3 non-collinear points A(9, 3), B(7, –1) and C(1, –1). Let P(a, b) be the centre and R is the
radius of circle ‘S’ passing through points A, B, C. Also H(x, y) are the coordinates of the orthocentre of
triangle ABC whose area be denoted by  .

33. If D, E and F are the middle points BC, CA and AB respectively then the area of the triangle DEF is :
(a) 12 (b) 6 (c) 4 (d) 3
34. The value of a + b + R equals :
(a) 3 (b) 12 (c) 13 (d) none of these
35. The ordered pair (x, y) is :
(a) (9, 6) (b) (–9, 6) (c) (9, –5) (d) (9, 5)

Comprehension # 13
Let A  (0, 0), B  (5, 0), C  (5, 3) and D  (0, 3) are the vertices of rectangle ABCD. If P is a variable
point lying inside the rectangle ABCD and d(P, L) denote perpendicular distance of point P from line L.

36. If d(P, AB)  min {d(P, BC), d(P, CD), d(P, AD)}, then area of the region in which P lies is :
17 19 21 23
(a) (b) (c) (d)
4 4 4 4
37. If d(P, AB)  max {d(P, BC), d(P, CD), d(P, AD)}, then area of the region in which P lies is :
1 3 1
(a) 1 (b) (c) (d)
2 4 4
2
 3
38. If  d(P, AB)    d(P, AD) 2  1, then area of region in which P lies is :
 2
 
(a) 15  2 (b) 10  (c) 15   (d) 15 
2 2

MOMENTUM : Betiahata Chowk, Gorakhpur PH. 0551-2332808, 2205569 Page # 85


Comprehension # 14
The equation of an altitude of an equilateral triangle is 3x  y  2 3 and one of its vertices is (3, 3)
then

39. The possible number of triangles is :


(a) 1 (b) 2 (c) 3 (d) 4
40. Which of the following can’t be the vertex of the triangle :
(a) (0, 0) (b) (0, 2 3) (c) (3,  3) (d) None of these
41. Which of the following can be the possible orthocentre of the triangle :
(a) (1, 3) (b) (0, 3) (c) (0, 2) (d) None of these

Comprehension # 15
Given point A(6, 30) and point B(24, 6), equation of line AB is 4x + 3y = 114. Point P(0,  ) is a point on
y-axis such that 0    38 and point Q(0, k) is a point on y-axis such that k > 38.

42. For all positions of point P, angle APB is maximum when point P is :
(a) (0, 12) (b) (0, 15) (c) (0, 18) (d) (0, 21)
43. The maximum value of angle APB is :
  2 3
(a) (b) (c) (d)
3 2 3 4
44. For all position of point Q, angle AQB is maximum when point Q is :
(a) (0, 54) (b) (0, 58) (c) (0, 60) (d) None of these

Comprehension # 16
OACB is a square on x-y plane where O is the origin. A line through A intersects the diagonal OC at D
internally, side OB at E internally and side CB at F externally. Given that AD : DE = 4 : 3, AD = 5 units and
the square lies completely in first quadrant.

45. The area of square will be :


(a) 36 (b) 42 (c) 49 (d) 82
46. The abscissa of F will be :
8 7 5 4
(a)  (b)  (c)  (d) 
3 3 3 3
47. Let O’ be the reflection of O along AD. The equation of circumcircle of AO ' E will be :
(a) x2 + y2 – 7x – 21y = 0 (b) 4(x2 + y2) – 7x – 21y = 0
(c) 4(x2 + y2 –7x) – 21y = 0 (d) x2 + y2 – 21x – 7y = 0

Comprehension # 17
A variable line ‘L’ is drawn through O(0, 0) to meet lines L1 : y – x – 10 = 0 and L2 : y – x – 20 = 0 at points
A and B respectively. A point P is taken on line ‘L’.
2 1 1
48. If   , then locus of P is :
OP OA OB
(a) 3x + 3y = 40 (b) 3x + 3y + 40 = 0 (c) 3x – 3y = 40 (d) 3y – 3x = 40
2
49. If OP = (OA) (OB), then locus of P is :
(a) (y – x)2 = 100 (b) (y – x)2 = 50 (c) (y – x)2 = 200 (d) (y – x)2 = 250

MOMENTUM : Betiahata Chowk, Gorakhpur PH. 0551-2332808, 2205569 Page # 86


1 1 1
50. If 2
  , then locus of P is :
OP (OA) (OB) 2
2

(a) (y – x)2 = 80 (b) (y – x)2 = 100 (c) (y – x)2 = 144 (d) (y – x)2 = 400

Comprehension # 18
A ray of light travelling along the line OP (O being origin) is reflected by the line mirror 2x – 3y + 1 = 0, the
point of incidence being P(1, 1). The reflected ray, travelling along PQ is again reflected by the line mirror
2x – 3y – 1 = 0, the point of incidence being Q, from Q ray move along QR, where R lies on the line
2x – 3y + 1 = 0

51. The equation of QR is :


(a) 13x – 13y = 20 (b) 13x – 13y + 20 = 0
(c) y = x – 1 (d) 13x – 13y + 17 = 0
52. The ordinate of point R is :
73 53 23
(a) (b) (c) (d) 1
13 13 13

COLUMN-MATCHING :
1. Column-I Column-II
(A) The points (2, –2), (8, 4), (5, 7) and (–1, 1) (P) square
taken in order consistute the vertices of a
(B) The points (0, –1), (2, 1), (0, 3) and (–2, 1) (Q) rectangle
taken in order are the vertices of a
(C) The points (3, –5), (–5, –4), (7, 10), (15, 9) (R) trapezium
taken in order are the vertices of a
(D) The points (–3, 4), (–1, 0), (1, 0) and (3, 4) (S) parallelogram
taken in order are the vertices of a (T) cyclic quadrilateral

2. Column-I Column-II
(a) The points (2, – 2), (– 2, 1) and (5, 2) (p) are the vertices of a right angled triangle
(b) The points (1, – 2), (– 3, 0) and (5, 6) (q) are the vertices of a right angled isosceles
triangle
(c) The points (3, 7), (6, 5) and (15, – 1) (r) are the vertices of an equilateral triangle
(d)  
The points (2, 2), (– 2, – 2) and 2 3, 2 3 (s) do not form a triangle

3. Column-I Column-II
(Equation of Straight Line)
(a) Which cuts-off an intercept 4 on the x-axis and (p) 2x + y + 1 = 0
through the point (2, –3).
(b) Which cuts-off equal intercepts on the co-ordinate (q) x+y=7
passes through (2, 5)
(c) Which makes an angle of 135o with the axis of x and which (r) 3x – 2y = 12
cuts the axis of y at a distance - 8 from the origin and.
(d) Through the point (4, 1) and making with the axes in first (s) x+y+8=0
quadrant a triangle whose are in 8.

MOMENTUM : Betiahata Chowk, Gorakhpur PH. 0551-2332808, 2205569 Page # 87


4. Column-I Column-II
(a) Four lines x + 3y – 10 = 0, x + 3y – 20 = 0 (p) a quadrilateral which is neither a
3x – y + 5 = 0 and 3x – y – 5 = 0 form a a parallelogram nor a trapezium
figure which is nor a kite.
(b) The point A(1, 2), B(2, –3), C(–1, –5) and (q) a parallelogram
D(–2, 4) in order are the vertices of
(c) The lines 7x + 3y – 33 = 0, 3x – 7y + 19 = 0 (r) a rectangle of area 10 sq. units
3x – 7y – 10 = 0 and 7x + 3y – 4 = 0 form a
figure which is
(d) Four lines 4y – 3x – 7 = 0, 3y – 4x + 7 = 0, (s) a square
4y – 3x – 21 = 0, 3y – 4x + 14 = 0 form a
figure which is

5. Set of family of lines are described in column-I and their mathematical equation are given in column-II.
Match the entry of column-I with suitable entry of column-II, (m and a are parameters)
Column-I Column-II
(a) having gradient 3 (p) mx – y + 3 – 2m = 0
(b) having y intercept three times the x-intercept (q) mx – y + 3m = 0
(c) having x intercept (–3) (r) 3x + y = 3a
(d) concurrent at (2, 3) (s) 3x – y + a = 0

6. Let D(0, 3), E(1, 0), F(–1, 0) be the feet of perpendiculars dropped from vertices A, B, C to opposite
sides BC, CA, AB respectively of triangle ABC
Column-I Column-II
(a) The ratio of the inradius of ABC to the inradius (p) 2
of DEF is
(b) Let ‘H’ be the orthocentre of ABC, then the (q) 3
greatest integer which is less than or equal to square
of the length AH is
(c) The square of the sum of ordinates of points A, B and C is (r) 4
(d) The length of side AB of ABC is (s) 5

7. Column-I Column-II
(a) If a, b, c are in A.P., then lines ax + by + c = 0 are (p) (–4, –7)
concurrent at
(b) A point on the line x + y = 4 which lies at a unit distance (q) (–7, 11)
from the line 4x + 3y = 10 is
(c) Orthocentre of triangle made by lines x + y = 1, (r) (1, –2)
x – y + 3 = 0, 2x + y = 7 is
(d) Two vertices of a triangle are (5, –1) and (–2, 3). If (s) (–1, 2)
orthocentre is the origin then coordinates of the third vertex are

MOMENTUM : Betiahata Chowk, Gorakhpur PH. 0551-2332808, 2205569 Page # 88


8. Column-I Column-II
(a) The number of integral values of ‘a’ for which point (p) 0
(a, a2) lies completely inside the triangle formed by
lines x = 0, y = 0, 2y + x = 3.
K
(b) The number of values of a of the form where (q) 1
3
K  I so that point (a, a2) lies between the lines
x + y = 2 and 4x + 4y – 3 = 0
(c) The reflection of point (t – 1, 2t + 2) in a line is (2t + 1, t) (r) 2
then the slope of line is
(d) In a triangle ABC, the bisector of angles B and C lie along (s) 4
the lines y = x and y = 0. If A is (1, 2) then 10 d(A, BC)
equals (where d(A, BC) denotes the perpendicular distance
of A from BC.)

9. Vertex A of the ABC is at origin. The equation of medians through B and C are 15x – 4y – 240 = 0 and
15x – 52y + 240 = 0 respectively.
Column-I Column-II
 56 
(a) The coordinates of incenter of ABC are (p)  3 ,10

(b) The coordinates of centroid of ABC are (q) (21, 12)


(c) The coordinates of excenter opposite to vertex (r) (12, 21)
C of ABC are
(d) The coordinates of orthocenter of ABC are (s) (–4, 7)
(t) (0, 63)

INTEGER TYPE :

1. P(3, 1), Q(6, 5) and R(x, y) are three points such that angle PRQ is right angle and the area of PRQ is
7, then number of such points R is.
2. The number of integral values of a for which the point P(a2, a) lies in the region corresponding to the acute
angle between the lines 2y = x and 4y = x is.
3. The number of integral values of b for which the origin and the point (1, 1) lie on the same side of straight
line a2x + aby + 1 = 0 for a  R  {0} is.
4. If the pair of lines 6x2 – xy – 3y2 – 24x + 3y +  = 0 intersect on x-axis, then find the value of 20    .
5. If n1 is the number of points on the line 3x + 4y = 5 which is at distance of 1 + sin2  units from (2, 3) and
n2 denotes the number of points on the line 3x + 4y = 5 which is at distance of sec 2   2 cos ec2 units
from (1, 3), then find the sum of roots of equations n 2 x 2  6x  n1  0.
6. In a ABC, the vertex A is (1, 1) and orthocenter is (2, 4). If the sides AB and BC are members of the
family of straight lines ax + by + c = 0. Where a, b, c are in A.P. then the coordinates of vertex C are (h, k).
Find the value of 2h + 12k.

MOMENTUM : Betiahata Chowk, Gorakhpur PH. 0551-2332808, 2205569 Page # 89


7. Let P be any point on the line x – y + 3 = 0 and A be a fixed point (3, 4). If the family of lines given by the
equations (3 sec   5 cosec ) x  (7 sec   3 cosec )y  11(sec   cosec )  0 are concurrent at a
point B for all permissible values of  and maximum value of | PA – PB | = 2 2n (n  N), then find the
value of n.
8. There exists two ordered triplets (a1, b1, c1) and (a2, b2, c2) for (a, b, c) for which the equation 4x2 – 4xy
+ ay2 + bx + cy + 1 = 0 represents a pair of identical straight lines in x-y plane. Find the value of a1 + b1
+ c1 + a2 + b2 + c2.
9. Each side of a square is of length 4 units. The center of the square is at (3, 7) and one of the diagonals is
parallel to the line y = x. If the vertices of the square be (x1, y1), (x2, y2), (x3, y3) and (x4, y4) then find the
value of max (y1, y2, y3, y4) – min(x1, x2, x3, x4).
10. The base of an isosceles triangle is the intercept made by the line x + 2y = 4 with the coordinate axes. If the
equations of the equal sides be x = 4 and y = mx + c then find the value of 8m + c.
11. The slope of one of lines given by ax2 + 2hxy + by2 = 0 be the square of the slope of the other, if ab(a + b)
+ bh  h 3  0, then    is equals.
12. The equation of a line through the mid point of the sides AB and AD of rhombus ABCD, whose one
diagonal is 3x – 4y + 5 = 0 and one vertex is A(3, 1) is ax + by + c = 0. Find the absolute value of (a + b
+ c) where a, b, c are integers expressed in lowest form.
13. If there a real value of  for which the image of point ( ,   1) by the line mirror 3x + y = 6 is the point
( 2  1,  ) ? Then find  .
14. Consider two lines L1  x – y = 0 and L  x + y = 0 and a moving point P(x, y). Let d(P, L1), i =1, 2
represents the perpendicular distance of the point P from L1. If point P moves in certain region R in such a
2
A
way that  d(P, L ) [2, 4]. Let the area of region R is A, then find
i 1
1
4
.

15. In a ABC, A  (, ), B(1, 2), C(2, 3) and point A lies on line y = 2x + 3, where  ,   I. If the area of
ABC be such that area of triangle lies in interval [2, 2). Find the number of all possible coordinates of A.
16. Consider ABC with A (m, – m – 1), B(–1, 0), C(l, l + 1) is such that a line of slope 2, drawn through
centroid of ABC meets the circumcentre of ABC on y-axis, then find the value of l + m.

MOMENTUM : Betiahata Chowk, Gorakhpur PH. 0551-2332808, 2205569 Page # 90


Answer Key
EXERCISE-1 ELEMENTARY EXERCISE
2. 1 : 3 internally 3. (4, 5) 4. (3, –3) 5. (7, 2) or (1, 0) 6. (i) 15 sq. units, (ii) |ab| sq. units
7. y = ± 2x 8. (3x – 1) + 9y2 = a2 + b2
2
10. x – y – 5 = 0, x + y + 5 = 0
11. AB : 2x – 3y + 15 = 0, 2 13 BC : 2x + y – 5 = 0, 4 5 , CA : 6x – y – 27 = 0, 2 37 12. 3, – 1
3

 
 1 23  x2 y 1 
 ,      r
13. 14.  1 1  15. 2/5 17. 3 sq. unit 18. x – 2y + 1 = 0 20. k = 2
 6 9 
 2 2 
 

1 1 1
21. 2
 2  24. 2 25. 5/3  7/2 26. 3x – y – 11 = 0
x y 4
27. (x – 4 y + 3) (x – y) = 0 or x2 – 5 x y + 4 y2 + 3 x – 3 y = 0 28. (1, - 2), yes (1/3, - 2/3)
3  9  3 3 9 3
29.  2 , 0   4 , 0  ,  2 , 4  ,  4 , 4  30. x – y = 0 32. 2x – y + 3 = 0, 2x + y – 7 = 0; x – 2y – 6 = 0
      

FG 8 IJ , (b) 4 FG 3 , 22 IJ 31
33. (a) 2,
H 3K 34. H7 7 K 35. (33, 26) 36. K = 7 or
9
37. 1 : 2 ; Q(–5, –3)

FG 7 , 13IJ or FG  3 , 3IJ
38. 83x – 35y + 92 = 0 39. 2x + y – 1 = 0 40. H 2 2 K H 2 2K 42. x – y = 0 43. 91 sq. units

44. 6 units 45. a2 + b2 = a'2 + b'2 ; (a + a')x + (b + b')y + (c + c') = 0; (a – a')x + (b – b')y = 0

46. 3 units 47. C = – 4; B(2, 0); D(4, 4) 48. x  5y  5 2  0 or x  5y  5 2  0

1  10
49. x – 3y – 31 = 0 or 3x + y + 7 = 0 50. 2 x  2 y  2  0 51. 52. x – 5 = 0
3
5
53. 7 54. x + y = 1; x + 9y = 1 55. 0     tan 1 3 56. 533
6

FG 1 , 2 IJ
57. (i) area = 6 sq. units. (ii) diagonals are 5 & 53 . 59. a = 11, c = 78 60. (1, –2), yes
H 3 3K
FG 5 IJ
62. 7x + 24y + 182 = 0 or x = – 2 64. (0, 0) or 0,
H 2K
3 3 FG IJ
66. 3x + 6y – 16 = 0; 8x + 8y + 7 = 0; 12x + 6y – 11 = 0
2 H
sq. units, 3,3, , isosceles
68.
4 K
69. 6x2 – xy – y2 – x – 12y – 35 = 0 70. 2x – y + 3 = 0, 2x + y – 7 = 0, x – 2y – 6 = 0
71. y2 = a(x – 3a)

MOMENTUM : Betiahata Chowk, GKP. PH. 0551- 2332808, 2205569 Page # 91


EXERCISE-2 (MAIN EXERCISE)
STRAIGHT LINES
1.C 2.B 3.B 4.A
5.A 6.B 7.A 8.A 9.B 10.D 11.A 12.A 13.D
14.D 15.A 16.C 17.A
18.A 19.D 20.B 21.B 22.A 23.A 24.B 25.B 26.C
27.C 28.C 29.C 30.B
31.A 32.A 33.A 34.B 35.C 36.B 37.A 38.D 39.C
40.C 41.C 42.A 43.B
44.D 45.C 46.B 47.B 48.A 49.D 50.D 51.B 52.A
53.B 54.A 55.B 56.C
57.A 58.A 59.A 60.A 61.A 62.C 63.A 64.D 65.A
66.C 67.C 68.D 69.A
70.B 71.D 72.C 73.D 74.C 75.C 76.D 77.A 78.C
79.D 80.D 81.A 82.A
83.D 84.B 85.B 86.C 87.A 88.C 89.D 90.A 91.D
92.B 93.D 94.B 95.B
96.A 97.A 98.C 99.D 100.B 101.B 102.A 103.B 104.B
105.B 106.D 107.B 108.B
109.D 110.D 111.B 112.A 113.A 114.C 115.B 116.B 117.D
118.A 119.A 120.B 121.B
122.B 123.D 124
PAIR OF STRAIGHT LINES
1.B 2.D 3.C 4.B 5.A 6.B 7.A 8.C 9.C 10.C 11.A 12.B 13.B
14.A 15.D 16.A 17.D 18.A 19.C 20.D 21.A 22.A 23.CD 24.A 25.B 26.A
27.C 28.A 29.C 30.D 31.D 32.ABC 33.AB 34.A 35.AC 36.B 37.AC 38.D
39.AC 40.ABD 41.B 42.C 43.B 44.A 45.A 46.AC 47.A 48.C 49.C 50.A
51.C 52.A 53.BC 54.ABCD 55.A 56.A 57.D 58.C 59.B 60.A 61.AB 62.A
63.BC 64.B 65.A 66.A 67.D

EXERCISE-3 (ADVANCED EXERCISE)


OBJECTIVE (SINGLE CORRECTS) :
1.B 2.C 3.A 4.A 5.A 6.C 7.C 8.B 9.B 10.B 11.A 12.B 13.C
14.B 15.C 16.C 17.B 18.D 19.D 20.C 21.B 22.B 23.A 24.D 25.A 26.B
27.D 28.B 29.A 30.D 31.D 32.B 33.B 34.A 35.A 36.A 37.D 38.A 39.C
40.D 41.B 42.B 43.D 44.D 45.A 46.B 47.D 48.B 49.A 50.B 51.C 52.B
53.B 54.B 55.D 56.C 57.B 58.A 59.C 60.D
MORE THAN ONE :
1.BD 2.BC 3.ABD 4.BC 5.BC 6.ABCD 7.BC
8.BC 9.ABCD 10.AC 11.BC 12.CD 13.AC 14.AB
15.AB 16.AD 17.AD 18.ACD 19.BD 20.AB 21.BCD
22.AB 23.BCD 24.BD 25.AB 26.ABC 27.AC 28.AD
29.AB 30.BC 31.ABCD 32.BD 33.AB 34.AB 35.AC
36.ACD 37.AD
Comprehension Type Questions:
1.A 2.B 3.C 4.D 5.A 6.C 7.C 8.D 9.B 10.C 11.B 12.D 13.C
14.A 15.B 16.C 17.D 18.B 19.A 20.D 21.B 22.D 23.A 24.D 25.A 26.C
27.D 28.B 29.D 30.A 31.B 32.C 33.D 34.B 35.C 36.C 37.D 38.D 39.B
40.D 41.A 42.C 43.B 44.B 45.C 46.B 47.C 48.D 49.C 50.A 51.A 52.B
Matrix Match :
1. A-QST; B-PQST; C-S; D-RT 2. A-PQ; B-P; C-S; D-R 3. A-R; B-Q; C-T; D-S
4. A-QRS; B-P; C-QS; D-Q 5. A-S; B-R; C-Q; D-P 6. A-P; B-S; C-Q; D-R
7. A-R; B-Q; C-S; D-P 8. A-P; B-R; C-Q; D-S 9. A-Q; B-P; C-S; D-T
INTEGER TYPE :
1.(0) 2.(1) 3.(3) 4.(6) 5.(3) 6.(14) 7.(5) 8.(2) 9.(8) 10.(8) 11.(2) 12.(1) 13.(2)
14.(6) 15.(4) 16.(0)
MOMENTUM : Betiahata Chowk, GKP. PH. 0551- 2332808, 2205569 Page # 92

You might also like